You are on page 1of 336

‫‪Prometric exam collection 2017‬‬

‫)النسخة الرابعة)‬

‫ودالشمال‪4‬‬

‫ودالشمال‪4‬‬
‫مع تمنياتي بالتوفيق والنجاح للجميع‬
‫ودالشمال‪2017‬م‬

‫مناقشة امتحانات البرومتريك للتمريض‬

‫‪https://www.facebook.com/groups/1639727976293954‬‬ ‫‪Page 1‬‬


Prometric exam collection 2017

)‫)النسخة الرابعة‬

4‫ودالشمال‬

1. The nurse is planning care for several children who were admitted
during the shift. Daily weights should be the plan of care for the
child who is receiving:

a. Total parenteral nutrition(TPN)


b. Supplement oxygen
c. Intravenous anti-ineffective
d. Chest physiotherapy

• Answer : A

hospitalization
2. The nurse is caringand “wants
for a 4-year-old
Text towithplay.”
patient a diagnosisWhat
of
cystic fibrosis and pneumonia. The child is feeling better on the 3
oice of entertainment? Text
day of the hospitalization and “wants to play.” What would be the
w
rd

best choice of entertainment?

a. Blowing bubbles
owing bubbles b. Looking at picture books
c. Watching videos
ooking at picture books
d. Riding in a wagon

• Answer : A

‫مناقشة امتحانات البرومتريك للتمريض‬

https://www.facebook.com/groups/1639727976293954 Page 2
Prometric exam collection 2017

)‫)النسخة الرابعة‬

4‫ودالشمال‬

3. A nurse is caring for an 8-year-old male with cystic fibrosis. Based


on the nurse’s understanding of the disease. What nursing
intervention should the nurse expect to perform?

a. Restrict sodium and fluid intake


b. Give antidiarrheal medications
c. Discourage coughing after postural drainage
d. Administer pancreatic enzymes with each meal

• Answer: D

4. A nurse is caring for a child with a diagnosis of cystic fibrosis and


pneumonia. The plan of care includes nebulizer treatment and
chest physiotherapy. The nurse should perform chest
physiotherapy:

a. Continuously during the nebulizer treatment


b. Prior to the nebulizer treatment
c. After the nebulizer treatment
d. Intermittently during the nebulizer treatment

• Answer: C

‫مناقشة امتحانات البرومتريك للتمريض‬

https://www.facebook.com/groups/1639727976293954 Page 3
Prometric exam collection 2017

)‫)النسخة الرابعة‬

4‫ودالشمال‬

5. While caring for a patient with an ileostomy, the nurse would


expect the ostomy to be located In Which Quadrant of the
abdomen?

a. Right lower
b. Left lower
c. Left upper
d. Right upper

• Answer : A

6. A patient has been assessed and found to have severe dysphagia


and will need long-term nutritional support, which one of the
following types of feeding would MOST likely to be beneficial for
this patient?

a. Gastrostomy
b. Patenteral
c. Nasogastric
d. Nasoduodenal

• Answer : C

7. A nurse is caring for a patient receiving total parenteral nutrition


(TPN). The patient reports the sudden onset of feeling short of
breath and anxious. The nurse hears crackles in bilateral lower
‫مناقشة امتحانات البرومتريك للتمريض‬

https://www.facebook.com/groups/1639727976293954 Page 4
Prometric exam collection 2017

)‫)النسخة الرابعة‬

4‫ودالشمال‬

lobes of the lungs and the patient’s O2 saturation is 90%on room


air. The nurse must IMMEDIATELY:

a. Turn off the TPN


b. Notify the physician
c. Asses the patient’s capillary blood glucose level
d. Attempt to suction the patient’s airway

• Answer : A

8. A nurse has just started total parenteral nutrition (TPN) as


prescribed for a patient with severe dysphagia low prealbumin
levels. In one to two hours, the nurse should anticipate assessing
the patient’s:

a. Blood glucose level


b. Weight
c. Liver
d. Spo 2

• Answer : D

• Answer : B

9. The nurse is evaluating the patient with end stage chronic


obstructive pulmonary disease (COPD).The patient has not
‫مناقشة امتحانات البرومتريك للتمريض‬

https://www.facebook.com/groups/1639727976293954 Page 5
Prometric exam collection 2017

)‫)النسخة الرابعة‬

4‫ودالشمال‬

achieved any of the goals in the plan of care. The spouse reports
concerns about the patient’s mood and increased dependency
.What action should the nurse take FIRST?
a. Continue the care plan for 1more month
b. Refer the patient to psychiatric services
c. Collaborate with the patient and spouse to revise the care plan
d. Revise the care plan based on the spouse’s input

• Answer : C
10. A home care patient with chronic obstructive pulmonary disease
(COPD)reports an upset stomach. The patient is taking
theophylline(Theo-Dur) and triamcinoloneacetonide (Azmacort)
The nurse should instruct the patient to take:

a. Theo-dur an empty stomach


b. Theo-dur and azmacortat the same time
c. Theo-dur and azmacort12 hours apart
d. Theo-dur milk or crackers

• Answer : B

11. When giving post-operative discharge instructs a patient who had


abdominal surgery, all of the following regarding wound healing
are true EXCEPT:

a. Bathing to soak abdomen is preferred


‫مناقشة امتحانات البرومتريك للتمريض‬

https://www.facebook.com/groups/1639727976293954 Page 6
Prometric exam collection 2017

)‫)النسخة الرابعة‬

4‫ودالشمال‬

b. Avoid tight belts and cloths with seams that may rub the wound
c. Pain medication may affect ability to drive.
d. Irregular bowel habits can be expected

• Answer: A

12. A nurse giving post-operative discharge instructs a patient who had


abdominal surgery, when teaching the patient about wound healing
all of the following are the true EXCEPT:

a. Wound may feel tightly or itchy as healing occurs


b. Scabs promote infection of the new skin underneath them
c. Numbness or a slight pulling sensation is normal
d. Wound should not have any drainage

• Answer : C

13. A 12-year-old child who has been diagnosed with insulin


dependent mellitus (IDDM) since age3.Comes to the clinic for a
routine visit. The patient has begun to self-manage care with
parental supervision. The patient injects 28 units of NPH insulin
every morning and 8units at bedtime. The patient checks blood
sugar 4 times every day. The patient’s weight is stable and diet is
unchanged. However, the patient reports several hypoglycemic

‫مناقشة امتحانات البرومتريك للتمريض‬

https://www.facebook.com/groups/1639727976293954 Page 7
Prometric exam collection 2017

)‫)النسخة الرابعة‬

4‫ودالشمال‬

reactions every week. The nurse knows the MOST likely cause is
that:

a. The patient is not eating the adequate number of calories reported


b. The dosages of insulin may need to be decreased as the patient
continues to grow
c. There may be changes in exercise or stress levels or the beginning
of a growth Spurt
d. The patient may not be competent in techniques of drawing up and
injecting insulin

• Answer : C

14. A nurse visits a patient at home who does not understand how to
take a newly prescribed medication. The prescription reads: 5 ml
PO TID p.c. meals. The nurse explains to the patient that the
correct way to take the medication is:
a. 1 teaspoon by mouth, 3times a day, before meals
b. 1 teaspoon by mouth, 3times a day, after meals
c. 1 tablespoon by mouth, 3times a day, before meals
d. 1 tablespoon by mouth, 3times a day, after meals

• Answer : B

‫مناقشة امتحانات البرومتريك للتمريض‬

https://www.facebook.com/groups/1639727976293954 Page 8
Prometric exam collection 2017

)‫)النسخة الرابعة‬

4‫ودالشمال‬

15. The nurse is caring for a patient who had major abdominal surgery
under general anesthetic 4 hours ago. An appropriate goal for the
patient includes:

a. Having minimal fine crackles in the base of the lungs


b. Using the incentive spirometry every 4 hours
c. Expectorating minimal amount of secretions
d. Performing Coughing Exercises every hour while awake

• Answer : D

16. While caring for a child with aventriculoperitoneal shunt revision,


the nurse find the patient lying with the head and feet flexed back.
The nurse should call for help and prepare for a(n):

a. Spinal tap
b. Shunt culture
c. Electrocardiogram
d. Ventricular tap

• Answer : D

‫مناقشة امتحانات البرومتريك للتمريض‬

https://www.facebook.com/groups/1639727976293954 Page 9
Prometric exam collection 2017

)‫)النسخة الرابعة‬

4‫ودالشمال‬

17. During surgery requiring general anesthesia, the patient heart’s


stops and a carotid pulse is not palpated. How many compressions
per minute should be administered?

a. 50
b. 60
c. 80
d. 100

• Answer :D

18. When teaching a community class on cerebrovascular accidents


(stroke), which of the following should participants of the class
know at the completion of the class?

a. Muscle and ligament damage is not reversible


b. Expressive aphasia is resolved by voice rest
c. There is a risk for mood disorders such as depression
d. Liquids should be consumed at the same times as solids food

• Answer : D

19. A community health care nurse visits a patient who had


cerebrovascular accident. The patient is at risk for deficient volume
due to voluntary reduction intake fluid intake to avoid the use of
‫مناقشة امتحانات البرومتريك للتمريض‬

https://www.facebook.com/groups/1639727976293954 Page 10
Prometric exam collection 2017

)‫)النسخة الرابعة‬

4‫ودالشمال‬

the bathroom. The nurse educates the patient on the importance of


drinking fluids and maintaining hydration. Which of the following
indicates the efficacy of the nursing intervention?

a. Amber color urine


b. Respiration of 35
c. Tachycardia
d. Moist mucous membrane

• Answer : A

20. A home health nurse is visiting a patient following a


cerebrovascular accident (CVA). The patient is having trouble
sleeping and is feeling sad. The patient’s spouse tells the nurse that
the patient is not eating much and often cries when nooneis
watching. Which of the following would be the nurse’s MOST
likely intervention?

a. Assess for changes in cognitive abilities


b. Complete a depression index
c. Strengthen family coping methods
d. Screen for pain

• Answer : B
‫مناقشة امتحانات البرومتريك للتمريض‬

https://www.facebook.com/groups/1639727976293954 Page 11
Prometric exam collection 2017

)‫)النسخة الرابعة‬

4‫ودالشمال‬

21. A home health nurse is visiting a patient who recently suffered a


Cerebrovascular accident (CVA). The nurse would MOST likely
implement which of the following interventions to prevent muscle
and ligament deformities?

a. Daily moist heat and isometric exercises


b. Daily balance training and routine medications for pain
c. Instruct patient to use non-affected side to perform activities
of daily living
d. Daily range of motion exercises.

• Answer : C

22. A nurse is assigned to do a home visit for an 81-year-old patient.


The patient lives at home with an adult caretaker and is completely
bed-bound following a Cerebrovascular accident (CVA) 2 weeks
ago. In planning caregiver education, The nurse should be prepared
to instruct the caretaker in:

a. How to select a nursing home for the patient


b. Performing passive range of motion exercises
c. The importance of avoiding viscous drinks
d. Forming a local chapter of a care giver support group

‫مناقشة امتحانات البرومتريك للتمريض‬

https://www.facebook.com/groups/1639727976293954 Page 12
Prometric exam collection 2017

)‫)النسخة الرابعة‬

4‫ودالشمال‬

• Answer : D
23. A home care nurse makes a follow-up visit to a patient who
recently suffered a cerebrovascular accident. The patient is mobile
and able to perform activities of daily living. However, the patient
has not sleeping and has lost weight due to lack of appetite. The
patient also feels overwhelmed with sadness. Which of the
following is the most appropriate evaluation?

a. Patient’s progress is as expected and no further intervention


is necessary
b. Patient needs referral to a nutritionist
c. Patient needs intervention for depression
d. Patient needs sleeping medication

• Answer : C

24. A patient admitted with a cerebrovascular accident (CVA), is


unable to chew or swallowed. The patient is a risk for aspiration.
The nurse would anticipate receiving which of the following orders
for this patient?

a. Give no food by mouth and start intravenous hydration


b. Start a pureed diet with thickened liquids
c. Refer the patient to a psychiatrist for depression related to the
CVA
d. Refer the patient to physical therapy for muscle strengthening
‫مناقشة امتحانات البرومتريك للتمريض‬

https://www.facebook.com/groups/1639727976293954 Page 13
Prometric exam collection 2017

)‫)النسخة الرابعة‬

4‫ودالشمال‬

• Answer : A

25. While the nurse is administering a large volume enema, the patient
complains of cramping. The nurse should:

a. Increase the flow rate


b. Lower the fluid container
c. Elevate the head of the bed
d. Gently massage the abdomen

• Answer : B
26. A home health nurse has entered a home to complete an admission
assessment on a patient who has a methicillin-resistant
Staphylococcus aureus (MRSA) urinary tract infection. The patient
will receive intravenous anti-infective via a peripherally inserted
central catheter (PICC) for 3 weeks. Which of the following
actions should the nurse take FIRST?

a. Shake the patient’s hand


b. Place the nursing supply bagon a clean, dry surface
c. Obtain the patient’s written consent for home health care
d. Perform hand hygiene per the agency protocol

• Answer : D

‫مناقشة امتحانات البرومتريك للتمريض‬

https://www.facebook.com/groups/1639727976293954 Page 14
Prometric exam collection 2017

)‫)النسخة الرابعة‬

4‫ودالشمال‬

27. A home health nurse is teaching a family member about the care of
patient’s peripherally inserted central catheter (PICC). Which of
the following statements would be appropriate for the nurse to
make?

a. Place the used intravenous tubing in a leak proof container


and then place this sealed container inside a second leak proof
container.”
b. “You will need to put on a disposable face mask before you
connect the intravenous tubing to the port of thePICC. The port of
the PIC catheter will need to be cleansed with povidone-iodine
(BETADINE) after the infusion is completed.”
c. “The empty medication container can be placed in the same
container as your Household refuses.”

• Answer : A

28. A patient had a craniotomy with resection of a nonmalignant


neoplasm for the temporal lobe. The patient’s vital signs are within
the base line normal range. The nurse observes that the patient has
developed bilateral per orbital edema. Which of the following
actions would be appropriate for the nurse to take?

a. Apply cold compresses to the patient’s eyes


b. Apply warm compresses to the patient’s eyes
‫مناقشة امتحانات البرومتريك للتمريض‬

https://www.facebook.com/groups/1639727976293954 Page 15
Prometric exam collection 2017

)‫)النسخة الرابعة‬

4‫ودالشمال‬

c. Elevate the head of the patient’s bed to 60 degrees


d. Elevate the head of the patient’s bed to 45 degrees

• Answer :D

29. To decrease the incidence of aspiration of gastric contents in a


child hospitalization with severe burns, the nurse should position
the head:

a. Flat except during meals


b. Elevates 30-45 degrees during meals
c. Elevated 15-30 degrees for12-hours after meals
d. Elevated 45 degrees at all times

• Answer : B

30. A home health nurse visits a patient with diabetes and primary
open-angle glaucoma. The patient takes metformin (Glucophage)
500 mg once a day for diabetes and timolol ophthalmic solution
twice a day in each eye for glaucoma. Which of the following
evaluations indicates that the patient is noncompliant with
glaucoma management?

a. Patient has not been taking Glucophage


b. Patient has tearing of the eye
‫مناقشة امتحانات البرومتريك للتمريض‬

https://www.facebook.com/groups/1639727976293954 Page 16
Prometric exam collection 2017

)‫)النسخة الرابعة‬

4‫ودالشمال‬

c. Patient has not refilled prescription for timolol in 3 months


d. Patient has yellow discharge from the eyes

• Answer : C

31. A patient is having difficulty with cognitive abilities after a stroke.


What part of the brain was MOST likely affected?

a. Midbrain
b. Cerebrum
c. Medulla oblongata
d. Cerebellum

• Answer : B

32. A 16-years old patient present to the clinic requesting birth control.
With the diagnosis of health seeking behaviors, the BEST goals
have the patient:

a. Verbalizing understanding of safe sex practices and


following safe sexual practices in all encounters
b. Not engaging in sexual encounters until she is over18 years
old and maintaining a healthy life style
c. Recognizing the sign of pregnancy and the symptoms of
sexually transmitted diseases

‫مناقشة امتحانات البرومتريك للتمريض‬

https://www.facebook.com/groups/1639727976293954 Page 17
Prometric exam collection 2017

)‫)النسخة الرابعة‬

4‫ودالشمال‬

d. Understanding safe sexual practices and use a condom to


prevent pregnancy and sexually transmitted diseases

• Answer : D

33. A nurse plans to teach a group of 20to25-year-old women about


oral contraceptives. The nurse should instruct that oral
contraceptives may:

a. Increase the risk of pelvic inflammatory disease


b. Cause acne to worsen
c. Decrease the risk of breast and cervical cancer
d. Decrease the risk of endometriosis

• Answer : A

34. Following lumbar surgery a patient has a 4 millimeter (mm)


surgical incision. The incision is clean and the edges are well
appropriate. This type of tissue healing is classified as which of the
following?
a. Primary intention
b. Secondary intention
c. Tertiary intention
d. Superficial epidermal

‫مناقشة امتحانات البرومتريك للتمريض‬

https://www.facebook.com/groups/1639727976293954 Page 18
Prometric exam collection 2017

)‫)النسخة الرابعة‬

4‫ودالشمال‬

• Answer : A

35. Shrinkage device is applied after surgery for amputation of the leg.
The goal of the shrinkage device is to from the residual limb into
what shape?

a. Cone
b. Oval
c. Mushroom
d. Cylinder with blunt end

• Answer : D

36. A surgeon instructs a nurse to serve as a witness to an elderly


patient’s informed consent for surgery. During the explanations to
the patient, it becomes clear that the patient is confused and does
not understand the procedure, but reluctantly sign the consent
form. The nurse should:

a. Sign the form as a witness, making a nation that the patient did not
appear to understand
b. Not sign the form as a witness and notify the nurse supervisor
c. Not sign the form and answer the patient’s questions after the
surgeon leaves he room
d. Sign the form and tell surgeon that the patient doesn’t understand
the procedure
‫مناقشة امتحانات البرومتريك للتمريض‬

https://www.facebook.com/groups/1639727976293954 Page 19
Prometric exam collection 2017

)‫)النسخة الرابعة‬

4‫ودالشمال‬

37. The nurse administered a dose of morphine sulfate as prescribed to


a patient who is in the post anesthesia care unit (PACU). The
patient appears to be resting comfortably, the respiratory rate is 8
and the O2saturation is 21 oxygen via cannula is 86%. The nurse
should IMMEDIATELY administer:

a. Flumazenil (Romazicon)
b. Medazolum (versed)
c. Naloxone (Narcan)
d. Ondansetron (Zofran)

• Answer : C

38. A patient schedule for a major surgery in one hour is very nervous
and upset. Which of the following order medications would the
nurse administer to relax this patient?
a. Meperidine Hydrochloride(Demerol)
b. Scopolamine (Transderm-Scop)
c. Pentobarbital sodium(Nembutal sodium)
d. Trazodone hydrochloride(Trazadone)

• Answer : A

‫مناقشة امتحانات البرومتريك للتمريض‬

https://www.facebook.com/groups/1639727976293954 Page 20
Prometric exam collection 2017

)‫)النسخة الرابعة‬

4‫ودالشمال‬

39. A patient with poor wound healing and poor appetite has an order
to begin total parental nutrition (TPN). Waiting for the TPN
solution to arrive from the pharmacy, the nurse should obtain:

a. A pair of sterile gloves


b. An infusion pump
c. IV tubing with a micro-drip chamber
d. Povidine-iodine (Beta dine)swabs

• Answer : B
40. When conducting discharge teaching for the parent of a child
newly diagnosed with cystic fibrosis. Which of the following
statement by the parent indicates the need for further teaching?

a. Weekly weights help evaluate effectiveness of nutritional


interventions
b. Weekly weights help the doctor know if may child is absorbing
nutrients
c. Weekly weights reassure my child that recovery is progressing
d. Weekly weights help the doctor know if my child needs additional
enzymes

• Answer : D

‫مناقشة امتحانات البرومتريك للتمريض‬

https://www.facebook.com/groups/1639727976293954 Page 21
Prometric exam collection 2017

)‫)النسخة الرابعة‬

4‫ودالشمال‬

41. A patient with a pulmonary embolus and a nursing diagnosis of


impaired gas exchange has an order to obtain arterial blood gases.
The FIRST intervention by the nurse is to:

a. Perform an Allens test


b. Explain the procedure
c. Gather the equipment
d. Document the procedure

• Answer : A

42. A patient is diagnosed with pulmonary hypertension. Which of the


following nursing diagnoses should be the PRIORITY?

a. Impaired gas exchanged related to altered blood flow


secondary to pulmonary capillary constriction
b. Fatigue related to hypoxia
c. Anxiety related to illness and loss of control
d. Activity intolerance related to imbalance between oxygen supply
and demand due to right and left ventricular failure

• Answer : D
43. A patient who had abdominal surgery is in the post anesthesia care
unit (PACU).Which of the following nursing diagnosis takes
PRIORITY?

‫مناقشة امتحانات البرومتريك للتمريض‬

https://www.facebook.com/groups/1639727976293954 Page 22
Prometric exam collection 2017

)‫)النسخة الرابعة‬

4‫ودالشمال‬

a. Disturbed sleep pattern


b. Acute pain
c. Risk for infection
d. Ineffective airway clearance

• Answer : D

44. While caring for a patient in the post-anesthesia care unit (PACU),
a nurse observes the onset of rapid breathing cyanosis, and
narrowing blood pressure. The nurse should plan to:

a. Administer bolus glucose


b. Suction the airway
c. Turn the patient to the right side
d. Administer intra venous fluids

• Answer : B

45. While caring for a patient in the post-anesthesia care unit (PACU)
Who has developed Hypovolemic shock, a nurse should position
the patient:

a. Flat with legs elevated


b. In Trendelenburg position
c. With the head of the bed elevated 45 degrees
‫مناقشة امتحانات البرومتريك للتمريض‬

https://www.facebook.com/groups/1639727976293954 Page 23
Prometric exam collection 2017

)‫)النسخة الرابعة‬

4‫ودالشمال‬

d. Completely flat

• Answer : B

46. A patient had a vitrectomy and is about to be transported to the


post anesthesia care unit (PACU). The patient should be placed in
which of the following positions before transport to the PACU?

a. Semi-fowler’s
b. Prone
c. Dorsal recumbent
d. Sim’s

• Answer : B
47. While caring for a patient in the post-anesthesia care unit (PACU),
a nurse plans to Keep the patient warm. What is the MUST
important reason for this action?

a. To preserve nutritional stores


b. To prevent cutaneous vessel dilation
c. To decrease patient anxiety
d. To lower risk of infection resulting from chill

• Answer : C

‫مناقشة امتحانات البرومتريك للتمريض‬

https://www.facebook.com/groups/1639727976293954 Page 24
Prometric exam collection 2017

)‫)النسخة الرابعة‬

4‫ودالشمال‬

48. A patient had a total abdominal hysterectomy 2days-ago and has


not been out of the bed yet. The patient is complaining left leg pain
and swelling. What should the nurse do FIRST?

a. Gently massage the patient’s leg


b. Assess the patient’s pain level
c. Assess the patient for Homan’s sign
d. Instruct the patient to reflex the left knee and hip

• Answer : C

49. To minimize a toddler from scratching and picking at a healing


skin graft site, the nurse should utilize?

a. Hand mittens
b. Mild sedatives
c. Punishment for picking
d. Distraction

• Answer : D

50. The nurse is teaching the mother of a 3-months-old infant about


bottle feeding. Which statement indicates the mother understands
of appropriate procedure?

‫مناقشة امتحانات البرومتريك للتمريض‬

https://www.facebook.com/groups/1639727976293954 Page 25
Prometric exam collection 2017

)‫)النسخة الرابعة‬

4‫ودالشمال‬

a. “I should hold my baby in as lightly reclined position, close


to my body”
b. “It is OK to prop the bottle on a pillow”.
c. “It can feed my baby whole milk”
d. “I should warm the bottles in the microwave if they come out of
the Refrigerator”.

• Answer : A

51. A 9-month-old child who has had four ear infections in the past 6
months is being discharged. Which statement by the parent
indicates the need for further discharge teaching?

a. I should never put my baby to bed with bottle


b. My child should not use a pacifier after age 6 months
c. My child should drink his bottle while lying flat in my lap
d. My child should not be around people who smoke

• Answer : B

52. A Patient complains of severe menstrual cramping. Bleeding is not


un usually heavy and the patient has no uterine disorders. Which of
the following interventions should the nurse anticipate the doctor
will order promote comfort?

a. Acetaminophen (Tylenol)
‫مناقشة امتحانات البرومتريك للتمريض‬

https://www.facebook.com/groups/1639727976293954 Page 26
Prometric exam collection 2017

)‫)النسخة الرابعة‬

4‫ودالشمال‬

b. Strict bed rest


c. Heating pad to the back of neck
d. Ibuprofen (Motrin)

• Answer : D

53. During Pre-operative preparation of a patient for amputation of the


left leg. The nurse has primary responsibility for:

a. Witnessing the patient signature on the consent form


b. Explaining the procedure to the patient
c. Explaining the risks of the surgery to the patient
d. Making appropriate incision lines on the leg.

• Answer : A

54. A 52-years-old is admitted to the nursing unit from the physician’s


office with a diagnosis of acute cholecystitis. Physician orders on
admission include: monitor vital sign every 4 hours; IV of ringer’s
lactate 125ml per hour; 1500 calorie, low-fat liquid diet, morphine
sulfate 2mg IV every 2 hours as needed for pain, notify physician
for sudden increase in frequency or intensity of pain,
promethazine12.5 mg IV every 4 hours as needed for nausea or
vomiting. Which of the following should the nurse plan to do
FIRST?
‫مناقشة امتحانات البرومتريك للتمريض‬

https://www.facebook.com/groups/1639727976293954 Page 27
Prometric exam collection 2017

)‫)النسخة الرابعة‬

4‫ودالشمال‬

a. Remove any high-foods from the patient’s room


b. Notify the dietitian of the diet order
c. Obtain venous access and start Ringer’s lactate infusion
d. Obtain an emesis basin and clean linens for the be side

• Answer : C

55. A parent brings a 10-month-old infant into the department saying,


“my baby put a button in her mouth and now she is not breathing!”
After the nurse determines the infant is not breathing. What should
the nurse do NEXT?

a. Perform the Heimlich maneuver


b. Initiate cardio pulmonary resuscitation (CPR)
c. Administer 4 back blows
d. Administer 4 thrusts midline on the patient back

• Answer : D

56. An infant arrives in the emergency department not breathing and


does have a pulse. When starting cardio pulmonary resuscitation
(CPR), where is the correct place to assess for a pulse in this
patient?

a. Carotid
‫مناقشة امتحانات البرومتريك للتمريض‬

https://www.facebook.com/groups/1639727976293954 Page 28
Prometric exam collection 2017

)‫)النسخة الرابعة‬

4‫ودالشمال‬

b. Radial
c. Brachial
d. Temporal

• Answer : C

57. A 5 years old patient who underwent abdominal surgery suffers


from deficient fluid volume related to nothing by mouth (NPO)
status; intravenous fluid therapy is given for hydration. Which of
the following indicates that the treatment is effective?

a. Urinary output of 15ml/hr.


b. Respiration rate, 35
c. Heart rate 100
d. Good skin turgor

• Answer : D

58. A 7-years-old child is brought to the physician office due to


sudden onset of bright redness on the cheeks. The nurse observes
that the child has a temperature of 380 C (100.40 F) With chills the
nurse suspects that the MOST like diagnosis would be:

a. Fifth disease
b. Rotavirus
c. Roseolainfantum
‫مناقشة امتحانات البرومتريك للتمريض‬

https://www.facebook.com/groups/1639727976293954 Page 29
Prometric exam collection 2017

)‫)النسخة الرابعة‬

4‫ودالشمال‬

• Answer : A

59. A nurse instructs a community education class on breast health.


Which statement BEST described understanding of the appropriate
age to start screening mammograms is a woman of average risk?

a. At menopause
b. At 65-years-old
c. At the cessation of breastfeeding
d. At 40-years-old

• Answer : A

60. A patient is taught how to perform a breast self-exam by a nurse.


Which statement is BEST described as understanding of the proper
procedure for doing a breast self-exam?

a. Use of the palm of the hand to feel for lumps


b. Apply three different levels of pressure to feel breast tissue
c. Stand when performing breast self-exam
d. Perform self-exam annually

• Answer : A

‫مناقشة امتحانات البرومتريك للتمريض‬

https://www.facebook.com/groups/1639727976293954 Page 30
Prometric exam collection 2017

)‫)النسخة الرابعة‬

4‫ودالشمال‬

61. While caring for a child with in effective airway clearance related
to increased mucus production, the nurse should encourage fluids
to:

a. Maintain nutrition
b. Prevent boredom
c. Stimulate coughing
d. Thin secretions

• Answer : D

62. A 59-years old patient with lung cancer and metastases to the bone
is in the hospital for pain management. The patient rates the pain
10 on a scale of 0(no pain) to 10 (severe pain). The BEST goal for
the nurse diagnosis of alteration is comfort is that the patient will:

a. Show no objective signs of pain


b. Not complain of pain
c. State pain is at a tolerable level
d. State that all pain is relieved

• Answer : D

63. A patient with advanced lung cancer is exhibiting cyanosis and


edema of the head and upper extremities. Which of the following

‫مناقشة امتحانات البرومتريك للتمريض‬

https://www.facebook.com/groups/1639727976293954 Page 31
Prometric exam collection 2017

)‫)النسخة الرابعة‬

4‫ودالشمال‬

intervention would MOST likely provide an immediate benefit for


this patient?
a. Place in Trendelenburg position
b. Position on the right side
c. Elevate the head of the bed
d. Elevate extremities

• Answer : C

64. If a patient develops a complication during a blood transfusion, the


nurse first should be to:

a. Stop the transfusion


b. Notify the practitioner
c. Administer an antihistamine
d. Administer an anti- inflammatory medication

• Answer : A

65. Which of the following types of health care services is an example


of the primary level of care?

a. Diagnosis
b. Acute care
c. Restoration
‫مناقشة امتحانات البرومتريك للتمريض‬

https://www.facebook.com/groups/1639727976293954 Page 32
Prometric exam collection 2017

)‫)النسخة الرابعة‬

4‫ودالشمال‬

d. Immunization

• Answer : D

66. In planning for the care of a patient with Crohn’s disease, the nurse
and patient discuss the interventions. Which of the following
treatment modalities would MOST likely be considered a primary
intervention for this disease?

a. Surgery
b. Medications
c. High-residue diet
d. Blood replacement

• Answer : B

67. A patient with acute crohn’s disease has been prescribed an


elemental diet. The MOST likely rationale for this is to:

a. Reset the bowel


b. Improve nutrition
c. Improve medication absorption
d. Prepare for surgery

• Answer : C

‫مناقشة امتحانات البرومتريك للتمريض‬

https://www.facebook.com/groups/1639727976293954 Page 33
Prometric exam collection 2017

)‫)النسخة الرابعة‬

4‫ودالشمال‬

68. A patient has a6-year history of inflammatory bowel disease that is


resistant to medical therapy. The patient can BEST decreased the
like hood of the disease progressing to

a. Consuming only elemental foods


b. Stopping smoking
c. Using effective birth control
d. Avoiding over heating

• Answer : A

69. A home health nurse is setting up a medication administration


schedule for an elderly patient. The patient is talking Oscal
(calcium corbonate), Feosol (ferrous sulfate), and Orazinc (Zinc
sulfate). The patient eats meals at 8:00 AM, 12 noon, and6:00 PM.
Which of the following medication administration times would the
nurse MOST likely implement for this patient?

a. Oscal, Orazinc, and foesal at 8:00AM


b. Oscal at 6:00AM, Orzinc at 12:00 noon, Foesal At 4:00PM
c. Oscal and Foesal at 12:00 noon and Orazinc at 6:00PM
d. Orazinc at 6:00 AM, Oscal at 12:00 noon, and Foesal at
6:00PM

• Answer : C

‫مناقشة امتحانات البرومتريك للتمريض‬

https://www.facebook.com/groups/1639727976293954 Page 34
Prometric exam collection 2017

)‫)النسخة الرابعة‬

4‫ودالشمال‬

70. A Community Health nurse is administering tuberculin skin tests


purified protein derivative (PPD), which of the following time
frames should the nurse tell the patient to return to the clinic for
the test to be read?
a. In 12-24 hours
b. In 24-36 hours
c. In 36-48 hours
d. In 48-72 hours

• Answer : D

71. A patient who is scheduled for a tonsillectomy is in pre-operative


unit. The nurse notes an order for pre anesthetic medication to be
given “on call to operation room”. The nurse should give this
medication:

a. Immediately upon being notified to prepare the patient for


transport
b. When the operation room staff arrive to transport the patient
c. Only if clearly needed after assessment
d. Upon the patient’s arrival in the operation room

• Answer : A
‫مناقشة امتحانات البرومتريك للتمريض‬

https://www.facebook.com/groups/1639727976293954 Page 35
Prometric exam collection 2017

)‫)النسخة الرابعة‬

4‫ودالشمال‬

72. A patient recently underwent coronary artery bypass graft surgery


(CABG). The Nursing diagnosis includes sleep deprivation related
to intensive care environment. The goal for this diagnosis would be
that the patient:

a. Gets 4 hours of uninterrupted sleep during the right


b. Takes naps during the day
c. Is free of pain in the first hour post-surgery
d. Ambulates 3 hours post-surgery

• Answer : B

73. The nurse is assisting a patient to ambulate in the hall. The patient
a history of coronary artery disease(CAD), and had coronary artery
bypass graft surgery(CABG) 3 days ago, the patient reports chest
pain rated 3 on a scale of 0 (no pain)to 10 (severe pain) the nurse
should FIRST:

a. Determine how long it has since the patient’s last dose of


aspirin
b. Obtain a chair for the patient so sit down
c. Assess the patient’s radial pulse
d. Ask the patient to take several slow, deep breaths
‫مناقشة امتحانات البرومتريك للتمريض‬

https://www.facebook.com/groups/1639727976293954 Page 36
Prometric exam collection 2017

)‫)النسخة الرابعة‬

4‫ودالشمال‬

• Answer : A

74. A 35-years-old female has an inherited gene mutation for


achondroplasia, an autosomal dominate genetic disorder. Her
husband does not have genemutation. In planning genetic
counseling for this patient, the nurse would be MOST correct in
including which of the following statements regarding the risk of
their children inherited the genetic mutation?

a. Each child has a 50% chance of inheriting the gene mutation


b. Female children have 50% chance of inheriting the gene
mutation
c. Male children will not inherited the gene mutation
d. All female children will inherit the gene mutation.

• Answer : A

75. A patient is one day post-operative repair of a large umbilical


hernia. The patient complains of abdominal pain and described
feeling the sutures give way. Upon assessment of the abdomen the
nurse observes an evisceration. The nurse’s IMMEDIATE
response should be to:

‫مناقشة امتحانات البرومتريك للتمريض‬

https://www.facebook.com/groups/1639727976293954 Page 37
Prometric exam collection 2017

)‫)النسخة الرابعة‬

4‫ودالشمال‬

a. Medicate the patient for pain


b. Instruct the patient to cough hard
c. Have the patient perform the valsalvas maneuver
d. Cover the abdomen with a sterile soaked dressing

• Answer : D

76. A 3-years old child is seen at the pediatrician’s office. The parents
the child has had vomiting and diarrhea for the past 15 hours. The
child’s is lethargic with the following vital signs: temperature 37.20
C (99.0 F), heart rate 145,respiration rate 25, and blood pressure
level 95/55 mmHg. Which of the vital sign is abnormal?

a. 37.20 C (99.00 F)
b. Heart rate 145
c. Respiration rate 25
d. Blood pressure level 95/55

• Answer : B

77. A home health nurse is teaching a family member about the care of
a patient’s peripherally inserted central catheter (PICC). Which of
the following would be appropriate for the nurse to make?
a. “Place the used intravenous tubing in a leak proof container
and then this in sealed container inside a second leak proof
container”.
‫مناقشة امتحانات البرومتريك للتمريض‬

https://www.facebook.com/groups/1639727976293954 Page 38
Prometric exam collection 2017

)‫)النسخة الرابعة‬

4‫ودالشمال‬

b. “You will need to put on a disposable face mask before you


connect the port of the PICC.”
c. “The port of the PICC catheter will need to be cleansed with
providence-iodine Betadine) after the insulin is completed.”
d. “The empty medication container can be placed in the same
container as your house hold refuses.”

• Answer : A

78. While Obtaining the pre-operative history of a patient schedules


for cosmetic surgery, the most valuable skill at the nurse disposal
is:

a. knowledge of the procedure


b. Time management skills
c. Listening skills
d. Empathy

• Answer : D

79. A community health nurse screens a group of high risk adults for
tuberculosis. Which gauage needle should the nurse use for an
intradermal injection on the ventral surface of the forearm?

a. 16 gauge needle
b. 20 gauge needle
‫مناقشة امتحانات البرومتريك للتمريض‬

https://www.facebook.com/groups/1639727976293954 Page 39
Prometric exam collection 2017

)‫)النسخة الرابعة‬

4‫ودالشمال‬

c. 22 gauge needle
d. 26 gauge needle

• Answer : D

80. A patient hospitalized with tuberculosis (TB) has a productive


cough and hemoptysis. Which of the following types of isolation
room would be the best choice for the patient?

a. Reverse isolation
b. Standard isolation
c. Positive-pressure
d. Negative-pressure

• Answer : D

81. A patient diagnosed with tuberculosis is prescribed isoniazid


(Isoniazid), Rifampin (Rifadin), pyrazinamide (Rifamate),
ethambutol HCL (Myambuton), one month later the patient comes
to the physician office with hepatitis. Which drug is the MOST
likely cause?

a. Ethambutol(Myambuton),
b. Acetaminophen,
c. Izoniazid (Izoniazid),
‫مناقشة امتحانات البرومتريك للتمريض‬

https://www.facebook.com/groups/1639727976293954 Page 40
Prometric exam collection 2017

)‫)النسخة الرابعة‬

4‫ودالشمال‬

d. Pyrazinamide (Rifamate).

• Answer : C

82. A Patient with tuberculosis can transmit the disease to another


individual Through:

a. Air droplets
b. Physical contact
c. Hand to mouth exchange
d. Blood and body fluids

• Answer : A

83. A patient recently underwent joint replacement surgery, which of


the following nursing diagnosis takes PRIORITY?

a. Risk for peripheral neurovascular dysfunction


b. Deficient knowledge on appropriate activity precaution
c. Impaired physical mobility
d. Sexual dysfunction related to pain

• Answer : C

‫مناقشة امتحانات البرومتريك للتمريض‬

https://www.facebook.com/groups/1639727976293954 Page 41
Prometric exam collection 2017

)‫)النسخة الرابعة‬

4‫ودالشمال‬

84. The parents are anxious after the doctor tells that their child needs
surgery. The assess parents’ ability to cope with this anxiety,
which of the following questions should the nurse ask

a. “Did you know that feeling anxious about your child’s


surgery is normal?”
b. “Can you wait until after surgery to begin to cope with being
anxious?”
c. “How do you think feeling of anxiety will affect your child?”
d. “What has helped you when you felt anxious in the past?”

• Answer : A

85. A 4-year-old child brought to the community health clinic for


scheduled immunizations. The child should receive:

a. Varicella, rotavirus, pneumococcal and hepatitis B


b. Measles, mumps, rubella and varicella
c. Rotavirus and inactivated polio virus
d. Varicella andhaemophilus influenza

• Answer : B

86. The nurse is conducted a community-based educational program


about Diabetes mellitus. Which of the following statements by a
participant would indicate correct understanding of the teaching?
‫مناقشة امتحانات البرومتريك للتمريض‬

https://www.facebook.com/groups/1639727976293954 Page 42
Prometric exam collection 2017

)‫)النسخة الرابعة‬

4‫ودالشمال‬

a. Lantus insulin can be mixed with other insulin


b. It is necessary to wipe off the top the insulin vial with alcohol
to prevent infection
c. Insulin will changed color after opening
d. Needles can be placed in a hard plastic container with a
tightly secure lid

• Answer : B

87. A child is treated for superficial (first-degree) thermal burns to the


thigh. The child is in great discomfort and does not eat. Which of
the following diagnosis should receive PRIORITY?

a. Altered nutrition
b. Impaired skin integrity
c. Risk for infection
d. Acute pain

• Answer : D

88. The nurse calls together an inter disciplinary team with members
from medicine, social services, the clergy, and nutritional services
to care for a patient with a terminal illness. Which of the following
types of care would the team MOST likely is providing?
a. Palliative
‫مناقشة امتحانات البرومتريك للتمريض‬

https://www.facebook.com/groups/1639727976293954 Page 43
Prometric exam collection 2017

)‫)النسخة الرابعة‬

4‫ودالشمال‬

b. Curative
c. Respite
d. Preventive

• Answer : A
89. A nurse makes a home visit to a patient recently diagnosed with
chronic obstructive pulmonary disease (COPD), which of the
following should the nurse teach the patient about managing
COPD?

a. Recognizing signs of impending respiratory infection


b. Limiting fluids intake minimize bronchial secretions
c. Correct technique to auscultate the lung fields
d. Importance of starting antibiotic therapy

• Answer : A

90. A patient with chronic obstructive pulmonary disease (COPD)


experiencing frequent dyspnea which of the following exercise
would teach the patient how to BETTER control breathing?

a. Lower side rib


b. Segmental
c. Pursed-lip
d. Diaphragmatic

‫مناقشة امتحانات البرومتريك للتمريض‬

https://www.facebook.com/groups/1639727976293954 Page 44
Prometric exam collection 2017

)‫)النسخة الرابعة‬

4‫ودالشمال‬

• Answer : C
91. In evaluating the appropriateness of various exercises enjoyed by a
patient with osteoporosis, the nurse would recommend:
a. Walking
b. Bowling
c. Sit-ups
d. Golf

• Answer : A

92. A patient presents to the clinic with “pins and needles” sensations
of the left foot and complains that objects appear “Shimmering”.
The patient is diagnosed with optic neuritis and referred for further
testing. The patient is MOST likely to be tested for:

a. Glaucoma
b. Multiple sclerosis
c. Lesion of brain stem
d. Psychosis

• Answer : B

93. A 3-years-old has returned to the clinic 4 days after being


diagnosed with gastroenteritis and dehydration. A parent reports
‫مناقشة امتحانات البرومتريك للتمريض‬

https://www.facebook.com/groups/1639727976293954 Page 45
Prometric exam collection 2017

)‫)النسخة الرابعة‬

4‫ودالشمال‬

that the vomiting has stopped, and the child is tolerating liquids,
rice, apple sauce, and bananas. The diarrhea persists, but seems to
be decreasing in volume. When evaluating for signs of
dehydration, the nurse will assess the patient’s skin turgor by:

a. Grasping the skin over the abdomen with two fingers raising
the skin with two fingers
b. Grasping the skin over the forehead with two fingers and
raising the skin with two fingers
c. Holding the patient’s mouth open and assessing the tongue
for deep creases or Furrows
d. Drawing two tubes of blood and running blood urea nitrogen
(BUN) and creatinine (Cr).

• Answer : A

94. When administering albuterol to a child with asthma, the nurse


should observe for sign of what major side effect to this
medication?

a. Tachycardia
b. Renal failure
c. Apnea Blurred vision

• Answer : A
‫مناقشة امتحانات البرومتريك للتمريض‬

https://www.facebook.com/groups/1639727976293954 Page 46
Prometric exam collection 2017

)‫)النسخة الرابعة‬

4‫ودالشمال‬

95. A child with asthma is experiencing thick respiratory secretions


resulting in increased work of breathing. The best nursing
intervention is to:

a. Encourage fluids
b. Eliminate dairy products
c. Decrease relative humidity of the room
d. Have the child lay on the left side.

• Answer : C

96. What would be the long-term goal for a child with asthma?

a. Quickly reverse airflow obstruction


b. Correct hypoxemia
c. Deliver humidified oxygen via nasal cannula
d. Develop a home and school management plan

• Answer : D

97. A nurse administers an albuterol nebulizer on a child with asthma


exacerbation. Which of following indicates effectiveness of the
treatment?

‫مناقشة امتحانات البرومتريك للتمريض‬

https://www.facebook.com/groups/1639727976293954 Page 47
Prometric exam collection 2017

)‫)النسخة الرابعة‬

4‫ودالشمال‬

a. Adventitious breath sound with cough


b. O2 saturation 94%
c. Nasal flaring
d. Respiration rate 28

• Answer : B

98. A Child is diagnosed with asthma exacerbation. Which of the


following nursing diagnoses should be the FIRST priority?
a. In effective airway clearance related to broncho spasm and
mucosal edema
b. Fatigue related to hypoxia
c. Anxiety related to illness and loss of control
d. Deficient knowledge related to potential side effect of the
medication

• Answer : A
99. An asthmatic patient presents with wheezing and coughing.
Oxygen saturation is 88% on room air. Which of the following
nursing diagnosis would take priority?

a. Imbalanced nutrition related to decreased food intake


b. Activity intolerance related to inefficient breathing
c. Anxiety-related dyspnea and concern of illness
d. Ineffective gas exchange related to broncho spasm

‫مناقشة امتحانات البرومتريك للتمريض‬

https://www.facebook.com/groups/1639727976293954 Page 48
Prometric exam collection 2017

)‫)النسخة الرابعة‬

4‫ودالشمال‬

• Answer : D

100. The nurse is visiting the asthmatic patient at home to reinforce the
importance of eliminating environmental allergens and to assess
the patient’s response to the environmental changes. This type of
implementation is called:

a. Supervision and coordination


b. Discharge planning
c. Monitoring and surveillance

Ans c

101. A patient finds their 2-weeks-old infant unresponsive. The infant is


limp, cyanotic and pale. There is no respiration, while the skin is
cold to the touch. The parent begins resuscitation, and the infant is
transferred to the hospital where the infant expires. The MOST
likely diagnosis is:

a. Sudden infant death syndrome


b. Apparent life-threatening event
c. Apnea of infancy
d. Apnea of unknown origin

• Answer : A

‫مناقشة امتحانات البرومتريك للتمريض‬

https://www.facebook.com/groups/1639727976293954 Page 49
Prometric exam collection 2017

)‫)النسخة الرابعة‬

4‫ودالشمال‬

102. A neonatal nurse performs Apgar assessment at 1 minute of birth


to evaluate the physical condition of the newborn and immediate
need for resuscitation. At 1 minute, Apgar score is 7. At 5 minutes
Apgar score is to the progression of scores suggests:

a. A healthy newborn
b. The need for supplement oxygen
c. A genetic defect
d. The infant is becoming stable

• Answer : A

103. The nurse is caring for full-term newborn who was delivered
vaginally 5minutes ago. The infant’s APGAR Score was 8 at one
minute and 10 at 5minutes. Which of the following has the highest
priority?

a. Maintaining the infant in the supine position


b. Assessing the infant’s red reflex
c. Preventing heat loss from the infant
d. Administering humidified oxygen to the infant

• Answer : A

104. Which of the following can be used to determine if a prescribed


pain management therapy is effective for a non-verbal patient?
‫مناقشة امتحانات البرومتريك للتمريض‬

https://www.facebook.com/groups/1639727976293954 Page 50
Prometric exam collection 2017

)‫)النسخة الرابعة‬

4‫ودالشمال‬

a. Papanicolaoutest
b. Faces rating scale
c. Braden’s scale
d. Apgar assessment tool

• Answer : B

105. While caring for a neonate with a meningocele, the nurse should
AVOID positioning the child on the:

a. Abdomen
b. Left side
c. Right side
d. Back

• Answer : D

106. A patient with exacerbation of congestive heart failure has a


nursing diagnosis of excess fluid volume. The nurse monitors
fluids intake and output and administers furosemide, as ordered.
Which of the following indicates the efficacy of the intervention?

a. The patient has pitting edema


b. The patient has shortness of breath
c. The patient has a decrease in weight
‫مناقشة امتحانات البرومتريك للتمريض‬

https://www.facebook.com/groups/1639727976293954 Page 51
Prometric exam collection 2017

)‫)النسخة الرابعة‬

4‫ودالشمال‬

d. The patient has jugular vein distention

• Answer : C

107. A 62-year-old patient has been treated for congestive heart failure
and a Nursing diagnosis of fluid volume excess. After diuretic
therapy and dietary Interventions, the patient has met all short-term
goals. The nurse should:

a. Revise the care plan with a diagnosis of risk for alteration in


fluid balance
b. Add a new diagnosed of risk of fluid volume deficit
c. Discontinue the care plan as the diagnosis is resolved
d. Continue the care plan as written

• Answer : D

108. A patient with congestive heart failure and severe peripheral


edema has a nursing diagnosis of fluid volume excess. What are
the two MOST important interventions for the nurse to initiate?

a. Diuretic therapy and intake and output


b. Nutritional education and low-sodium diet
c. Daily weights and intake output
d. Low-sodium diet and elevate legs when in bed

‫مناقشة امتحانات البرومتريك للتمريض‬

https://www.facebook.com/groups/1639727976293954 Page 52
Prometric exam collection 2017

)‫)النسخة الرابعة‬

4‫ودالشمال‬

• Answer : A

109. A patient has exacerbation of congestive heart failure, with one of


the nursing diagnosis being excess fluid (Lasix). The nurse closely
monitors fluid intake and output and administers furosemide
(Lasix). Which of the following indicates the efficacy of the
nursing intervention?
a. The patient has leg edema
b. The patient has shortness of breath
c. The patient has decreased in weight
d. The patient has jugular vein distention

• Answer : C

110. When caring for a patient with an ostomy, the nurse knows that
extra skin protection for the peristomal skin is MOST important for
those with a(n):

a. Ileostomy
b. Ascending colostomy
c. Transverse colostomy
d. Sigmoid colostomy

• Answer : B
‫مناقشة امتحانات البرومتريك للتمريض‬

https://www.facebook.com/groups/1639727976293954 Page 53
Prometric exam collection 2017

)‫)النسخة الرابعة‬

4‫ودالشمال‬

111. While evaluating the nutritional intake of a bedridden patient with


multiple pressure sores, the nurse should make sure the patient
INCREASES the intake of:

a. Protein-rich foods
b. Water Foods
c. Rich in vitamin
d. A Fiber rich foods

• Answer : A

112. In what position should a dyspnea patient be placed?

a. Prone
b. Recumbent
c. Semi-fowler’s
d. Trendelenburg

• Answer : C
113. A patient presents to the clinic for a routine visit and has the
following vital signs: temperature 37.00C (98.60F), heart rate 82,
respiration rate 18 and blood pressure level of 130/94 mmHg.
Which vital sign is abnormal?

‫مناقشة امتحانات البرومتريك للتمريض‬

https://www.facebook.com/groups/1639727976293954 Page 54
Prometric exam collection 2017

)‫)النسخة الرابعة‬

4‫ودالشمال‬

a. Temperature
b. Pulse
c. Respiration
d. Blood pressure

• Answer : D

114. A female patient admitted for abdominal pain complains of


generalized pain, nausea vomiting and constipation. Nursing
assessment finds: temperature,38.60C (101.50F), heart rate-92;
respiration rate-18; blood pressure level, 130/68mmHg. The
patient has rebound tenderness and abdominal rigidity. In the past
hour, her pain has localized on the right side. The nurse suspects:

a. Intestinal obstruction
b. Influenza
c. Appendicitis
d. Pyloric Stenosis

• Answer : C
115. A community health nurse is implementing an adult immunization
program in the neighborhood. Which of the following would
MOST likely be a universally recommended adult vaccination and
dose frequency general population?

a. Tetanus-diphtheria toxoid every 20 years


‫مناقشة امتحانات البرومتريك للتمريض‬

https://www.facebook.com/groups/1639727976293954 Page 55
Prometric exam collection 2017

)‫)النسخة الرابعة‬

4‫ودالشمال‬

b. Pneumococcal vaccination every 2 years


c. Influenza vaccination every year
d. One time typhoid vaccine followed by boosters every5 years

• Answer : C

116. A 6-year-old patient has been diagnosed with acute rheumatic


fever. Then nurse knows that the antibiotic of choice for this illness
is:

a. Bezathgine penicillin(Megacillin)
b. Amoxicillin (Amoxil)
c. Erythromycin (Eryhrocin)
d. Vancomycin (Vancocin)

• Answer : A

117. A child is admitted to the hospital with congenital heart disease.


Which of the following nursing diagnoses should receive
PRIORITY?

a. Decreased cardiac output related to decreased myocardial


function
b. Activity intolerance related to cachexia
c. Impaired gas exchanged related to altered pulmonary blood flow

‫مناقشة امتحانات البرومتريك للتمريض‬

https://www.facebook.com/groups/1639727976293954 Page 56
Prometric exam collection 2017

)‫)النسخة الرابعة‬

4‫ودالشمال‬

d. Imbalanced nutrition: less than body requirements related to


excessive energy demands

• Answer : A

118. Prior to initiating therapy with unfractionated heparin for a patient


hospitalized with a deep vein thrombosis, the nurse should plan to:
a. Weigh the patient
b. Administer aspirin
c. Limit fluid intake
d. Undress the patient

• Answer : B

119. Prior to initiating therapy with unfractionated heparin for a patient


hospitalized with a deep vein thrombosis, this treatment requires:

a. Bed rest
b. Aspirin therapy
c. Fluid restrictions
d. A high protein diet

• Answer : B

‫مناقشة امتحانات البرومتريك للتمريض‬

https://www.facebook.com/groups/1639727976293954 Page 57
Prometric exam collection 2017

)‫)النسخة الرابعة‬

4‫ودالشمال‬

120. A patient with chronic liver disease secondary to hepatitis C has


been admitted with malnutrition. With a nursing diagnosis of
alteration in nutrition, less than body requirements, the BEST long-
term goal is the patient will:
a. Gain at least 10% of bodyweight
b. Attain and maintain ideal weight
c. Verbalize understanding nutritional needs
d. Include high quality protein in diet

Answer: B

121. The nurse is assessing a patient with a history of a seizure disorder.


While checking the patient’s vital signs, the patient develops
rhythmic, jerking movements of the arms and legs. The nurse
should IMMEDIATELY place the patient in which of the
following positions?

a. Prone
b. Supine
c. Semi-fowler’s
d. Lateral

• Answer : D

122. A nurse is assessing to care for a child with a seizure disorder. The
nurse observes the child becomes stiff and lose consciousness,
‫مناقشة امتحانات البرومتريك للتمريض‬

https://www.facebook.com/groups/1639727976293954 Page 58
Prometric exam collection 2017

)‫)النسخة الرابعة‬

4‫ودالشمال‬

following by jerking movements for 1 minute after which the child


becomes very sleepy, which of the following types of seizures
occurred?

a. Absence (petit mal)


b. Generalized (tonic-clonic)
c. Partial Psychomotor(temporal lobe)
d. Status epilepticus

• Answer : B

123. A patient is scheduled for an abdominal aneurysm repair. This is


what type of surgical intervention?

a. Diagnostic
b. Transplant
c. Curative
d. Palliative

• Answer : C

124. A Community health nurse is teaching a health class about


infectious disease processes. The nurse instructs the class that
rabies would be considered which of the following types of
infection?

‫مناقشة امتحانات البرومتريك للتمريض‬

https://www.facebook.com/groups/1639727976293954 Page 59
Prometric exam collection 2017

)‫)النسخة الرابعة‬

4‫ودالشمال‬

a. Viral
b. Protozoan
c. Fungal
d. Bacterial

• Answer : A,

125. A patient receiving chemotherapy developed some raised; red


edematous wheals on the skin, which of the following care plan
alter natives MOST likely need to occur before the treatment?

a. Rain forced relaxation techniques


b. Continue chemotherapy without change
c. Continue with radiation therapy only
d. Pre-medicate the patient with an antihistamine

• Answer : D

126. A 6-year-old patient has presented to the clinic with fever, malaise
and anorexia. The patient was treated 2 weeks ago for a
streptococcal infection of the throat. The nurse should expect the
physician to order what test?

a. Electrocardiogram
b. Jones test
c. Spinal tap
‫مناقشة امتحانات البرومتريك للتمريض‬

https://www.facebook.com/groups/1639727976293954 Page 60
Prometric exam collection 2017

)‫)النسخة الرابعة‬

4‫ودالشمال‬

d. Heart biopsy

• Answer : B

127. A community is experiencing an outbreak of staphylococcal


infections. The nurse instructs residents that the MOST common
mode of transmission is by:

a. Respiratory droplets
b. Contaminated foods
c. Hands
d. Soil

• Answer :D

128. A hospitalized patient has fallen from bed. The nurse notes
shortening of the left leg.Pain upon movement of the left leg, and
rapid, swallow respirations. What action should the nurse take
FIRST?

a. Call for help


b. Immobilize the left leg
c. Obtain blood pressure
d. Evaluate lung sounds

• Answer : B
‫مناقشة امتحانات البرومتريك للتمريض‬

https://www.facebook.com/groups/1639727976293954 Page 61
Prometric exam collection 2017

)‫)النسخة الرابعة‬

4‫ودالشمال‬

129. A community health nurse visits a patient who has suffered a


stroke. The patient’s spouse explains to the nurse that the patient
chokes on foods at times. Which of the following referral ordered
would the nurse anticipate needing for this patient?

a. Speech therapist
b. Dietician
c. Physician therapist
d. Neurologist

• Answer : A

130. A 59-year-old patient arrives in the emergency department


diaphoretic and complains of chest pain and shortness of breath.
The patient’s sibling states that this has happened before and it is
just anxiety. Upon evaluation the physician diagnosis unstable
angina and prescribes anti-anginal medications. What is the
expected results of this drug therapy

a. Balanced between oxygen supply and demand


b. Increase in blood flow to the heart
c. Reduction in oxygen demand and consumption
d. Vessel relaxation

• Answer : B
‫مناقشة امتحانات البرومتريك للتمريض‬

https://www.facebook.com/groups/1639727976293954 Page 62
Prometric exam collection 2017

)‫)النسخة الرابعة‬

4‫ودالشمال‬

131. A patient with end-stage cardiomyopathy and angina pectoris to


the office complaining of frequent chest pain and severe dyspnea.
With a nursing diagnosed of alteration in comfort, what is the
BEST long term goal for this patients?

a. Perform all activities of daily living without complaints of


chest pain or shortness of breath
b. Verbalize and employ strategies to decrease pain and
increase coronary blood flow
c. Take pain medications around the check and use supplement
oxygen at all times
d. Understand the disease process and accept the limitation that
it places on his lifestyle

• Answer : A

132. A patient has an order for a pneumatic compression device. Which


of the following is an appropriate goal?

a. Reduce the risk deep vein thrombosis


b. Reduce lower extremity edema
c. Reduce lower extremity pain
d. Reduce the risk of phlebitis

• Answer : A

‫مناقشة امتحانات البرومتريك للتمريض‬

https://www.facebook.com/groups/1639727976293954 Page 63
Prometric exam collection 2017

)‫)النسخة الرابعة‬

4‫ودالشمال‬

133. A patient with severe diverticulitis had surgery for placement of


colostomy. The patient is upset, crying and will not look at the
colostomy. Which of the following would be the HIGHEST
priority nursing diagnosis at this time?

a. Knowledge deficit, colostomy care


b. Distorted body image
c. Self-care deficit, toileting
d. Alteration in comfort

• Answer : B

134. A patient presents to the emergency department with complaints of


head ache, dizziness and confusion. Clinical symptoms include
tachypnea and dyspnea with the use of accessory muscles to
facilitate breathing. Which of the following orders would the nurse
MOST likely implement to reduce the patient’s confusion and
disorientation?

a. Oxygen therapy
b. Chest physical therapy
c. Bronchodilators
d. Hydration fluids

‫مناقشة امتحانات البرومتريك للتمريض‬

https://www.facebook.com/groups/1639727976293954 Page 64
Prometric exam collection 2017

)‫)النسخة الرابعة‬

4‫ودالشمال‬

• Answer : A
135. A 6-month-old boy is admitted with a diagnosis of failure to thrive.
According to the growth chart at 3 months of age the infant’s
weight is in which percentile?

a. 25th
b. 5th
c. 10th
d. Below the 5th

• Answer : B

136. A patient is 2-days post-operative hernia repair and has an order


for a dressing change patients has been diagnosed with auto
immune deficiency disease syndrome (AIDS). While performing
the dressing change the nurse should take which of the following
actions?

a. Put the patient in a private room


b. Wear gloves during the dressing change
c. Wear gloves gown, and mask during dressing change
d. Put the patient in reverse isolation

• Answer : C
137. When administering an enema to adult patient, how far should the
nurse insert the tubing into the rectum?
‫مناقشة امتحانات البرومتريك للتمريض‬

https://www.facebook.com/groups/1639727976293954 Page 65
Prometric exam collection 2017

)‫)النسخة الرابعة‬

4‫ودالشمال‬

a. 2.2 to 4.4cm (1 to 2 inches)


b. 4.4 to 6.6cm (1 to 3 inches)
c. 6.6 to 8.8cm (3 to 4 inches)
d. 8.8 to 11cm (4 to 5 inches)

• Answer : C

138. A nurse is implementing nursing interventions to monitor a patient


following kidney surgery. Which of the following complications
would be the MOST likely post-operative risk after renal surgery?

a. Deep vein thrombosis


b. Hemorrhage
c. Nausea
d. Hemiparesis

• Answer : B
139. As per of a neurological assessment, which of the following is
associated with the higher score on the Glasgow coma scale?

a. Eye opening to pain, no verbalization


b. Confused, obey commands
c. Localized pain, abnormal extension
d. Eye opening to speech confused

‫مناقشة امتحانات البرومتريك للتمريض‬

https://www.facebook.com/groups/1639727976293954 Page 66
Prometric exam collection 2017

)‫)النسخة الرابعة‬

4‫ودالشمال‬

• Answer : B

140. While caring for a patient prior to surgery to amputate the leg.
What is the MOST affective measure to prevent phantom limb
sensation after the amputation?

a. Control pain prior to the surgery


b. Make sure the patient understands the procedure
c. Elevate the limb on two pillows
d. Help the patient grieve for the limb

• Answer : D

141. If a patient develops a complication during a blood transfusion, the


nurse’s first action should do to:

a. Stop the transfusion


b. Notify the practitioner
c. Administer an antihistamine
d. Administer an anti-inflammatory medication

• Answer : A

142. A patient has an elevated prothrombin (PT) time. Which


medication should the Nurse consider as a possible cause of the
elevated PT Time?
‫مناقشة امتحانات البرومتريك للتمريض‬

https://www.facebook.com/groups/1639727976293954 Page 67
Prometric exam collection 2017

)‫)النسخة الرابعة‬

4‫ودالشمال‬

a. Rifampin
b. Vitamin K
c. Birth control pills
d. Phenytoin (Dilantin)

• Answer : C

143. A home care nurse visits a patient with a new-below-the knee


amputation. The site of the incision is red, warm and tender with
purulent yellow drainage. The patient has a new prescription for
cephalexin (Keflex) and oxycodone (oxycontin). What would the
nurse instruct the patient to do FIRST?

a. Take oxycodone as soon as possible


b. Take cephalexin as soon as possible
c. Wash the incision site and apply bacitracin cream
d. Wash the incision site and apply hydrocortisone

• Answer : C

144. A patient has the following order: cephalexin (keflex) 500


milligrams (mg) by mouth 4 times a day. The pharmacy has the

‫مناقشة امتحانات البرومتريك للتمريض‬

https://www.facebook.com/groups/1639727976293954 Page 68
Prometric exam collection 2017

)‫)النسخة الرابعة‬

4‫ودالشمال‬

following dose: 250mg per 5milliliters (ml). The nurse should


administer:

a. 5ml
b. 10 ml
c. 15 ml
d. 20 ml

• Answer : B

145. A marathon runner experiences a sudden onset of sharp pain in calf


immediately after a workout. The nurse in the clinic notes mild
swelling of the calf and tenderness to touch. Which of the
following would the nurse suspect the patient is experiencing?

a. Bursitis
b. Tendonitis
c. Plantar fasciitis
d. Joint dislocation

• Answer : C

146. A mastectomy patient has developed lymphedema of the left arm.


The nurse should teach the patient that the BEST position for the
arm is:
‫مناقشة امتحانات البرومتريك للتمريض‬

https://www.facebook.com/groups/1639727976293954 Page 69
Prometric exam collection 2017

)‫)النسخة الرابعة‬

4‫ودالشمال‬

a. Immobilized across the chest


b. Dependent
c. Elevated
d. In traction

• Answer : C

147. A patient is seen in the emergency room for a 20cm (7.8 inch)
laceration to the right fore arm. The course prepares for which type
of anesthesia to be administered before the laceration is repaired by
the physician?

a. Intravenous
b. Regional
c. General
d. Local

• Answer : B
148. A nurse in a community health clinic is in charges of
immunizations. When patients visits the clinic the nurse knows that
immunizations should be reviewed:

a. At the age they are scheduled to be administered


b. One month prior to recommended immunization schedule
c. At every clinic visit
‫مناقشة امتحانات البرومتريك للتمريض‬

https://www.facebook.com/groups/1639727976293954 Page 70
Prometric exam collection 2017

)‫)النسخة الرابعة‬

4‫ودالشمال‬

d. At monthly intervals

• Answer : C

149. A child was admitted to the hospital three hours ago with a closed
head injury. The child responds appropriately but sluggishly to
stimuli, and drift in and out of sleep. Which of the following best
describes this patient’s level of consciousness?

a. Lethargic
b. Obtunded
c. Semi-comatose
d. Comatose

• Answer :B

150. A healthy patient is in doctor’s office for a pre-operative visit


before a total replacement. The nurse interviewing the patient
charts the following medications: aspirin 81 mg once a day,
vitamin E 260 international units once a day, and unknown amount
of a herbal supplement once a day, based on the patient’s
medication list which of the following labs would be important pre
operatively?

a. Prostate specific antigen(PSA)


‫مناقشة امتحانات البرومتريك للتمريض‬

https://www.facebook.com/groups/1639727976293954 Page 71
Prometric exam collection 2017

)‫)النسخة الرابعة‬

4‫ودالشمال‬

b. Blood glucose
c. Creatine phosphokinaseisoenzymes (CPK enzymes)
d. Prothrombin time

Answer: D
151. A patient with long-standing diabetes mellitus (type I) is scheduled
for surgical amputation of 4 gangrenous toes on the right foot.
Which surgical intervention would this be classified as?

a. Palliative
b. Curative
c. Reconstructive
d. Diagnostic

• Answer : A

152. The nurse is caring for a patient who just had a chest tube inserted
due to spontaneous pneumothorax. An appropriate goal is that the
patient will:

a. Be free of pain with in 4hours


b. Report decreased pain
c. Rest quietly
d. Sleep with few movements

‫مناقشة امتحانات البرومتريك للتمريض‬

https://www.facebook.com/groups/1639727976293954 Page 72
Prometric exam collection 2017

)‫)النسخة الرابعة‬

4‫ودالشمال‬

• Answer : C
153. A patient with the deep vein thrombosis (DVT) is being treated
with a low-molecular weight heparin.(LMWH). The patient reports
increased pain in the affected extremely. The nurse observe the
affected extremity has increased in size by 0.2 cm (0.8 inches)
during the past 24 hours. Which of the following actions should the
nurse take?

a. Administer the next dose of LMWH before the scheduled


time.
b. Apply dry heal to the site
c. Elevate the extremity
d. Reinforce the importance of ankle circling exercises

• Answer : C,

154. A physician orders Lactated Ringer Solution to infuse at 125


cc/hour. This is an example of which type of solution?

a. Hypotonic
b. Isotonic
c. Hypertonic
d. Hyper alimentation

• Answer : B
‫مناقشة امتحانات البرومتريك للتمريض‬

https://www.facebook.com/groups/1639727976293954 Page 73
Prometric exam collection 2017

)‫)النسخة الرابعة‬

4‫ودالشمال‬

155. A physician orders an intravenous fluid of D5NS at 100cc/hr. This


is an example of which of the solution?

a. Hypotonic
b. Isotonic
c. Hypertonic
d. Hyper alimentation

• Answer : C

156. A patient is in the preoperative area to lumbar surgery. The patient


reports anxiety about being intubated and expresses concern about
waking up during the surgery. The nurse MUST discuss the
patient’s concern with the

a. Anesthesia provider
b. Surgeon
c. Scrub nurse
d. Charge nurse

• Answer : B

157. The nurse is caring for a patient diagnosed with human immune
deficiency virus. Which of the following nursing diagnoses takes
priority?
‫مناقشة امتحانات البرومتريك للتمريض‬

https://www.facebook.com/groups/1639727976293954 Page 74
Prometric exam collection 2017

)‫)النسخة الرابعة‬

4‫ودالشمال‬

a. Diarrhea related to medication side effects


b. Risk for infection related to inadequate immune system
c. Imbalanced nutrition relate to decreased appetite
d. Impaired tissue integrity related to cachexia and
malnourishment

• Answer : B

158. A nurse assesses a 3-month-old infant. The patient expresses


anxiety and feeling over whelmed. The nurse offer information on
available parenting support. This level of child abuse prevention is
classified as which of the following?

a. Intervention
b. Primary
c. Secondary
d. Tertiary

• Answer : B

159. The nurse is caring for a patient with a coronary thrombosis who is
receiving prescribed streptokinase (striptease). The patient reports
‫مناقشة امتحانات البرومتريك للتمريض‬

https://www.facebook.com/groups/1639727976293954 Page 75
Prometric exam collection 2017

)‫)النسخة الرابعة‬

4‫ودالشمال‬

the onset of a rash as well as feeling hot while experiencing chills.


The nurse should IMMEDIATELY implemented the plan of care
for:

a. A medication side effect


b. An allergic embolus
c. A Pulmonary embolus
d. Peripheral artery occlusion

• Answer : B

160. The nurse is teaching a patient who was just diagnosed with
narcolepsy. The nurse should teach the patient that which of the
following typically INCREASES the level of fatigue?

a. Taking brief naps


b. Participating in an exercise program
c. Eating large meals
d. Working in a cool environment

• Answer : B
161. The physician has prescribed quinidine polygalacturonate (Apo-
Quinidine), 8.25 mg/kg every 4 hours for a patient who weighs
50kgs. The drug is available as a 275 mg tablet. The nurse should
administer how many tablets for each dose?
‫مناقشة امتحانات البرومتريك للتمريض‬

https://www.facebook.com/groups/1639727976293954 Page 76
Prometric exam collection 2017

)‫)النسخة الرابعة‬

4‫ودالشمال‬

a. 2.5
b. 2
c. 1.5
d. 1

• Answer : C

162. The nurse is teaching the parent of a child with celiac


disease.Which of following diets should be reviewed with the
parent?

a. Gluten-free
b. Dairy free
c. Vegetarian
d. Sodium-restricted

• Answer : A

163. A patient has peripheral vascular disease. The nursing diagnosis is


ineffective tissue perfusion: peripheral.Which of the following is
an appropriate goal?

a. The patient will identify three factors to improve peripheral


circulation
b. The patient will have palpable peripheral pulses in1week
‫مناقشة امتحانات البرومتريك للتمريض‬

https://www.facebook.com/groups/1639727976293954 Page 77
Prometric exam collection 2017

)‫)النسخة الرابعة‬

4‫ودالشمال‬

c. The patient’s feet will be warm to touch


d. The patient will ambulate the length of the hall way

• Answer : B

164. On the second day of hospitalization for ventriculoperitoneal shunt


revision, a child with spina bifida developed hives, itching and
wheezing. The nurse should determine if the patient has been
exposed to:

a. Peanuts
b. Strawberries
c. Eggs
d. Latex

• Answer : D

165. A patient with malignant cancer has decided to stop chemotherapy


and receive hospice care. What is the PRIORITY nursing
diagnosis?

a. Alteration in comfort
b. Hopelessness
‫مناقشة امتحانات البرومتريك للتمريض‬

https://www.facebook.com/groups/1639727976293954 Page 78
Prometric exam collection 2017

)‫)النسخة الرابعة‬

4‫ودالشمال‬

c. Powerlessness
d. Non-compliance

• Answer : B

166. A nurse assessing a 16-month-old child observes bruises scattered


over the body that are at different stage of healing. The child also
has poor and diaper rash. The goal of treatment for this child is to:

a. Ensure the physical and emotional safety of the child


b. Remove the child from the parents
c. Admonish the parents of the child
d. Ensure that the child stays with the biological parents

• Answer : A

167. While visiting a patient who had a left hip replacement surgery one
week ago, the Patient complains to the home care nurse of episodic
numbness and tingling of the lower left extremities. Assessment of
the patient shows that the lower left extremities are slightly cool to
touch when compared to the lower right extremities. There is no
swelling or redness on assessment. What would be the NEXT
nursing intervention?

a. Reassure the patient that this normal after surgery


b. Refer the patient to the surgeon immediately
‫مناقشة امتحانات البرومتريك للتمريض‬

https://www.facebook.com/groups/1639727976293954 Page 79
Prometric exam collection 2017

)‫)النسخة الرابعة‬

4‫ودالشمال‬

c. Encourage the patient to decrease activities involving the left


hip and extremities
d. Refer the patient to a physical therapist immediately

• Answer : C

168. A nurse is evaluating a patient 5 days after a right total hip


replacement. Which of the following goals is appropriate for the
patient?

a. Maintain hip abduction without dislocation


b. Rest with legs elevate while sitting
c. Tie shoes and put on undergarments without assistive devices
d. Perform scissors-like leg exercise daily

• Answer : A

169. Prior to providing care for a hospitalized infant, the nurse MUST:

a. Introduce self to parent


b. Perform hand hygiene
c. Have a witness present
d. Assess the child’s developmental level

• Answer : B

‫مناقشة امتحانات البرومتريك للتمريض‬

https://www.facebook.com/groups/1639727976293954 Page 80
Prometric exam collection 2017

)‫)النسخة الرابعة‬

4‫ودالشمال‬

170. When caring for a patient with new sigmoid colostomy, the nurse
knows that the stoma may be expected to decrease in size from up
to:

a. One months
b. Two months
c. Six months
d. One year

• Answer : A

171. A 7-week-old infant boy is admitted with projectile vomiting


decreased urine output, decreased bowel movements and weight
loss. He has poor turgor and appears hungry. The nurse observes
left-to right peristaltic waves after he vomits. The nurse would
expect to find which of the following during the physical
assessment?

a. Hepato-spleenomegaly
b. A palpable pyloric mass
c. Lymphadenopathy
d. Bulging fontanelles

• Answer : B
‫مناقشة امتحانات البرومتريك للتمريض‬

https://www.facebook.com/groups/1639727976293954 Page 81
Prometric exam collection 2017

)‫)النسخة الرابعة‬

4‫ودالشمال‬

172. A nurse will need to change the dressing on a patient’s central


venous catheter during the shift. The nurse should plan to:

a. Limit the patient’s activity for an hour dressing change


b. Position the patient on to the left side before removing the
old dressing
c. Put on sterile gloves after explaining the procedure to the
patient
d. Cleanse the insertion site using a circular motion

• Answer : B

173. During the postoperative period, a nurse is assigned to care for a


morbidly obese patient with an abdominal incision. The nurse
knows that this patient’s weight increases the risk of:

a. Left-sided heart failure


b. Pressure sores of the coccyx
c. Constipation and ileus
d. Wound dehiscence

• Answer : D

174. Which of the following takes place during the implementation


phase of the nursing process?
‫مناقشة امتحانات البرومتريك للتمريض‬

https://www.facebook.com/groups/1639727976293954 Page 82
Prometric exam collection 2017

)‫)النسخة الرابعة‬

4‫ودالشمال‬

a. Development of a goals and a nursing care plan


b. Identification of actual or potential health problems
c. Actualization of the care plan through nursing
interventions
d. Determination of the patient’s responses to the nursing
interventions

• Answer : C

175. For a patient with a colostomy, which of the following-intervention


is appropriate for preventing the risk of the impaired skin related to
exposure excretions?

a. Empty pouch when it is completely full


b. Remove the skin barrier inspect the skin monthly
c. Recaps Skin barrier opening to size of stoma with each
change
d. Cut an opening in the skin barrier then the circumference of
the stoma

• Answer : B

176. An infant who weighs 9 kg (19.8 lbs) requires 900ml of fluids per
day for maintenance fluids. The infant typically consumes 120ml
‫مناقشة امتحانات البرومتريك للتمريض‬

https://www.facebook.com/groups/1639727976293954 Page 83
Prometric exam collection 2017

)‫)النسخة الرابعة‬

4‫ودالشمال‬

during each feeding. The infant must have how many feedings per
day to meet the fluid maintenance needs?

a. 4
b. 8
c. 10
d. 12

• Answer : B

177. A patient has pacemaker implanted. Which of the following


interventions is appropriate for the nursing diagnosis of risk for
injury?

a. Have patient avoid exposure to magnetic resonance


imaging(MRI)
b. Observe incision site for redness, purulent drainage,
c. Offer back rubs to promote relaxation
d. Instruct patient in dorsiflexion exercises of ankles

• Answer : A

178. A patient undergoing treatment for cancer with bone metastasis is


experiencing severe pain. Which of the following treatment would
‫مناقشة امتحانات البرومتريك للتمريض‬

https://www.facebook.com/groups/1639727976293954 Page 84
Prometric exam collection 2017

)‫)النسخة الرابعة‬

4‫ودالشمال‬

the nurse MOST likely expect to improve the patient’s pain


control?

a. Adjuvant radiation therapy


b. Palliative radiation therapy
c. Curative radiation therapy
d. Radio surgery (stereotactic)

• Answer : B

179. A home care nurse visits a patient with diabetes. The patient cast
three well balanced meals sweet dessert and exercises 30 minutes a
day twice a week. Also, the patient is complaint with taking
hypoglycemia medications Blood glucose level ranges from 150-
200 mg/dl. The nurse sets a goal of eliminating sweet desserts and
increasing the frequency of exercises to 3 times a week. This week,
the patient exercised 3 times for 30 minutes and ate dessert only
after dinner. The glucose ranges from 100-150 mg/dl. The nurse
evaluate that:

a. The goal will not be met


b. Progression is being made towards the goal
c. The goal is met
d. The goal is inappropriate

‫مناقشة امتحانات البرومتريك للتمريض‬

https://www.facebook.com/groups/1639727976293954 Page 85
Prometric exam collection 2017

)‫)النسخة الرابعة‬

4‫ودالشمال‬

• Answer : B

180. A nurse is assigned to care for a patient with an ileostomy. The


nurse would expect the ostomy discharge to be:

a. Fluid mushy
b. Mushy
c. Liquid
d. Solid

• Answer : C

181. A nurse educates a patient diagnosed with diabetes, on the


importance of exercise and a well-balanced, low-carbohydrate diet.
The patient takes metforin(Glucophage) 500 mg once a day. Which
following indicates the patient’s plan of Care needs to be re-
evaluated?

a. Blood glucose level is 90mg/dl


b. HbA1C (glycosylated hemoglobin)level is 9.0%
c. Total H DL level is 60mg/dl
d. Low density Lipoprotien is130 mg/dl

• Answer : B

‫مناقشة امتحانات البرومتريك للتمريض‬

https://www.facebook.com/groups/1639727976293954 Page 86
Prometric exam collection 2017

)‫)النسخة الرابعة‬

4‫ودالشمال‬

182. A nurse schedules a patient for a surgical procedure to take place


in 1week. When would the nurse MOST likely implement surgical
education?

a. After admission to the hospital


b. Start during this visit
c. Immediately prior to anesthesia
d. After the operation

• Answer : B

183. The nurse is inserting a nasogastric (NG) tube into a patient as


prescribed. The nurse has advanced the tube into patient’s posterior
pharynx. The nurse should ask the patient to:

a. Hold the breath


b. Stare upwards with eyes towards the ceiling
c. Perform the valsalvas maneuver
d. Lower the chin towards the chest

• Answer : B

184. A home care nurse visits a diabetic patient who was started on
insulin injections. Upon examination, the nurse observes small
lumps and dents on the right upper arm where the patient has
injected insulin. What is the BEST nursing intervention?
‫مناقشة امتحانات البرومتريك للتمريض‬

https://www.facebook.com/groups/1639727976293954 Page 87
Prometric exam collection 2017

)‫)النسخة الرابعة‬

4‫ودالشمال‬

a. Refer patient to dermatologist for diabetic cellulites


b. Instruct the patient to rotate the sites of injection
c. Refer patient to an end for better control of glucose level
d. Instruct patient to inject in the muscular area instead of endo
area

• Answer : A

185. After cardiac surgery, a patient has been prescribed low-sodium,


low cholesterol diet. Which of the following menus is
BEST?

a. Salami, rye bread, sanerkrant


b. Baked chicken thigh, iceberg lettuce, sliced tomatoes
c. Pasta with canned tomato sauce, peas, wheat bread
d. Bacon, lettuce and tomato sandwich with mayonnaise dressing

• Answer : C

186. A home health nurse visits a patient with chronic obstructive


pulmonary disease (COPD) using home oxygen at 2 liters per
minute. The patient reports periods of shortness of breath and
inquires about increasing the oxygen to 4 liters/minute. The nurse
explains that increasing the supplemental oxygen will:

‫مناقشة امتحانات البرومتريك للتمريض‬

https://www.facebook.com/groups/1639727976293954 Page 88
Prometric exam collection 2017

)‫)النسخة الرابعة‬

4‫ودالشمال‬

a. Increased activity tolerance


b. Suppress the hypoxic drive
c. Alleviate the shortness of breath
d. Prevent lung infection

• Answer : B

187. The nurse should avoid the use of the dorsogluteal site for an
intramuscular injection in children because of the risk of injury to
which of the following nerves?

a. Vagus
b. Sciatic
c. Llioinguinal
d. Lumbar plexus

• Answer : B

188. Twelve hours after removal of a benign liver tumor, the nurse
observed that the patient has decreasing blood pressure,
decreasing pulse pressure, increasing heart rate and increasing
respiratory rate. The patient’s skin is cool and pale after lowering
the head of the bed, what should the nurse do next?

a. Call the physician


b. Administer pain medication
‫مناقشة امتحانات البرومتريك للتمريض‬

https://www.facebook.com/groups/1639727976293954 Page 89
Prometric exam collection 2017

)‫)النسخة الرابعة‬

4‫ودالشمال‬

c. Position the patient on the left side


d. Apply cool, wet cloths under the arm

• Answer : A

189. The nurse is assigned to care for an elderly patient with a low-
exudates stage III pressure ulcer, which of the following types
of dressings would the nurse MOST likely plan to use?

a. Hydrogel
b. Hydrocolloid
c. Polyurethane
d. Polyurethane foam

• Answer : B

190. A patient with an unnecessary gait and a history of falls has a


care plan intervention that includes keeping the walker in reach
and pathway free of obstacle. On evaluation after 1 week, the
patient has had no falls, but the gait remains unsteady. The nurse
should:

a. Continue the plan of care as written


b. Allow the patient to replace the walker with a cane
‫مناقشة امتحانات البرومتريك للتمريض‬

https://www.facebook.com/groups/1639727976293954 Page 90
Prometric exam collection 2017

)‫)النسخة الرابعة‬

4‫ودالشمال‬

c. Allow the patient to ambulate short distance without the


walker
d. Have the patient practice stepping over small objects

• Answer : A

191. The nurse is caring for a patient who had a total proctocolectomy
24 hours ago due to a malignant neoplasm in the rectum. The
patient continues to receive intravenous fluids and has started a
clear liquid diet. The nurse understands that the patient is at
INCREASED risk for which of the following postoperative
complications?
a. Dissemination intravascular coagulopathy (DIC)
b. Atelectasis
c. Syndrome of inappropriate anti-diuretics
hormone(SIADH)
d. Hypokalemia

• Answer : D

192. When doing community-based teaching for latex allergies, the


nurse should plan to teach the patient that :

‫مناقشة امتحانات البرومتريك للتمريض‬

https://www.facebook.com/groups/1639727976293954 Page 91
Prometric exam collection 2017

)‫)النسخة الرابعة‬

4‫ودالشمال‬

a. Food handled by people wearing latex gloves stimulates an


allergies response
b. Food containing nuts may trigger an allergic cross-response
in people with latex allergies
c. The patient should wear a face while in the hospital due to
large amount of airborne latex
d. Hoses used on gases pumps contain latex and should be
avoided.

• Answer : B

193. The nurse is assessing 16-month old girl. The nurse observes poor
hygiene, diaper rash and bruises over the child’s body that is at
different stages of healing. Which of the following interventions
would reduce fear and promotes the trust of the child?

a. Avoid scaring the child by saying “No or setting limits


b. Challenge the information the parents give regarding the
injury
c. Question the parents of the child regarding the abuse
d. Assign one nurse to care for the child over the course of
hospital stay

‫مناقشة امتحانات البرومتريك للتمريض‬

https://www.facebook.com/groups/1639727976293954 Page 92
Prometric exam collection 2017

)‫)النسخة الرابعة‬

4‫ودالشمال‬

• Answer : C

194. A patient is who is prepared for hip surgery has an order for
external pneumatic compression devices. The nurse teaches the
patient that pneumatic compression can help prevent:

a. Upper respiratory infection


b. Decreased breath sounds
c. Deep vein thrombosis
d. Bleeding at the surgical site

• Answer : C

195. A patient presents with a productive cough with a moderate


amount of while Frothy sputum and dispend. The patient is
anxious and the nurse notices on assessment that the patient is
using accessory muscle including intercostals spaces to breathe and
has jugular vein distention. The patient has a history of
hypertension and heart failure. What should the nurse administer
FIRST?

a. Digoxin (lanoxin) toimprovethe abilityof the heart topump


effectively
‫مناقشة امتحانات البرومتريك للتمريض‬

https://www.facebook.com/groups/1639727976293954 Page 93
Prometric exam collection 2017

)‫)النسخة الرابعة‬

4‫ودالشمال‬

b. Oxygen therapy to combathypoxemia


c. Furosemide (lasix) toreduce blood volume andpulmonary
congestion
d. Morphine sulface(Duramorph) to reduceanxiety

• Answer : A

196. A nurse is preparing to meet with an individual whose spouse


recently diagnosed with Alzheimer’s disease. The nurse should
know that the primary goals of treatment are:

a. Curing the Alzheimer’sdisease


b. Maximizing the functional ability and improve quality of life
c. Having the Alzheimer’s patient placed in a safe controlled
environment
d. Making all decisions for the patient and confirming to home

• Answer : B
197. A Community nurse interviews an 87-year-old patient diagnosed
with early Alzheimer’s disease. Because the patient provides
conflicts information, the nurse compares subjective and objectives

‫مناقشة امتحانات البرومتريك للتمريض‬

https://www.facebook.com/groups/1639727976293954 Page 94
Prometric exam collection 2017

)‫)النسخة الرابعة‬

4‫ودالشمال‬

data to find a possible reason for the conflicting data. This process
of assessment is called:

a. Data verification
b. Analytical interpretation
c. Mental assessment
d. Subjective observation

• Answer : A

198. The nurse assesses an elderly patient for health problem. The
family reports that the patient has trouble remembering and they
are concerned about Alzheimer’s. Which of the following are risk
factors for Alzheimer’s disease?

a. Genetic history and male gender


b. Ethnic group and dietary habits
c. Genetic history and female gender
d. Dietary habits and male gender

• Answer : C

‫مناقشة امتحانات البرومتريك للتمريض‬

https://www.facebook.com/groups/1639727976293954 Page 95
Prometric exam collection 2017

)‫)النسخة الرابعة‬

4‫ودالشمال‬

199. A patient with Alzheimer’s disease has a fall, which results to a


fracture of the right leg, after repair of the fracture the patient is
discharged home with family with instructions of wound care, the
family verbalizes that the patient has been doing well ,which of
the following instructions would the nurse give to the family?

a. Instruct the family how to provide skin integrity


b. Suggest to the family that if the stress is overwhelming
,placement in a skilled nursing facility may be needed
c. Suggest collecting the patient on a regular schedule and
applying incontinence brief at all times
d. Assess for the cause of incontinence and add an appropriate
nursing diagnosis post and interventions

• Answer : A

200. A nurse assists a patient with Alzheimer’s disease in teeth


brushing. The patient indicates warning to complete the task alone,
but is unable to get the toothpaste on the toothbrush. The nurse can
MUST effectively help the patient by:

a. Providing privacy to complete the task


b. Completing task
c. Providing hand-over-hand assistance with the task
‫مناقشة امتحانات البرومتريك للتمريض‬

https://www.facebook.com/groups/1639727976293954 Page 96
Prometric exam collection 2017

)‫)النسخة الرابعة‬

4‫ودالشمال‬

d. Telling the patient to brush the teeth today

• Answer : C

201. A child with iron deficiency complains of feeling tired all the
times. The nursing diagnosis of fatigue is related to:

a. A decreased ability of the blood to transparent oxygen to the


tissues
b. An increased paroxysmal abdominal pain and distension to
the stomach
c. A decreased anxiety level during hospitalization
d. A decreased nutritional intake with mal absorption of
nutrition

• Answer : A

202. A patient arrives in the emergency room with burns over the upper
trunk and arms. The nurse should obtain the patient’s pulse at
which of the following arterial location?

a. Radial
‫مناقشة امتحانات البرومتريك للتمريض‬

https://www.facebook.com/groups/1639727976293954 Page 97
Prometric exam collection 2017

)‫)النسخة الرابعة‬

4‫ودالشمال‬

b. Carotid
c. femoral
d. Apical

• Answer : C

203. A patient with a spinal cord injury states, “I have no control over
my situation, I can’t do anything for myself”. This patient is
exhibiting:
a. Powerlessness
b. Delusions
c. Suicidal ideation
d. Resignation

• Answer : D

204. A nurse is teaching a prenatal class to a group of the first time


mothers, each at different points in their gestation, which of the
statement is TRUE regarding the management of fatigue?

a. Rest flat on back, especially during the third trimester


b. Exercise programs should focus on their training
c. Frequent 15 minute to 30minute rest periods are important
d. Six hours of sleep a night is adequate
‫مناقشة امتحانات البرومتريك للتمريض‬

https://www.facebook.com/groups/1639727976293954 Page 98
Prometric exam collection 2017

)‫)النسخة الرابعة‬

4‫ودالشمال‬

• Answer : C

205. A nurse is caring for a postoperative patient who is on


subcutaneous, low dose heparin. This medication is used to
prevent:

a. Deep vein thrombosis


b. Congestive heart failure
c. Paralytic Ileus
d. Pneumonia

• Answer : A
206. A Patient is recovering following surgery for placement of a
colostomy. The nurse goes to the patient’s room to instruct the
patient how to care for the colostomy. The patient’s roommate has
visitors and the patient does not want to participate at this time.
What should the nurse do?

a. Document the patient’s refusal and add non-compliance to


the care plan
b. Tell the patient that this is vital information and may delay
discharged
c. Plan a time convenient to both the patient and the nurse
d. Pull the curtain around be bed and speak, ensuring privacy

‫مناقشة امتحانات البرومتريك للتمريض‬

https://www.facebook.com/groups/1639727976293954 Page 99
Prometric exam collection 2017

)‫)النسخة الرابعة‬

4‫ودالشمال‬

• Answer : C

207. Which of the following actions would be appropriate for the nurse
to take when Caring for a patient on contact precautions?

a. Serve the patient’s meals on the disposable with plastic


eating utensils
b. Instruct visitors to talk to the nurse before entering the
patient’s room
c. Rinse both hands with water after removing gloves
d. Place a surgical mask on the patient during transport

• Answer : B

208. A patient is recently diagnosed with Herpes Zoster. The nurse


establishing the care plan would MOST likely assign the highest
priority to which of the following nursing diagnosis?

a. Anxiety
b. Social Isolation
c. Peripheral neurovascular dysfunction
d. Acute pain

ANS D
‫مناقشة امتحانات البرومتريك للتمريض‬

https://www.facebook.com/groups/1639727976293954 Page 100


Prometric exam collection 2017

)‫)النسخة الرابعة‬

4‫ودالشمال‬

209. In order to reduce the risk of disease transmission from a patient


with diphtheria, which of the following standard precautions would
be the nurse implemented?

a. Airborne
b. Contact
c. Droplets
d. Ventilatory

• Answer : C

210. A patient with measles (rubella) is on airborne precautions, which


of the following Precautions techniques would be ESSENTIAL to
implement for non-immune person entering the room?

a. Gloves
b. Gowns
c. Face shields
d. Masks

• Answer : D

‫مناقشة امتحانات البرومتريك للتمريض‬

https://www.facebook.com/groups/1639727976293954 Page 101


Prometric exam collection 2017

)‫)النسخة الرابعة‬

4‫ودالشمال‬

211. A patient sustained multiple musculoskeletal trauma after a motor


vehicle collision and is now in skeletal traction awaiting surgery.
The nurse observes that the patient has developed a large area of
flat, Pin point purple-colored areas on the thorax. Which of the
following actions would be appropriate for the nurse to take?

a. Discontinue the opioid that is being administered


b. Place an extra blanket on the patient
c. Release the weights on the patient’s skeletal traction
d. Administer diphenhydramine(Benadryl) prescribed p.r.n
allergic reaction

• Answer : D

212. A physician has ordered gavage feeding every 4 hours for a 12-
week-old infant with failure to thrive. In order to know how far to
insert the feeding tube. The nurse should measure the distance
from:

a. The infant’s mouth to the xiphoid process of the sternum


b. The tip of the infant’s nose to the ear and then to the
umbilicus
c. The infant’s mouth to the ear and then to the umbilicus
d. The tip of the infant’s nose to the ear and then to the xiphoid
process of the sternum
‫مناقشة امتحانات البرومتريك للتمريض‬

https://www.facebook.com/groups/1639727976293954 Page 102


Prometric exam collection 2017

)‫)النسخة الرابعة‬

4‫ودالشمال‬

• Answer : D

213. A nurse is assessing an infant diagnosed with failure to thrive. In


addition to accurate anthropometric measurements, complete
nutritional history, infant feeding ability, and head-to-toe
assessment the nurse should asses which of the following

a. Parent-to-child interaction
b. Number of sibling in the home
c. Current sleep patterns
d. Exposure to second hand smoke

• Answer : A

214. A school nurse refers a child who failed the school vision
screening for eye doctor. The child returns with glasses to be worn
at all times. The nurse should monitor this child for:

a. Redness of the eye


b. Episodes of seizures
c. Improved vision with glasses
‫مناقشة امتحانات البرومتريك للتمريض‬

https://www.facebook.com/groups/1639727976293954 Page 103


Prometric exam collection 2017

)‫)النسخة الرابعة‬

4‫ودالشمال‬

d. Lazy eye

• Answer : C

215. A2-years-old child in the emergency department exhibits


symptoms of bacterial meningitis. Which of the following tests
confirm or rule out this diagnosed?

a. Magnetic resonanceimaging (MRI)


b. Magneto encephalogram
c. Computed tomography scan(CT)
d. Lumbar puncture (LP)

• Answer : D

216. A patient exhibits clinical manifestation of a pulmonary embolism.


Arterial blood gas (ABG) levels and a chest x-ray are ordered.
Which of the following test is used to diagnose this condition?

a. Computer tomography scan(CT scan)


b. Magnetic resonance imaging (MRI)
c. Pulmonary angiography
d. Pulmonary function test

‫مناقشة امتحانات البرومتريك للتمريض‬

https://www.facebook.com/groups/1639727976293954 Page 104


Prometric exam collection 2017

)‫)النسخة الرابعة‬

4‫ودالشمال‬

• Answer : C

217. A patient is admitted to the emergency department with a sucking,


chest wound has diminished breath sounds or auscultation. Which
of the following interventions would the nurse perform FIRST?

a. Monitor O2 saturation and arterial blood gas (ABG)levels


b. Apply Petroleum Gauze to wound
c. Prepare the patient for emergency thoracentesis
d. Position the patient in an upright position.

• Answer : B

218. A patient has pulmonary embolism. Which of the following


nursing diagnoses has PRORITY?

a. Anxiety related to pain, dyspnea, and concern of illness


b. Risk for injury related to altered hemodynamic status
c. Acute pain related to congestion and possible lung infarction
d. Ineffective breathing pattern related to acute increase in
alveolar dead air space
• Answer: D

219. Which test should be added to the yearly physical of a patient who
has recently turned 50 years old?

‫مناقشة امتحانات البرومتريك للتمريض‬

https://www.facebook.com/groups/1639727976293954 Page 105


Prometric exam collection 2017

)‫)النسخة الرابعة‬

4‫ودالشمال‬

a. Culture and sensitivity


b. Fecal occult blood
c. Routine urine analysis test
d. Angiography studies

• Answer : A

220. The normal range of pH in arterial blood is:

a. 7.15-7.20
b. 7.25-7.30
c. 7.35-7.45
d. 7.50-7.55

• Answer : C

221. To prevent pressure on the feet of a bed-bound patient with


decreased tissue perfusion, the BEST intervention the nurse should
take is:

a. Place sheep skin under the heels


b. Place a foot cradle on the bed
c. Pad the side rails with foam tubing
d. Use only natural fiber linens

‫مناقشة امتحانات البرومتريك للتمريض‬

https://www.facebook.com/groups/1639727976293954 Page 106


Prometric exam collection 2017

)‫)النسخة الرابعة‬

4‫ودالشمال‬

• Answer : D

222. The nurse assists with a lumbar puncture on a child with suspect
bacterial meningitis. If the diagnosis is correct, the cerebrospinal
fluid, should have which of the following qualities?

a. High glucose level


b. Low protein level
c. Cloudy or turbid appearance
d. Pink or blood-tinged appearance

• Answer : C

223. An elderly patient with severe degenerative joint disease comes to


the clinic for routine follow up of pain management. The patient
reports that over the past month, the pain has begun to increase in
severity. The patient requests an increase in dosage of the pain
medication. The nurse recognize that this is MOST likely due to:

a. Drug addiction
b. Drug tolerance
c. An improvement in condition
d. Lack of efficacy of the current medication

• Answer : D

‫مناقشة امتحانات البرومتريك للتمريض‬

https://www.facebook.com/groups/1639727976293954 Page 107


Prometric exam collection 2017

)‫)النسخة الرابعة‬

4‫ودالشمال‬

224. A patient has hepatitis B (HBV) and is now a chronic carrier. In


planning care, the nurse would explain an HBV carrier would
MOST likely be at risk for developing a super infection with which
other type of hepatitis?

a. A
b. C
c. E
d. D

• Answer : B

225. A preoperative patient has a large volume cleansing enema


ordered. In order to facilitate the flow of the solution into the
rectum and colon, the nurse should position the patient in the:

a. Supine position with legs flexed to chest


b. Right lateral position with left sharply flexed
c. Supine position with legs spread
d. Left lateral position with right leg sharply flexed

• Answer : D

226. Respiratory depression is a potentially life-threatening adverse


effect of

‫مناقشة امتحانات البرومتريك للتمريض‬

https://www.facebook.com/groups/1639727976293954 Page 108


Prometric exam collection 2017

)‫)النسخة الرابعة‬

4‫ودالشمال‬

a. Opioids
b. Anticoagulants
c. Immune modulators
d. Non-steroidal (NSAIDS)

• Answer : A

227. A child in the postictal state of a seizure should show which of the
following signs or symptoms?

a. Feeling sleepy or exhausted


b. Stiffness over entire body
c. Verbalizes having an aura
d. Eyes fixed in one position

• Answer : A

228. Standards of pain management dictate the nurses:

a. Administer analgesic via injection whenever possible


b. Avoid the use of the word “pain”
c. Screen for pain at each encounter
d. Discourage around-the clock dosage of analgesics

• Answer : C

‫مناقشة امتحانات البرومتريك للتمريض‬

https://www.facebook.com/groups/1639727976293954 Page 109


Prometric exam collection 2017

)‫)النسخة الرابعة‬

4‫ودالشمال‬

229. The nurse observes a patient who is eating. The patient suddenly
stands up, places both hands onto the neck and is unable to speak
when the nurse asks if the patient can speak. The nurse observes
that the patient is neither coughing not cyanotic. The nurse should
IMMEDIATELY:

a. Lay the patient flat before compressing the mediastinal area


b. Insert a finger into the patient’s mouth to feel for any food
c. Stand behind the patient while performing abdominal thrusts
d. Activate the emergency call light near the patient

• Answer : C

230. A patient required long-term antibiotic has a central line catheter


inserted into the right subclavian vein by the physician .Which of
the following must be verified prior to the first use of the catheter?

a. Blood return
b. X-ray
c. Catheter potency
d. Length of catheter

• Answer : B

231. When planning discharge teaching for a patient hospitalized for


treatment of third-degree burns over 30% of the body, the nurse
‫مناقشة امتحانات البرومتريك للتمريض‬

https://www.facebook.com/groups/1639727976293954 Page 110


Prometric exam collection 2017

)‫)النسخة الرابعة‬

4‫ودالشمال‬

knows it is MOST important to include which of the following


instruction regarding the loss of large amounts of serum occurring
with burns and the resulting loss of immune function?

a. Wash hands frequently each day


b. Wear masks while in public spaces
c. Wear supplement oxygen at night
d. Take a multiple vitamin tablet each night

• Answer : A

232. Which of the following is the MOST important discharge planning


instruction for a patient with mononucleosis?

a. Avoid activities that may increase injury to the spleen


b. Avoid crowded areas to prevent the spread of infection
c. Consume vitamin K rich food to decrease the risk of bleeding
d. Take an antibiotics a prescribed to treat infection

• Answer : B

233. Which of the following tests measures the total quantity of


prothrombin In the blood and monitors the effectiveness of
warfarin sodium (Coumadin) therapy and prolonged deficiencies in
the extrinsic factor?

‫مناقشة امتحانات البرومتريك للتمريض‬

https://www.facebook.com/groups/1639727976293954 Page 111


Prometric exam collection 2017

)‫)النسخة الرابعة‬

4‫ودالشمال‬

a. Thrombin time (TT)


b. Prothrombin time (PT)
c. Partial prothrombin time(PTT)
d. Activated partial thromboplastin time (aPTT)

• Answer : B

234. While conducting a class for expected mothers, the nurse explains
the difference between true labor construction and false labor
contraction by indicating that the labor contractions:

a. Are located mainly in the abdomen and groin


b. Have increasing intensity
c. Occur with decreasing intervals
d. Occur at regular intervals

• Answer : B

235. One month after starting new medications for hypertension, a


patient returns to the clinic with blood pressure in the range. The
patient admits to taking the medications only when “feeling bad”
Which of the following actions would the nurse take?
a. Assess further determine the reason the reason for the
patient’s Actions
b. Add a new diagnosis of non-compliance

‫مناقشة امتحانات البرومتريك للتمريض‬

https://www.facebook.com/groups/1639727976293954 Page 112


Prometric exam collection 2017

)‫)النسخة الرابعة‬

4‫ودالشمال‬

c. Re-educate the patient about the importance of following his


medication plan
d. Reevaluate the need for daily medication since the blood
pressure is acceptable

• Answer : C

236. A home care nurse visits a patient who is wheelchair bound due to
recent motor vehicle accident. The patient has been sitting in the
wheel chair for extended periods of time which resulted in the
development of a stage pressure sore on the right buttocks. What is
the BEST nursing intervention?

a. Instruct caretaker to change the patient’s position every 2


hours
b. Apply hydrogel to the stage I pressure sore every 8 hours
c. Refer the patient to wound care specialist for debridement
d. Encourage the patient to consume an increased amount of
calcium

• Answer : A

237. Following an open-cholecystectomy, the nurse would instruct the


patient to expect to resume normal activities in:

‫مناقشة امتحانات البرومتريك للتمريض‬

https://www.facebook.com/groups/1639727976293954 Page 113


Prometric exam collection 2017

)‫)النسخة الرابعة‬

4‫ودالشمال‬

a. 1 to 2 weeks
b. 2 to 3 weeks
c. 4 to 6 weeks
d. 6 to 8 weeks

• Answer : A

238. A patient had a retinal detachment surgically repaired. The nurse


identified that the detachment would MOST likely be correct and
unlikely to reoccur if the retina remains attached at LEAST:

a. 3 days
b. 2 weeks
c. 2 months
d. 3 months

• Answer : B

239. A home care nurse visits an elderly patient who had a surgical
repair for fracture. The patient is taking opioid analgesics. Today,
the patient complaints of decreased appetite and absence of a
bowel movement for four days. Which of the following can be
inferred?

a. Constipation related to use of opioids


b. Decreased appetite due to depression
‫مناقشة امتحانات البرومتريك للتمريض‬

https://www.facebook.com/groups/1639727976293954 Page 114


Prometric exam collection 2017

)‫)النسخة الرابعة‬

4‫ودالشمال‬

c. Constipation due to acute pain


d. Decreased appetite due to use of opioid

• Answer : A

240. A child recently diagnosed with sickle cell anemia is being


prepared for discharge. Which of the following statement by one of
the parents would require ADDITIONAL teaching by the nurse?

a. High altitudes can be beneficial


b. Blood transfusion may be necessary in the future
c. Strenuous physical activity should be avoided
d. Increased fluid intake minimize pain

• Answer : A

241. A 13-year-old child is hospitalized for treatment of sickle cell


crisis. The nurse finds the child is crying and does not answer the
nurse when addressed. What should nurse do FIRST?

a. Interview the parents about the child’s pain tolerance and


usual medication requirements
b. Medicate the patient with the medication ordered for
breakthrough pain as soon as possible, the resume the evaluation
‫مناقشة امتحانات البرومتريك للتمريض‬

https://www.facebook.com/groups/1639727976293954 Page 115


Prometric exam collection 2017

)‫)النسخة الرابعة‬

4‫ودالشمال‬

c. Ask the child to describe the pain, it is located, and to rate it


on the Wong/baker pain scale.
d. Tell the child to rest while and the nurse will return at
another time for the evaluation

• Answer : B

242. The nurse is entering the room of a patient who is blind. The nurse
should:

a. Speak before touching the patient


b. Talk to the patient using aloud tone of voice
c. Ask then patient questions that can be answered “yes “or
“no”
d. Stand directly in front of the patient while talking

• Answer : A

243. A nurse has been visiting a bed-bound patient with decreased


bowel mobility in the home for one month. The family tells the
nurse that the patient is becoming incontinent of feces. The nurse
evaluates the plan of care and notes which of the following
intervention would MOST likely beneficial?
a.
b. An enema two times a week
c. Increased fiber in the diet
‫مناقشة امتحانات البرومتريك للتمريض‬

https://www.facebook.com/groups/1639727976293954 Page 116


Prometric exam collection 2017

)‫)النسخة الرابعة‬

4‫ودالشمال‬

d. Aroutinebisacodyl(Dulcolax) suppository
e. An enema three times a week

• Answer : B

244. A bed-bound patient has a care plan with interventions to include


re positioning every 2 hours. The patient develops a stage I
pressure sore on the right heel. What intervention should be added
to the care plan?

a. Massage the right heel four times per day


b. Add a trapeze to the bed
c. Float heels off bed with a pillow
d. Add a bed cradle to the bed

• Answer : C

245. A patient is receiving from surgery using spinal anesthesia. The


patient develops a spinal headache. Which of the following nursing
actions would be MOST appropriate?

a. Elevate the head of the bed30 degrees


b. Keep the patient well hydrated
c. Limit intake of salty food
d. Lower the temperature of the room

‫مناقشة امتحانات البرومتريك للتمريض‬

https://www.facebook.com/groups/1639727976293954 Page 117


Prometric exam collection 2017

)‫)النسخة الرابعة‬

4‫ودالشمال‬

• Answer : B
246. A nurse is giving discharge planning instruction to the parents of a
1-years old child with acute otitis media. Which of the following
discharge instruction take FIRST priority?

a. Administer antibiotics as prescribed


b. Breastfeed as long as possible
c. Administer influenza vaccination
d. Avoid smoking around the child

• Answer : A
247. Three weeks post amputation of the leg the patient is instructed to
massage the residual limb. The MOST likely rationale for this to:

a. Provide counter-irritation for pain control


b. Prepare for a prosthesis
c. Promote wound healing
d. Promote acceptance of the limb’s appearance

• Answer : B

248. A patient receives a blood transfusion for severe anemia after


surgery. While evaluating the patient the nurse finds that the
patient’s oral temperature has began to rise from 98.20F (36.80F) to
101.00F(38.30C). What should the nurse do?

‫مناقشة امتحانات البرومتريك للتمريض‬

https://www.facebook.com/groups/1639727976293954 Page 118


Prometric exam collection 2017

)‫)النسخة الرابعة‬

4‫ودالشمال‬

a. Give the patient an anti-pyretic medication and continue the


transfusion as ordered
b. Discontinue the intravenous line and restart in another site
c. Stop the transfusion, keep the vein open with normal saline,
and notify the doctor immediately
d. Use a blood cooling device to cool the blood as it infuses

• Answer : C

249. The nurse is teaching a patient who has just diagnosed with
bacterial conjunctivitis, The nurse should that the MOST effective
way to transmission of this to other people is by

a. Putting on clean gloves before cleansing the eye


b. Taking medication as prescribe
c. Wearing a gauze eye patch
d. Performing hand hygiene

• Answer : D

250. A nurse for a child with celiac disease (CD). The patient would
have a permanent inability to tolerate:
a. Protein
b. Dairy
c. Glutens
d. Fruits
‫مناقشة امتحانات البرومتريك للتمريض‬

https://www.facebook.com/groups/1639727976293954 Page 119


Prometric exam collection 2017

)‫)النسخة الرابعة‬

4‫ودالشمال‬

• Answer : C

251. The nurse is caring for a patient who had an acute pulmonary
edema. The nurse should understand that which of the following
prescribed medications will help to reduce the increased pressure?

a. Morphine sulfate
b. Potassium chloride
c. Warfarin sodium(coumadin)
d. Bisacodyl (dulcolax)

• Answer : A

252. When planning discharge teaching for the parent of an infant with
respiratory problems , the nurse should EMPHASIZE

a. Use of supplemental oxygen at night


b. Frequent hand washing
c. Sleeping in the supine position
d. Rice-thickened formula during night-time feedings

• Answer: C

‫مناقشة امتحانات البرومتريك للتمريض‬

https://www.facebook.com/groups/1639727976293954 Page 120


Prometric exam collection 2017

)‫)النسخة الرابعة‬

4‫ودالشمال‬

253. A nurse is caring for a child who is post-tonsillectomy and


adenoidectomy. The nurse should plan to assess which of the
following complications?

a. Pulmonary hypertension
b. Hemorrhage
c. Hearing loss
d. Corpulmonale

• Answer :B

254. A patient has multiple sclerosis and complains of over whelming


fatigue. The nurse would be MOST correct in instruction the
patient to:

a. Conserve energy during activities of daily living


b. Increase muscle strength through aerobic exercise
c. Ignore fatigue and keep working
d. Increase early afternoon intake of caffeine

• Answer : A

255. While caring for an edentulous patient with multiple pressure


sores, the nurse asked by the patient’s spouse to evaluate several
menus, which of the following menus would be MOST
therapeutic?
‫مناقشة امتحانات البرومتريك للتمريض‬

https://www.facebook.com/groups/1639727976293954 Page 121


Prometric exam collection 2017

)‫)النسخة الرابعة‬

4‫ودالشمال‬

a. Steamed carrots, milks and applesauce


b. Tuna fish with mayonnaise, boiled eggs and yogurt
c. Grilled steak, baked potato and peach pie
d. Chicken noodle soup ,banana and cocoa

• Answer : A

256. When administering an oral medication to a toddler, which of the


following interventions should the nurse plan to use?

a. Depress the child’s chin with thumb to open the child’s


mouth
b. Place the medication in a nipple for the child to suck
c. Give the child a small plastic medication cup for day
d. Tell the child that the medication tastes good

• Answer : A

257. The nurse is monitoring a patient’s urine to determine hydration


status what urine color would indicate the BEST hydration?

a. Clear
b. Amber
c. Tea
d. Pale gold
‫مناقشة امتحانات البرومتريك للتمريض‬

https://www.facebook.com/groups/1639727976293954 Page 122


Prometric exam collection 2017

)‫)النسخة الرابعة‬

4‫ودالشمال‬

• Answer : B

258. A patient is being evaluating due to onset of paleness, shortness of


breath and sensations of heart palpitations. Which of the following
component of complete blood count (CBC) should the nurse
review to determine if the patient has anemia

a. Leukocytes
b. Platelets
c. Erythrocytes
d. Thrombocytes

• Answer : C

259. While a nurse is assessing a patient who reports indigestion that


radiates into the jaw. The jaw pain is rated 8 scale of 0 (no pain) to
10 (severe pain). The patient reports the pain started an hour ago.
The nurse should IMMEDIATELY:

a. Assess the patient’s oral temperature


b. Determine what foods the patient ate
c. Place the patient in reverse Trendelenburg position
d. Obtain order and administer morphine sulfate

• Answer : D
‫مناقشة امتحانات البرومتريك للتمريض‬

https://www.facebook.com/groups/1639727976293954 Page 123


Prometric exam collection 2017

)‫)النسخة الرابعة‬

4‫ودالشمال‬

260. An elderly home-bound patient is visited by the community health


nurse. During evaluation, decreased skin turgor is noted. When
asked about fluids intake, the patient states that she does not drink
any fluids after lunch each day, and wake sup very thirsty. The
MOST appropriate question for the nurse to ask is:

a. “How much protein does you normally eats for dinner?”


b. “How much caffeine are you consuming each day?”
c. “Are you having trouble controlling your bladder at night?”
d. “Do you have enough money to buy liquids to drink?”

• Answer : C
261. A nurse is caring for a patient who had rhinoplasty 2-weeks ago.
Which of the following is an expected outcome?

a. Oral mucus membranes dry ,but pink and intact


b. Face and nose free from swelling
c. Able to make needs know, speech therapy started
d. Demonstrate throat clearing while eating

• Answer : B

262. A patient presents to the emergency room with complaints of eye


and drainage. In planning for the examination of the patients
‫مناقشة امتحانات البرومتريك للتمريض‬

https://www.facebook.com/groups/1639727976293954 Page 124


Prometric exam collection 2017

)‫)النسخة الرابعة‬

4‫ودالشمال‬

complaints, which of the following instruction would the nurse


MOST likely select?

a. Sphygmomanometer
b. Thermometer
c. Ophthalmoscope
d. Otoscope

• Answer: C

263. A home health nurse has completed the assessment of a 72-year-


old patient with & gait disturbance that will begin home physical
therapy. During the interview, the patient reported significant
difficulty sleeping more than 4 hours at night. Which of the
following responses would be appropriate for the nurse to make?

a. “Try doing some type of exercise two hours before bedtime”


b. “Drink a cup of warm tea before you go to bed”
c. “ Make sure the bedroom is dark when you get in bed”
d. “ A nap in the middle of the day should help”

• Answer : D

264. A nursing is caring for a 3-weeks-old infant who was just admitted
to the hospital. Which of the following nursing interventions does
NOT support this infant’s basic emotional and social needs?
‫مناقشة امتحانات البرومتريك للتمريض‬

https://www.facebook.com/groups/1639727976293954 Page 125


Prometric exam collection 2017

)‫)النسخة الرابعة‬

4‫ودالشمال‬

a. Provide for continual contact between parents and infant


b. Activity involve parents in caring for the infant
c. Keep the infant’s environment quiet, dim, and free of sensory
stimulation
d. Foster infant-sibling relationship as appropriate

• Answer : C
265. A home care nurse visits a patient who is discharged from a
hospital after a treatment of urosepsis. Which of the following post
discharge normal laboratory result BEST indicates desired
outcome?

a. WBC count
b. Hematocrit
c. Platelet level
d. Potassium level

• Answer : A

266. A nurse visits a patient who is 37-weeks pregnant and asking for
information about breast feeding versus feeding prepared infant
formula. A beneficial reason to breast feed includes:

a. Readily available and economical


b. Keeps a baby full longer
‫مناقشة امتحانات البرومتريك للتمريض‬

https://www.facebook.com/groups/1639727976293954 Page 126


Prometric exam collection 2017

)‫)النسخة الرابعة‬

4‫ودالشمال‬

c. Larger curds than cow’s milk and therefore is easier to digest


d. Encourage greater deposits of subcutaneous fat in an infant

• Answer : A

267. When implementing a feeding schedule for a full term 2-weeks old
infant, the nurse should expect the infant to be fed:

a. 2-4 times per day


b. 6-8 times per day
c. 10-12 times per day
d. 14-16 times per day

• Answer : B

268. A home care nurse makes a follow-up visit to a patient who had
shingles. A month since the onset, the patient pain level is 6 on a
scale of 1 to 10 where 1 is no pain and 10 is greater pain. Two
weeks ago, the pain Level decreases without any caring. The
patient’s condition has:

a. Met the expected outcome


b. Partially met the expected outcome
c. Has not improved
d. Has worsened
‫مناقشة امتحانات البرومتريك للتمريض‬

https://www.facebook.com/groups/1639727976293954 Page 127


Prometric exam collection 2017

)‫)النسخة الرابعة‬

4‫ودالشمال‬

Answer : A

269. The nurse is in public area of the health care facility when an adult
falls to the floor. Which of the following actions should the nurse
take NEXT?

a. Open the airway


b. Determine unresponsiveness
c. Activate the emergency call system
d. Obtain the automatic electronic defibrillator(AED)

• Answer : B

270. When caring for a patient who is receiving anticoagulant


medications, the nurse MUST monitor the patient, for signs of:

a. Skin breakdown
b. Bleeding
c. Pain
d. Confusion

• Answer : B

‫مناقشة امتحانات البرومتريك للتمريض‬

https://www.facebook.com/groups/1639727976293954 Page 128


Prometric exam collection 2017

)‫)النسخة الرابعة‬

4‫ودالشمال‬

271. A patient is being prepared for a right breast biopsy under general
anesthesia. The patient asks the nurse about the surgical scar and
possible postoperative complications. Which of the following
actions would be appropriate for the nurse to take?

a. Review the post operative risks with the patient


b. Notify surgeon about the patient’s questions
c. Complete the patient’s preoperative check list
d. Show the patient photos of breast surgical scars

• Answer : B

272. A patient with bowlegs due to abnormal bone formations and


deformities has a calcium level of 7.5mg/100ml. Which of the
following foods would the nurse MOST likely instruct the patients
to add to a diet?

a. Organ meats
b. Whole grains
c. Egg yolks
d. Lean means

• Answer : C

273. A patient has just diagnosed with hypothyroidism. Which of the


following instructions is correct?
‫مناقشة امتحانات البرومتريك للتمريض‬

https://www.facebook.com/groups/1639727976293954 Page 129


Prometric exam collection 2017

)‫)النسخة الرابعة‬

4‫ودالشمال‬

a. You will need to take thyroid hormone replacement therapy


your Entire life
b. You will need to take thyroid hormone replacement therapy
until your laboratory result
c. You will need to take thyroid hormone replacement therapy
for about 2 months.
d. You will need to take thyroid hormone replacement therapy
for 1 year.

• Ans . A

274. The stages of dying, as identified by Dr. Elizbathkubbler-ross,


occur in what order?
a. Anger, depression, bargaining, denial, acceptance
b. Bargaining ,denial, acceptance, depression
c. Denial, anger, bargaining, depression, acceptance
d. Depression, Denial, Anger, bargaining, acceptance

• Ans.C

275. A co-worker informs that the nurse about experiencing increased


level of stress associated with daily responsibilities to help cope
with professional stress, the nurse should encourage the co-worker
to ;
‫مناقشة امتحانات البرومتريك للتمريض‬

https://www.facebook.com/groups/1639727976293954 Page 130


Prometric exam collection 2017

)‫)النسخة الرابعة‬

4‫ودالشمال‬

a. Make a list of unfinished tasks


b. complete complex mental task before physical tasks
c. Acknowledge daily accomplishments
d. Spend time with colleague away from work

• Ans . B

276. A nurse is caring for a post-operative patient who is on


subcutaneous, low dose of heparin. When administering injection
on the abdomen, the nurse avoids the umbilicus area because of the
possibility of :

a. Entering a larger body vessel


b. Causing increased pain
c. Precipitating hyper ventilation
d. umbilical infection

• Ans . A

277. A pt with conjunctivitis reports the presence of photophobia and


moderate eye drainage. The nurse should teach pt to

a. Avoid touching the eye


b. Use sterile gauze to remove the drainage
c. Darken the room
d. rest in the prone position
‫مناقشة امتحانات البرومتريك للتمريض‬

https://www.facebook.com/groups/1639727976293954 Page 131


Prometric exam collection 2017

)‫)النسخة الرابعة‬

4‫ودالشمال‬

• ANS. C
278. During surgery the pt has the following intake and output: IV fluid
650 cc ,IV antibiotic 50 cc , 1 unit of packed red blood cells 350
cc,nasogastric output 120 cc,estimated blood loss 80 cc,and urine
in the folyes catheter 240 cc.wat is the patient’s total intake

a. 650 cc
b. 700cc
c. 900 cc
d. 1050 cc

• Ans. D
279. A community health nurse assesses a 68-year-old patient who
lives in a group home. The patient reports decreased appetite after
transferring to the group home because the food tastes too bland.
What type of data is the nurse collecting from the above
information?

a. Analytical
b. Derived
c. Objective
d. Subjective

‫مناقشة امتحانات البرومتريك للتمريض‬

https://www.facebook.com/groups/1639727976293954 Page 132


Prometric exam collection 2017

)‫)النسخة الرابعة‬

4‫ودالشمال‬

• Answer : D

280. The home care nurse is providing wound care for a patient. The
nurse evaluates the wound and notes the presence of granulation
tissue in the wound bed. This observation represents which phase
of wound healing?

a. Maturation
b. Inflammation
c. Proliferation
d. Finalization

• Answer : C

281. A nurse is caring for a 3-year-old child with a fractured arm.


Which of the following interventions is the MOST appropriate for
pain management?

a. Administer analgesics when necessary


b. Assess pain once a shift
c. Anticipate pain and intervene early
d. Encourage the use of self-quieting techniques

Answer : A

‫مناقشة امتحانات البرومتريك للتمريض‬

https://www.facebook.com/groups/1639727976293954 Page 133


Prometric exam collection 2017

)‫)النسخة الرابعة‬

4‫ودالشمال‬

282. Which statement by the patient with hyperlipidemia shows a basic


understanding of the disease and it treatment?

a. Exercise has no effect on cholesterol levels


b. Hyperlipidemia is usually symptomatic until significant
target organ damage is done
c. HDL cholesterol level of greater the 60 mg/dl increases the
chance of coronary artery disease
d. Cholestyramine (Quesram)should be taken in the morning
with other medications

• Answer : D

283. A nurse caring for a patient with acute pulmonary edema observes
that the patient’s cough produces white, frothy and that the patient
is extremely dyspneic. The patient has inspiratory and expiratory
wheezing on auscultation of the lungs. The immediate objective of
treatment is to

a. Improve oxygenation
b. Decrease anxiety
c. Improve tissue perfusion
d. Decrease risk for aspiration

• Answer : A

‫مناقشة امتحانات البرومتريك للتمريض‬

https://www.facebook.com/groups/1639727976293954 Page 134


Prometric exam collection 2017

)‫)النسخة الرابعة‬

4‫ودالشمال‬

284. When discussing dietary choices with a patient who is on heparin


therapy, the nurse should teach the patient that which of the
following foods may increase clotting time?

a. Grape fruit
b. Oranges
c. Bananas
d. Red grapes

• Answer : B

285. A 2-month-old child in the emergency department has projectile


vomiting after feeding. The vomitus is non-bilious containing milk
and gastric juices. Immediately after vomiting the child tries to
feed again. The nurse palpates the child’s abdomen during feeding
and notes a firm area to the right of the umbilicus at the upper right
quadrant. Which of the following is consistent with this history?

a. Hypertrophic pyloric stenosis


b. Hirschsprung’s disease
c. Gastro esophageal reflux
d. Tracheoesophagel fistula

• Answer : A

‫مناقشة امتحانات البرومتريك للتمريض‬

https://www.facebook.com/groups/1639727976293954 Page 135


Prometric exam collection 2017

)‫)النسخة الرابعة‬

4‫ودالشمال‬

286. A patient undergoing cancer treatment has developed acute


hypocalcaemia with sign of weakness, nausea and vomiting.
Which of the following would the nurse anticipate to be the initial
treatment?

a. Thiazide diuretic
b. Intravenous normal saline(0.9% NaCl)
c. A potassium supplement
d. Broad-spectrum antibiotic

• Answer : B

287. The patient is receiving mechanical ventilation set at fraction of


inspired oxygen (FIO2) 100%. The nurse should understand that
which of the following can improve this patient’s oxygenation?

a. Adding positive end expiratory pressure (PEEP)


b. Placing the patient in Trendelenburg position
c. Increasing the FIO2
d. Suctioning the patient hourly

• Answer : A

288. Which of the following nursing diagnosis takes PRIORITY for a


patient after gastrointestinal surgery?

‫مناقشة امتحانات البرومتريك للتمريض‬

https://www.facebook.com/groups/1639727976293954 Page 136


Prometric exam collection 2017

)‫)النسخة الرابعة‬

4‫ودالشمال‬

a. Impaired skin integrity related to surgical incision


b. Constipation related to surgery
c. Risk for infection related to surgical incision
d. Acute pain related to surgical incision

• Answer : D

289. Marijuana is an example of a drug classified as schedule:

a. C-I
b. C-II
c. C-III
d. C-IV

• Answer : A

290. A patient with a weight loss of 12 in 60 days has a nursing care


plan written interventions including offering a dietary supplement
three times per day. After 2 weeks, the patient has had another 1%
weight loss. The patient indicates no likely the supplements. The
nurse should:

a. Continue the plan of care as written


b. Replace the supplement with a high calorie food that the
patient likes
c. Encourage the patient drink supplements
‫مناقشة امتحانات البرومتريك للتمريض‬

https://www.facebook.com/groups/1639727976293954 Page 137


Prometric exam collection 2017

)‫)النسخة الرابعة‬

4‫ودالشمال‬

d. Offer smaller amounts of supplement more frequently

• Answer : B

291. The nurse is caring for a patient with magnesium toxicity. Which
of the following clinical manifestation should the nurse anticipate?

a. Paresthesia
b. Decreased deep-tendon reflexes
c. Cardiac palpitations
d. Decreased cardiac output

• Answer : B
292. A patient returning from a3-hour shoulder repair with general
anesthesia is being transported from the operating room (OR) to
the post-anesthesia care unit (PACU). The nurse knows that the
patient is at high risk for injury related to residual anesthesia.
During this time period the patient is at LOWEST risk for

a. Airway Obstruction
b. Vomiting
c. Impaired Circulation
d. a. Fluid volume deficit

• Answer : B
‫مناقشة امتحانات البرومتريك للتمريض‬

https://www.facebook.com/groups/1639727976293954 Page 138


Prometric exam collection 2017

)‫)النسخة الرابعة‬

4‫ودالشمال‬

293. For a patient scheduled for a total pancrectectomy, the nurse


would be instruct the patient that the procedure work MOST likely
cause

a. Pancreatic ascites
b. Chronic pancreatitis
c. Diabetes mellitus
d. Diabetes insipidus

• Answer : C

294. A nurse is assessing an infant for possible deafness. Which of the


following automatic reflexes would the nurse MOST likely check
to best determine whether the child has a serious hearing problem?

a. Blinking
b. Vertical suspension
c. Moro
d. Perez

• Answer : C

295. The nurse is teaching a group about aerobics exercises. When


discussing the target heart rate for exercise, the nurse should state
that this is calculated by:
‫مناقشة امتحانات البرومتريك للتمريض‬

https://www.facebook.com/groups/1639727976293954 Page 139


Prometric exam collection 2017

)‫)النسخة الرابعة‬

4‫ودالشمال‬

a. Counting the number of the heart beats during exercise for 6


sections, then multiply this number by 10
b. Subtracting the chronological age from the number 220
c. Counting then number of heart beats during exercise for 10
seconds, then multiply by 6
d. Subtracting he chronological age from 240

• Answer : C

296. While performing an assessment on a post-surgical patient 2 days


after surgery, the nurse notes shallow and rapid respirations. What
should the nurse do NEXT?

a. Asses the patient from pain


b. Obtain an order from supplemental oxygen
c. Elevate the head of the bed
d. Place a warmed blanket on the patient

• Answer : C

297. A patient is receiving intravenous fluids at a rate of 125


milliliters/hour (ml/hr). What volume fluids will the patient receive
during an 8-hour shift?
a. 1,500 ml
b. 1 liter
‫مناقشة امتحانات البرومتريك للتمريض‬

https://www.facebook.com/groups/1639727976293954 Page 140


Prometric exam collection 2017

)‫)النسخة الرابعة‬

4‫ودالشمال‬

c. 1.5 liters
d. 500 ml

• Answer : B

298. A patient has a history of severe, uncontrolled epistaxis. The


patient’s blood pressure and patient count are normal. The nurse
should teach the patient to

a. Sleep with the head elevated on at least two three pillows


b. Apply firm pressure to the nostrils four times a day
c. Use a cotton-filled applicator to apply a water-soluble
lubricant to the nasal septum twice daily
d. Minimize the intake of caffeine while increasing the intake of
fluids rich in vitamin K

• Answer : B

299. A nurse is caring for an infant with respiratory distress syndrome.


Which of the following nursing intervention is appropriate

a. Measure oxygen saturation level once a shift


b. Suction frequently for 30-45second each time
c. Monitor for symptoms of hyperglycemia
d. Maintain infant temperature between 36.70& 37.80C(970&
980 F)
‫مناقشة امتحانات البرومتريك للتمريض‬

https://www.facebook.com/groups/1639727976293954 Page 141


Prometric exam collection 2017

)‫)النسخة الرابعة‬

4‫ودالشمال‬

• Answer : A

300. A nurse is caring a patient who had a left mastectomy with lymph
node removal seven days ago. The patient asks about exercises to
regain function of the left arm. Which of the following activities
would be MOST appropriate?

a. Walking fingers up the wall


b. Using five pound weights
c. Knitting with a large needle
d. Rhythmic clapping

• Answer : A

301. What occurs during cardiogenic shock and result in inadequate


tissue perfusion?

a. Increased resistance of arterial vessels


b. Decreased effectiveness of the heart as a pump
c. Increased shunting of critical blood flow to heart
d. Decreased capacity of the venous beds

• Answer : B

‫مناقشة امتحانات البرومتريك للتمريض‬

https://www.facebook.com/groups/1639727976293954 Page 142


Prometric exam collection 2017

)‫)النسخة الرابعة‬

4‫ودالشمال‬

302. The nurse is caring for child admitted with viral pneumonia.
Which of the following nursing diagnoses should receive
PRIORITY?

a. Nutrition altered: less than body requirements


b. Ineffective airway clearance
c. Fluid volume deficit
d. Risk for injury

• Answer : B

303. A child has ingested an entire bottle acetaminophen(Tylenol).


Which of the following organs is affected?

a. Liver
b. Brain
c. Kidneys
d. Gallbladder

• Answer : A

304. A patient is seen in the emergency department with complaints of


angina. Nitroglycerin (Nitrostat) is ordered by the physician. This
medication is to be administered via which of the following routes?

a. Intradermal
‫مناقشة امتحانات البرومتريك للتمريض‬

https://www.facebook.com/groups/1639727976293954 Page 143


Prometric exam collection 2017

)‫)النسخة الرابعة‬

4‫ودالشمال‬

b. Buccal
c. Parental
d. Topical

• Answer : B

305. The nurse is teaching a group of patient about hepatitis A(HAV).


The nurse should state that HAV is MAINLY transmitted Via:

a. Blood contact
b. Food
c. Sexual activity
d. Saliva

• Answer : B

306. A child was recently diagnosed with spastic cerebral palsy. Which
of the following statement by the parent would indicate to the
nurse that parent understands teaching about illness?
a. Full recovery is possible
b. This illness should not progress
c. Cerebral palsy is a hereditary disease
d. Surgery can sometimes improve walking

• Answer : D

‫مناقشة امتحانات البرومتريك للتمريض‬

https://www.facebook.com/groups/1639727976293954 Page 144


Prometric exam collection 2017

)‫)النسخة الرابعة‬

4‫ودالشمال‬

307. A patient hospitalized with Crohn’s disease has developed fever


Increased respiratory rate, increased heart rate, chills, diaphoreses,
and increased abdominal discomfort. The nurse knows that patient
has MOST likely developed

a. Intestinal obstruction
b. Intestinal parasite infestation
c. Intestinal perforation
d. Ascites

• Answer : A

308. A child is admitted to the hospital with dehydration. The nurse


should Give PRIORITYto which of the followingnursing
diagnoses?

a. Anxiety related to hospitalization


b. Fluid volume deficit related to vomiting
c. Imbalance nutrition less than body requirementsrelated
diarrhea
d. Risk for infection relatedto presence of invasivelines

• Answer : B

309. The nurse is caring for patient with deep vein thrombosis (DVT).
The patient’s heparin sodium infusion has been discontinued and
‫مناقشة امتحانات البرومتريك للتمريض‬

https://www.facebook.com/groups/1639727976293954 Page 145


Prometric exam collection 2017

)‫)النسخة الرابعة‬

4‫ودالشمال‬

the patient is receiving prescribed warfarin sodium (Coumadin).


The nurse should advise the patient that which of the following
needs to be continued?

a. Daily complete blood count (CBC)


b. Laboratory tests for partial thromboplastin time (PTT)
c. Strict bed rest
d. Wearing elasticized support stockings

• Answer : C

310. When teaching the parents of neonate with spina bifida techniques
to promote bladder emptying, the nurse reviews a technique in
which firm, gentle pressure is applied to the abdomen press
towards the symphysis pubis. This method is known as:

a. Crede’s
b. Intermittent
c. Foley
d. Prophylactic

• Answer : A

311. A 50-year-old patient is being admitted to the hospital in a


vegetative state of unknown etiology what is the PRIORITY
nursing diagnosis?
‫مناقشة امتحانات البرومتريك للتمريض‬

https://www.facebook.com/groups/1639727976293954 Page 146


Prometric exam collection 2017

)‫)النسخة الرابعة‬

4‫ودالشمال‬

a. Risk for impaired skin integrity


b. Impaired swallowing
c. Altered cerebral tissue perfusion
d. Altered thought processes

• Answer : A
312. Prior to administering an enema, the nurse will assist the patient to
assume what position

a. Prone with pillow under knees


b. Left-side with right knee flexed
c. Right-side with left knee flexed
d. On back with head of bed flat

• Answer : B

313. A nurse interview a patient, recently admitted to long term care


facility, to obtain information on the patient’s health perception.
The nurse encourages the patient to elaborate about this change.
Which type of questioning would be MOST effective in this
situation?

a. Analytical
‫مناقشة امتحانات البرومتريك للتمريض‬

https://www.facebook.com/groups/1639727976293954 Page 147


Prometric exam collection 2017

)‫)النسخة الرابعة‬

4‫ودالشمال‬

b. Focused
c. Closed
d. Open-ended

• Answer : D

314. When selecting activities to help develop a child’s fine motor


skills, which of the following would BEST meet this goal?

a. Sorting cardboard objects that are in different shapes


b. Singing while turning the pages of a book that plays music
c. Jumping rope
d. Riding a three-wheeled cycle

• Answer : C
315. 60 years age a patient weighed 73 kilograms (161 pounds). During
the current clinic visit the nurse note the patient has an unintended
weight loss. This weight loss over 6 months would be considered
clinically significant as soon as it reaches the point of being more
than a:

a. 5% loss
b. 8% loss
c. 10% loss
d. 20% loss
‫مناقشة امتحانات البرومتريك للتمريض‬

https://www.facebook.com/groups/1639727976293954 Page 148


Prometric exam collection 2017

)‫)النسخة الرابعة‬

4‫ودالشمال‬

• Answer : D

316. A child with a diagnosis of tetralogy of fallot is scheduled to be


discharged from the hospital the nurse planning discharge
education should instruct the caregivers that during a hyper
cyanotic spell the position MOST likely to benefit the child is:

a. Supine
b. Side-lying
c. Prone
d. Knee-chest

• Answer : B

317. A child is treated for possible acetaminophen (Tylenol) overdose.


The child is currently stable with normal vital signs. Which of the
following organ function system would be MOST affected?

a. Liver
b. Stomach
c. Lungs
d. Heart

• Answer : A
‫مناقشة امتحانات البرومتريك للتمريض‬

https://www.facebook.com/groups/1639727976293954 Page 149


Prometric exam collection 2017

)‫)النسخة الرابعة‬

4‫ودالشمال‬

318. The nurse is caring for a patient with stage III pressure ulcer to the
coccyx. Three days after initiating the plan of care, the nurse
observes that the ulcer has hard black crust covering the center of
the ulcer. The nurse should understand that this indicates

a. Healing
b. Need for debridement
c. Inadequate nutrition
d. Infection

• Answer : A
319. To limit drug interactions, the nurse should advise the parent of
chronically ill child to:

a. Refer to the medications by the generic name


b. Teach the child the name of all medications prescribed
c. Give all medications one hour apart
d. Get all prescriptions filled at the same pharmacy

• Answer : A

320. The nurse receives an order to obtain an arterial blood gas (ABG)
specimen on a patient. The nurse will use the radial artery to obtain

‫مناقشة امتحانات البرومتريك للتمريض‬

https://www.facebook.com/groups/1639727976293954 Page 150


Prometric exam collection 2017

)‫)النسخة الرابعة‬

4‫ودالشمال‬

the specimen. Which of the following will the nurse assess before
puncturing the radial artery?

a. Allen test
b. Partial pressure of arterial oxygen
c. Partial carbon dioxide
d. Prothrombin time

• Answer : A

321. For an infant with hydrocephalus, a nurse should plan to monitor


for what sign or symptom of increased intracranial pressure?

a. High-pitched, shrill cry


b. Decrease in systolic blood pressure
c. Depressed fontanelle
d. Increase in respirations

• Answer : A

322. During surgery requiring general anesthesia, the patient’s heart


stops, Ventilations using the end tracheal tube (ETT) are started
with an ambubag. Which of the following compression to
ventilation rates is correct?

a. 10 to 2
‫مناقشة امتحانات البرومتريك للتمريض‬

https://www.facebook.com/groups/1639727976293954 Page 151


Prometric exam collection 2017

)‫)النسخة الرابعة‬

4‫ودالشمال‬

b. 15 to 2
c. 30 to 2
d. 50 to 2

Answer :C

323. A patient with pneumonia has a temperature, 40 C (104 F); heart


rate 20;respiratory rate 32 and dyspnea patient has an ineffective
airway clearance related to excessive tracheobronchial secretions.
Which of the following interventions would the nurse implement
to enhance the patient’s airway clearance?
a. Administer oxygen as ordered
b. Maintain a comfortable position
c. Increase fluid intake
d. Administer prescribed analgesic

• Answer : A

324. A 57-year-old patient in a hospital clinic is scheduled for a colon


biopsy. The patient speaks a different language than the hospital
staff, but does understand simple communication in the language
of the staff. When conduction patient education prior to the
procedure, the nurse should plan to:

a. Write all communication and avoid speech


b. Raise the volume and pitch of the voice
‫مناقشة امتحانات البرومتريك للتمريض‬

https://www.facebook.com/groups/1639727976293954 Page 152


Prometric exam collection 2017

)‫)النسخة الرابعة‬

4‫ودالشمال‬

c. Obtain an interpreter
d. Smile and nod frequently

• Answer : C

325. The following pain medications are ordered for a patient who had a
right leg debridement.Oxycodone 5 mg every 4 hours as needed
and morphine 5 mg every 4 hours as needed. The nurse
administered oxycodone 2 hours ago, but the patient report pain
Rated 8 on a scale of 0 (no pain) to 10 (Severe pain) as the
dressing change begins.Vital signs are: blood pressure level,
169/98 mmHg; heat rate, 112; Respiration rate 22; temperature
36.7 C (98.1 F).After evaluating the effectiveness of the pain
Medication, what action should the nurse take?

a. Administer additional oxycodone 5 mg


b. Administer morphine 5 mg
c. Change the dressing quickly
d. Encourage deep breathing
• Answer : B

326. A nurse is assessing the peripheral circulation of patient’s


extremities. The chart indicates the patient has edema in both
lower extremities. Which of the following assessment techniques
would the nurse MOST likely use to assess for this?

‫مناقشة امتحانات البرومتريك للتمريض‬

https://www.facebook.com/groups/1639727976293954 Page 153


Prometric exam collection 2017

)‫)النسخة الرابعة‬

4‫ودالشمال‬

a. Inspection and auscultation


b. Inspection and palpation
c. Palpation and percussion
d. Percussion and auscultation

• Answer : B

327. A child is admitted with temperature of 38.5 C (101.3 F), loss of


appetite and vomiting The nurse observes several joints are red,
swollen, warm and tender to touch. A non pruritic rash is on the
child’s trunk. Laboratory test results include an elevate
erythrocyte sedimentation rate (ESR), a positive c- reactive
protein, and an elevated white blood cell count (WBC). The nurse
should initiate the plan of care for:

a. Congestive heart failure


b. Meningitis
c. Rotovirus
d. Acute rheumatic fever

• Answer : D

328. A nurse is caring for a hospitalized diabetic patient with advanced


peripheral recovery. Which of the following nursing action is
MOST important?

‫مناقشة امتحانات البرومتريك للتمريض‬

https://www.facebook.com/groups/1639727976293954 Page 154


Prometric exam collection 2017

)‫)النسخة الرابعة‬

4‫ودالشمال‬

a. Moisturizing the skin with lotion each day


b. Ensuring that foods are not too hot
c. Facing the patient when speaking
d. Assessing the heels for breakdown

• Answer : D

329. A patient in a long-term care facility is in persistent vegetative


state with a right contracture of the right arm and hand. What is the
BEST goal over the next 90days for this patient related to the
nursing diagnosis of impaired mobility?

a. Develop no further contractures


b. Wear an arm and hand splint
c. Have no pain related to the contractures

• ANS C

330. To facilitate self-care for a 2-year-old child with spastic cerebral


palsy, the nurse should recommend:

a. Placing straws into beverage containers


b. Obtaining eating utensils that have large handles
c. Replacing zippers on clothing with metal snaps
d. Purchasing shoes that have an open heels area

‫مناقشة امتحانات البرومتريك للتمريض‬

https://www.facebook.com/groups/1639727976293954 Page 155


Prometric exam collection 2017

)‫)النسخة الرابعة‬

4‫ودالشمال‬

• Answer : D

331. A 21-year-old female is being discharged after a 2-day admission


for pelvic inflammatory disease (PID). Which statement BEST
identifies the patients understanding of follow-up care for PID?

a. “My sexual partner needs to be treated with antibiotics”


b. “It’s OK to resume sexual relation now”
c. “I need to inform any sexual partners I have had in the past
30 days that I had PID”
d. “In order to prevent getting PID I need to continue to take
birth control pills"

• Answer : A

332. A healthy 2-years-old child is brought to the community health


clinic for a routine checkup. At this visit the nurse should
administer the following vaccine:

a. Rotavirus
b. Hepatitis B
c. None at this time
d. Varicella

• Answer : C

‫مناقشة امتحانات البرومتريك للتمريض‬

https://www.facebook.com/groups/1639727976293954 Page 156


Prometric exam collection 2017

)‫)النسخة الرابعة‬

4‫ودالشمال‬

333. During an evaluation at a community clinic, the patient completes


the medical history. Which of the follow is NOT a risk factor for
an acute myocardial infarction?

a. Coronary artery disease


b. Smoking
c. Hemophilia
d. Hyperlipidemia

• Answer : C

334. Which of the following is the MOST common type of


cardiomyopathy in children and is treated with medications such as
digoxin (Lanoxin) and warfarin (Coumadin)?

a. Hypertrophic
b. Dilated
c. Restrictive
d. Diastolic

• Answer : A

335. The responsibility for teaching patients how to take medications


safely when they are discharged from the hospital belongs to the:

a. Nurse
‫مناقشة امتحانات البرومتريك للتمريض‬

https://www.facebook.com/groups/1639727976293954 Page 157


Prometric exam collection 2017

)‫)النسخة الرابعة‬

4‫ودالشمال‬

b. Physician
c. Dietitian
d. Therapist

• Answer : B

336. A nurse is discharging a patient after hospitalization due to


myocarditis. Which of the following statements should be included
in discharge teaching?

a. There is usually some residual heart enlargement


b. May resume previous activities as before hospitalization
c. Avoid immunizations against infectious disease
d. Rapidly beating heart is a common side effect of the illness
and is not dangerous

• Answer : A

337. A nurse is assessing a 4-month-old formula-fed infant. The parent


reports the infant has been irritable, crying excessively, not
sleeping well, and vomiting. Gastro-esophageal reflux is expected.
What nursing intervention should the nurse expect to teach the
parent?
‫مناقشة امتحانات البرومتريك للتمريض‬

https://www.facebook.com/groups/1639727976293954 Page 158


Prometric exam collection 2017

)‫)النسخة الرابعة‬

4‫ودالشمال‬

a. Place the infant in an infant seat after eating


b. Give large frequent feedings
c. Position the child in a swing
d. Thin formula with water

• Answer : C

338. An adult arrived at the outpatient facility due to the onset of chest
pain. The patient suddenly falls to the floor and is unresponsive.
What action should the nurse take NEXT?

a. Activate emergency call system


b. Open the patient’s airway
c. Check for a carotid pulse
d. Administer 2 rescue breaths

• Answer : C

339. A patient suffered a head trauma which resulted in a nasal fracture


requiringsurgical intervention. Which of the following nursing
diagnoses would MOST likely be a problem this patient?

a. Delayed surgical recovery


b. Impaired gas exchange system
c. Ineffective breathing pattern
‫مناقشة امتحانات البرومتريك للتمريض‬

https://www.facebook.com/groups/1639727976293954 Page 159


Prometric exam collection 2017

)‫)النسخة الرابعة‬

4‫ودالشمال‬

d. Risk for perioperative-positioning injury

• Answer : C

340. After administering inhaled corticosteroids to hospitalized child


with asthama, the nurse plans to have the child rinse the mouth and
gargle with water. The nurse knows the rationale for this action is
prevention of:

a. Tooth decay
b. Oral candidiasis
c. Dehydration
d. Hypertrophy of the gums

• Answer : B

341. The nurse is assessing a patient who is 2-weeks postoperative a


kyphoplasty ofL2 and L3.The patient has been participating in
physical therapy and has been doing daily stretching and
strengthening. Which of the following would indicate that the
patient has met discharge goals?
a. Reports pain in legs while sitting
b. Urinating every two hours while awake
c. Fatigue after performing activities of daily living
d. Ambulates outdoors without assistive devices
‫مناقشة امتحانات البرومتريك للتمريض‬

https://www.facebook.com/groups/1639727976293954 Page 160


Prometric exam collection 2017

)‫)النسخة الرابعة‬

4‫ودالشمال‬

• Answer : D

342. When a child is brought to the emergency department with acute


epiglottitis, which of the following nursing diagnoses should
receive PRIORITY?

a. Ineffective airway clearance


b. Activity intolerance
c. Fluid volume deficit
d. Impaired verbal communication

• Answer : A

343. The nurse is reviewing the medication of a patient who is


scheduled for a coronary artery bypass graft (CABG) in three days.
Which of the following medications MUST be discontinued at
least a week prior to surgery?

a. Digoxin (Lanoxin)
b. Furosemide (Lasix)
c. Propranolol hydrochloride(Inderal)
d. Warfarin sodium(Coumadin)

• Answer : D

‫مناقشة امتحانات البرومتريك للتمريض‬

https://www.facebook.com/groups/1639727976293954 Page 161


Prometric exam collection 2017

)‫)النسخة الرابعة‬

4‫ودالشمال‬

344. A patient with pneumonia experiences ineffective airway clearance


related to the presence of thick secretions secondary to infection.
Oxygen saturation is 89% on room air. Which of the following
nursing interventions takes priority?

a. Deliver oxygen with humidity


b. Encourage fluid intake
c. Assist patient into position ofcomfort
d. Inspect sputum for odor andcolor

• Answer : A

345. A nurse is assessing a 5-month-old infant. The parents’ state that


the infant is irritable, crying excessively, vomiting formula (not
projectile), arching, and stiffening. Based on this assessment, what
diagnosis should the nurse anticipate?

a. Esophageal astresia withtracheoesophageal fistula


b. Gastroesophageal reflux
c. Hirschsprung’s disease
d. Celiac disease

• Answer : B

‫مناقشة امتحانات البرومتريك للتمريض‬

https://www.facebook.com/groups/1639727976293954 Page 162


Prometric exam collection 2017

)‫)النسخة الرابعة‬

4‫ودالشمال‬

346. A Patient presents at the clinic with weight loss and complains of
trouble seeing at night. The nurse also observes numerous teeth
with decay. Upon Learning that the patient has avitamin
deficiency, which of thefollowing foods would the nurse MOST
likely instruct the patient to add to diet?

a. Cheese and breads


b. Liver and rice
c. Fish and rice
d. Fruits and vegetables

• Answer : D

347. While providing discharge teaching for the parents of a child


newly diagnosed with cystic fibrosis, the nurse includes teaching
regarding the role of salt in the disease. Which of the following
statements by the patient indicates the need for further teaching?

a. Salty foods may be eatenon occasion


b. My child does not need torestrict salt intake
c. Salt is lost more rapidlyin hot weather
d. Salt replacement shouldoccur every day

• Answer : D

‫مناقشة امتحانات البرومتريك للتمريض‬

https://www.facebook.com/groups/1639727976293954 Page 163


Prometric exam collection 2017

)‫)النسخة الرابعة‬

4‫ودالشمال‬

348. A patient visiting the clinic 10 days after sinus surgery for checkup
complains of having a bad taste in the mouth. When the nurse
smells a foul odor while examining the patients mouth, the nurse
suspects the patient have an:

a. Pulmonary decompensation
b. Hemorrhage
c. Aspiration
d. Infection

• Answer : D

349. A patient is scheduled for a total hip arthroplasty. The preoperative


nurse reviews the chest and notes the following: serum potassium
level of 2.8 mEq/l, AB positive blood type, and elevated ST
Segments on the electrocardiogram (ECG). Which of the following
would be the MOST appropriate action for the nurse to do next?

a. Report abnormal diagnostic results to the surgeon


b. Review the patientconsent for the surgicalprocedure
c. Educate the patient on therisk factors and side-effects of the
surgery
d. Ensure that the patient hasa post-surgery physicaltherapy
order

• Answer : A
‫مناقشة امتحانات البرومتريك للتمريض‬

https://www.facebook.com/groups/1639727976293954 Page 164


Prometric exam collection 2017

)‫)النسخة الرابعة‬

4‫ودالشمال‬

350. Which of the following discharge planning instructions takes


PRIORITY in patient with congestive heart failure?

a. Maintaining a low cholesterol, low sodium and low


potassium diet
b. Recognizing signs andsymptoms that requireimmediate
medicalattention
c. The importance ofremaining physicallyactive
d. The importance ofdrinking plenty of fluid

• Answer : B

351. Following ocular surgery the nurse establishes care interventions to


include orienting the patients to new changes in environment and
supervising the Patients ability to feed themselves and perform
self-care activities. Which of the following nursing diagnosis do
these activities support?

a. Activity intolerance
b. Impaired environmental interpretation syndrome
c. Disturbed sensoryperception
d. Risk for autonomicdysreflexia

• Answer : B

‫مناقشة امتحانات البرومتريك للتمريض‬

https://www.facebook.com/groups/1639727976293954 Page 165


Prometric exam collection 2017

)‫)النسخة الرابعة‬

4‫ودالشمال‬

352. During the immediate postoperative period, a patient reveals an


oxygen saturation level of 91%. The nurse should:

a. Position the patient on the left side


b. Administer supplemental oxygen
c. Continue to providesupportive care
d. Lower the temperature ofthe room

• Answer : c

353. Which of the following goal take PRIORITY when recovering


from general anesthesia in post anesthesia care unit (PACU)?

a. Thermoregulation
b. Plastic skin turgor
c. Patent airway
d. Patient voids freely

• Answer : C

354. A patient is to receive heparin sodium, 5,000 U, subcutaneous on


call to the operating room.Prior to administering this medication,
the nurse should advise patient that this will help to prevent:

a. Infections
b. Atelectasis
‫مناقشة امتحانات البرومتريك للتمريض‬

https://www.facebook.com/groups/1639727976293954 Page 166


Prometric exam collection 2017

)‫)النسخة الرابعة‬

4‫ودالشمال‬

c. Thrombosis formation
d. Positioning injuries

• Answer : C

355. When administering an intramuscular injection to an infant, which


of the following sites appropriate for the nurse to use?

a. Rectus femoris
b. Deltoid
c. Dorsogluteal
d. Ventrogluteal

• Answer : D

356. A patient is admitted to the medical unit with a diagnosis of fluid


volume deficit would the nurse expect the doctor to order?

a. 0.9% Sodium chloride


b. 0.45% Sodium chloride
c. Dextran in normal saline
d. 5% Sodium chloride

• Answer : A

‫مناقشة امتحانات البرومتريك للتمريض‬

https://www.facebook.com/groups/1639727976293954 Page 167


Prometric exam collection 2017

)‫)النسخة الرابعة‬

4‫ودالشمال‬

357. The nurse is discussing the human immunodeficiency virus (HIV)


with a group of high-risk patient. The nurse should state that this
virus is found MOSTcommonly in which of the following body
fluids?

a. Blood
b. Saliva
c. Breast milk
d. Vaginal secretions

• Answer : A

358. A parent is concerned their 8-year-old child has 23kg (5lb) over
the past 2 weeks and has been urination up to 30 times per day.
The child also seems to be eating and drinking constantly. Which
test would be MOST helpful in evaluating the child’s condition?

a. Chest X-ray
b. Complete blood count
c. Body fat analysis
d. Blood glucose level

• Answer : D

359. A patient has been transferred to the medical unit following a


parathyroidectomy.Surgery was performed under general
‫مناقشة امتحانات البرومتريك للتمريض‬

https://www.facebook.com/groups/1639727976293954 Page 168


Prometric exam collection 2017

)‫)النسخة الرابعة‬

4‫ودالشمال‬

anesthesia and the patients diet my advanceas tolerated. The


patient requests a sip of apple juice. The nurse should FIRST
assessthe patient’s:

a. Skin turgor
b. Cough reflex
c. Lung sounds
d. Bowel sounds

• Answer : B

360. The nurse sustains a needle-stick injury after administrating an


intramuscularinjection to a patient.It is recommended that the
nurse be tested for humanimmunodeficiency virus (HIV):

a. Immediately with repeat testin 6 weeks


b. If the patient refuses HIVtesting
c. If the patient has symptomsof HIV infection
d. A month after takingprophylactic antiviral

• Answer : A

361. A parent brings their teenage child the pediatrician’s office. The
parent reports that the patient frequently complains of abdominal
bloating and stomach pain after eating and also has a chronic sore

‫مناقشة امتحانات البرومتريك للتمريض‬

https://www.facebook.com/groups/1639727976293954 Page 169


Prometric exam collection 2017

)‫)النسخة الرابعة‬

4‫ودالشمال‬

throat. The patient’s labs show hypokalemia.Which of the


following diagnosis should the nurse anticipate?

a. Anorexia nervosa
b. Bulimia
c. Morbid obesity
d. Impulsive behavior

• Answer: B

362. A urinalysis is best evaluated for accurate result if specimen is


analyzed within:

a. 1 hour of collection or refrigerated until analyzed


b. 1 hour of collection or left at room temperature
c. 2 hours of collection
d. 4 hours of collection

• Answer: C

363. The nurse has started intravenous fluid therapy on a child. Which
of the followingaction is appropriate?
a. Using a padded arm board only if the child is active
b. Checking the site at leastonce every two hours
c. Determining the total volumeinfused every four hours
‫مناقشة امتحانات البرومتريك للتمريض‬

https://www.facebook.com/groups/1639727976293954 Page 170


Prometric exam collection 2017

)‫)النسخة الرابعة‬

4‫ودالشمال‬

d. Using an infusion pump toprovide controlled rate ofinfusion

• Answer : D
364. During the assessment phase of a preoperative interview, the
patient reports feeling nervous. The patient conveys to the nurse
that a parent died in surgery due to malignant hyperthermia. To
whom would this information be MOST pertinent?

a. Post-anesthesia care unit(PACU) nurse


b. Scrub nurse
c. Anesthesia team
d. Charge nurse

• Answer : C

365. A child presents to the emergency department with difficulty


breathing. The child’sParents report that child has a history of
bronchial asthma and has recently had an Upper respiratory tract
infection (URI). Upon auscultation, the nurse decreased Breath
sounds in the left-lower lung field. The nurse should NEXT assess
the child’s:
a. Oral temperature
b. O2 saturation
c. Apical pulse
d. Level of comfort

‫مناقشة امتحانات البرومتريك للتمريض‬

https://www.facebook.com/groups/1639727976293954 Page 171


Prometric exam collection 2017

)‫)النسخة الرابعة‬

4‫ودالشمال‬

• Answer : B

366. A patient with diabetic retinopathy is experiencing an episode of


unresolved hemorrhage in the eye. The nurse identifies the MOST
likely procedure to benefitthis patient would be:

a. Enucleation
b. Radial keratotomy
c. Vitrectomy
d. Peripheral Iridectomy

• Answer : C
367. A patient admitted to the hospital with acute cholecystitis, is
scheduled for surgery in the morning and is NPO. At 8amthe
patient develops a fever of 102.4 F (39.1 C).medication orders
include acetaminophen 650 mg orally every four hours asneeded.
The nurse should:

a. Give the medication asordered by the physician


b. Administer the ordered dose rectally
c. Put moist cool cloths on thepatient’s forehead and axillac
d. Notify the physician andrequest other orders
‫مناقشة امتحانات البرومتريك للتمريض‬

https://www.facebook.com/groups/1639727976293954 Page 172


Prometric exam collection 2017

)‫)النسخة الرابعة‬

4‫ودالشمال‬

• Answer : A
368. A home health nurse is preparing to administer a subcutaneous
injection of heparin.When site on the abdomen, the nurse will
choose a site:

a. More than 6 inches from the umbilicus


b. More than 2 inches from the umbilicus
c. As close as possible to the umbilicus
d. As close as possible to the umbilicus

• Answer : B

369. A patient with pulmonary emboli complains of pain, dyspnea, and


a fear of dying. Which of the following interventions would MOST
likely help to reduce the patient’s anxiety level?

a. Administer oxygen as ordered


b. Administer pain medication as ordered
c. Observe closely for signs of pain and discomfort
d. Listen to the patient’s concerns

• Answer : D
‫مناقشة امتحانات البرومتريك للتمريض‬

https://www.facebook.com/groups/1639727976293954 Page 173


Prometric exam collection 2017

)‫)النسخة الرابعة‬

4‫ودالشمال‬

370. A patient with bacterial meningitis is treated with intravenous


antimicrobial agent. Which of the following BEST indicates
effectiveness of treatment?
a. Severe headache
b. Negative kernig’s sign
c. Nuchal rigidity
d. Photophobia

• Answer : B

371. While caring for a patient with potassium deficiency, the nurse
should expect that the patient may exhibit:
a. Dysrhythmias
b. Oliguria
c. Diminished deep-tendon reflexes
d. Hypertension

• Answer : A

372. A patient who underwent hand surgery requiring general


anesthesia presents to the post anesthesia care unit (PACU) after
extubation, The nurse should FIRST assess

a. Circulatory status
b. Wound status
‫مناقشة امتحانات البرومتريك للتمريض‬

https://www.facebook.com/groups/1639727976293954 Page 174


Prometric exam collection 2017

)‫)النسخة الرابعة‬

4‫ودالشمال‬

c. Respiratory status
d. Hydration status

• Answer : C

373. Prior to administration of an albuterol nebulizer, the nurse should


help the patient assume what position?

a. Sitting and leaning forward


b. Feet elevated above level of heart
c. High fowler’s
d. Standing

• Answer : C

374. A patient presents to the doctor’s office 2-weeks status post-right-


sided mastectomy. The nurse needs to measure the blood pressure.
Which would be the BEST site?

a. Above the left brachial artery


b. Right popliteal artery
c. Above the right brachial artery
d. Left popliteal artery

• Answer : A

‫مناقشة امتحانات البرومتريك للتمريض‬

https://www.facebook.com/groups/1639727976293954 Page 175


Prometric exam collection 2017

)‫)النسخة الرابعة‬

4‫ودالشمال‬

375. A child with cystic fibrosis exacerbation presents to the emergency


room. Which nursing diagnosis takes FIRST? Priority in planning
for intervention?

a. Imbalanced nutrition related to increased metabolic requirements


because of mal absorption
b. Deficient knowledge regarding prevention of cystic fibrosis
exacerbation
c. Impaired gas exchange related to airway obstruction due to
mucous
d. Interrupted family processes related to hospitalization

• Answer : C

376. A nurse is evaluating the home of patient with left-sided paralysis.


Which of the following observations would indicate that the
patient is complying with home-based safety?

a. The telephone is on a bedside table with is next to the head of the


bed
b. The bedside commode is on the left-side of the bed with the back
of the commode facing the foot of the bed
c. The walker has wheels on its back legs and has tennis balls on the
front legs
‫مناقشة امتحانات البرومتريك للتمريض‬

https://www.facebook.com/groups/1639727976293954 Page 176


Prometric exam collection 2017

)‫)النسخة الرابعة‬

4‫ودالشمال‬

d. The stairs leading from the bedroom to the living area a handrail
on the right-side of the stairway

• Answer : A

377. A patient is admitted to the hospital with a cerebrovascular


accident, accident, right hemiplegia, and expressive aphasia. With
a nursing diagnosis of impaired verbal communication, what is the
BEST term goal for this patient?

a. Learn to speak clearly within30 days


b. Communicate effectively within one week
c. Have all needs anticipated by staff daily
d. Make basic needs known daily

• Answer : D

378. A Patient has a dissection aortic aneurysm. The patient’s surgery


would be categorized as:

a. Elective
b. Urgent
c. Emergency
d. Diagnostic

• Answer : C
‫مناقشة امتحانات البرومتريك للتمريض‬

https://www.facebook.com/groups/1639727976293954 Page 177


Prometric exam collection 2017

)‫)النسخة الرابعة‬

4‫ودالشمال‬

379. A patient presents to the emergency room due to an overdose of


morphine sulfate. Which of the following should the nurse has
readily available?

a. Glucagon
b. Antibiotic
c. Acetyl cysteine (Mucomyst)
d. Naloxone (Narcan)

• Answer : D

380. A patient with iron deficiency anemia due to an insufficient iron


intake needs to learn to select better food choices. The nurse works
with this patient to establish a plan of care and provide education
on proper nutrition and good sources of iron. Besides educating the
patient on a well-balanced diet the nurse would MOST likely
teach the patient that good source of iron include:

a. Seafood, cheese, soybean oil, and chocolate


b. Animal proteins, egg yolks, dried fruits, and nuts
c. Dairy products, citrus fruits, fish liver oils, and poultry
d. Seafood, fruit, poultry, and tomatoes

• Answer : C
‫مناقشة امتحانات البرومتريك للتمريض‬

https://www.facebook.com/groups/1639727976293954 Page 178


Prometric exam collection 2017

)‫)النسخة الرابعة‬

4‫ودالشمال‬

381. A 45-year-old patient is in a lower body cast following a motor


vehicle accident. In order to minimize muscle strength loss while
in the cast, the nurse will instruct the patient in the performance of:

a. Isometric exercises
b. Passive range of motion exercises
c. Active-assistive range of motion exercises
d. Resistive range of motion exercises

• Answer : C

382. A patient is being followed in the clinic for hypertension, adult


onset diabetes, and obesity. The patient is apathetic about learning
nutritional guide lines to reach the goals of weight loss and
consumption of a healthy diet. The patient admitted to eating
“whatever is put in front of me”. Which of the following actions
would the nurse take?

a. Collaborate with the patient to set goals


b. Add a nursing diagnosis of non-compliance
c. Refer for Psychiatric screening for depression
d. Discuss nutritional interventions with the spouse

• Answer : A

‫مناقشة امتحانات البرومتريك للتمريض‬

https://www.facebook.com/groups/1639727976293954 Page 179


Prometric exam collection 2017

)‫)النسخة الرابعة‬

4‫ودالشمال‬

383. A child is admitted to the pediatric ward with fever, lethargy, joint
pain and abdominal pain for several weeks. The patient has a
history of recurrent respiratory and ear infections. Physical
findings include wide spread ecchymosis, generalized lymph
adenopathy, hepato splenomegaly, and pallor. Lab work show a
low hemoglobin level, low RBC count, low hematocrit, and low
platelets. The nurse should expect the bone marrow stain to show
a:
a. Large number of lymphoblasts and lymphocytes
b. Low number of lymphoblasts and large number of lymphocytes
c. Low number of lymphoblasts and lymphocytes
d. Large number of lymphoblasts and low number of lymphocytes

• Answer : C
384. Immediately following the birth of a full term newborn, which of
the following nursing diagnoses should take PRIORITY?

a. Ineffective airway clearance related to nasal and oral secretions


b. Ineffective thermoregulation related to environmental factors
c. Risk for imbalanced fluid volume related to weak sucking reflex
d. Risk for injury related to immature defense mechanisms

• Answer : A

385. A patient receives intravenous therapy of 1000 cc normal saline


with 20mEq potassium chloride at a rate of 75cc per hour. Upon
‫مناقشة امتحانات البرومتريك للتمريض‬

https://www.facebook.com/groups/1639727976293954 Page 180


Prometric exam collection 2017

)‫)النسخة الرابعة‬

4‫ودالشمال‬

evaluation of the site, there is no edema, the vein appears slightly


red, and the patient complains of pain. What should the nurse do?

a. Slow the rate to prevent burning from the solution and continue to
monitor
b. Discontinue the intravenous line and restart in another site
c. Monitor at least every half-hour for edema but continues the order
state
d. Notify the doctor that the patient is having an adverse reaction to
the medication

• Answer :A

386. A healthy 26-year-old patient is at 39-weeks-gestation. The patient


is not considered high risk at the time of admission to the labor and
delivery unit. Which of the following pending laboratory test
results should receive PRIORITY?

a. Red blood cell count


b. Hematocrit
c. White blood cell count
d. Blood type

• Answer : D

‫مناقشة امتحانات البرومتريك للتمريض‬

https://www.facebook.com/groups/1639727976293954 Page 181


Prometric exam collection 2017

)‫)النسخة الرابعة‬

4‫ودالشمال‬

387. A patient comes to the emergency department with extreme


dyspnea, orthopnea, anxiety and complains of feeling panicky. The
patient is coughing up white frothy sputum and is cyanotic with
profuse perspiration. Inspiratory and expiratory wheezing and
bubbling sounds are auscultated. The patient is diagnosed with
acute pulmonary edema. What should the nurse do FIRST?

a. Identify precipitating factors and underlying conditions


b. Administer morphine(Dura morph) to reduce anxiety
c. Assess oxygen saturation rate
d. Administer digoxin(Lanoxin) to decrease fluid backing up into the
lungs

• Answer : C

388. During surgery, the patient has the following intake and output:
intravenous fluid 650cc, intravenous antibiotic 50cc, I unit of
packed red blood cells (PRBC) 350cc,nasogastric output 120cc,
estimated blood loss 80cc, and urine in the Foley catheter 240cc.
What is the patient’s total output?

a. 120cc
b. 200cc
c. 240cc
d. 440cc

‫مناقشة امتحانات البرومتريك للتمريض‬

https://www.facebook.com/groups/1639727976293954 Page 182


Prometric exam collection 2017

)‫)النسخة الرابعة‬

4‫ودالشمال‬

• Answer : D

389. A 25-year-old female presents to the emergency room with


lethargy, decreased reflexes, hypoventilation, hypotension, and
fixed and dilated pupils. A family member who is accompanying
the patient has an empty bottle of diazepam (Valium) which the
label states was recently refilled. The family member also indicates
that the patient has a history of depression what intervention
should the nurse expect to administer?

a. Flumazenil or (activated charcoal)


b. A tap water enema
c. Magnesium sulphate to reduce the risk of seizure
d. Nalaxazone

Answer: A

390. An asthmatic patient presents with wheezing and coughing.


Oxygen saturation is 88% on room air. Which of the following
nursing diagnosis would take priority?
a. Imbalanced nutrition related to decreased food intake
b. Activity intolerance related to inefficient breathing.
c. Anxiety related dyspnea and concern of illness.

‫مناقشة امتحانات البرومتريك للتمريض‬

https://www.facebook.com/groups/1639727976293954 Page 183


Prometric exam collection 2017

)‫)النسخة الرابعة‬

4‫ودالشمال‬

d. Ineffective gas exchange related to bronchospasm


Answer: D

391. A child is admitted to the hospital with congenital heart disease.


Which of the following nursing diagnoses should receive Priority?
• Decreased cardiac output related to decreased myocardial
functions.
• Activity intolerance related to cachexia
• Impaired gas exchange related to altered pulmonary blood flow
• Imbalanced nutrition : less than body requirement related to
excessive energy demands

• Ans: A
392. A patient scheduled for an abdominal aneurysm repair. This is
what type of surgical intervention?
a. Diagnostic
b. Transplant
c. Curative
d. Palliative

• Ans: C
‫مناقشة امتحانات البرومتريك للتمريض‬

https://www.facebook.com/groups/1639727976293954 Page 184


Prometric exam collection 2017

)‫)النسخة الرابعة‬

4‫ودالشمال‬

393. The patient present to the hospital voicing a concern about being
exposed to HEP A (HAV) 1 week upon questioning the nurse finds
the patient purchased food from a person recently diagnosed with
HEP A . Nurse would be most correct when instruct the patient
• The incubation period is 3-5 wks
• HAV is spread by seual contact
• HAV is spread by blood contact
• The incubation period is 2-6wks

ANS –d

394. While performing a pre-operative assessment on a pt having


arthroscopy of the right knee , a nurse examine the right leg for
baseline assessment . The nurse should include all the following
EXCEPT
a. Position and length of the leg
b. Bilateral pulse
c. Bony prominence of ankles and feet
d. Rotation of patella

ANS – D

395. A patient had right knee surgery and is being transferred to the post
anesthesia care unit. which of the following information is
ESSENTIAL to discuss
a. Pre-operative weakness of the lower extremities
‫مناقشة امتحانات البرومتريك للتمريض‬

https://www.facebook.com/groups/1639727976293954 Page 185


Prometric exam collection 2017

)‫)النسخة الرابعة‬

4‫ودالشمال‬

b. Anxiety related to inherited risk factors of surgery


c. Fear related to body image disturbances
d. Allergy to aspirin based products

ANS- B

396. A patient who underwent a right knee arthroplasty 2 days ago has a
nursing diagnosis of impaired mobility. The patient refuses to get
out of bed and ambulate due to chest pain. which of the following
action would the nurse MOST LIKELY implemented
a. Medicate the patient prior to ambulation
b. Add a nursing diagnosis of non-compliance
c. Let the patient rest now and then try to ambulate later
d. Assess to determine the course of the chest pain

ANS – D

397. After total knee replacement a patient being discharged to have


after which he will ambulate with for-prong cane. When providing
patient teaching regarding giving up and down stairs with the cane,
the first step in going up stairs is to ..,
a. Place the cane and the affected extremities upon the step
b. Place the cane and the unaffected extremity upon the step
c. Step up on the affected extremity
d. Step up on the unaffected extremity

‫مناقشة امتحانات البرومتريك للتمريض‬

https://www.facebook.com/groups/1639727976293954 Page 186


Prometric exam collection 2017

)‫)النسخة الرابعة‬

4‫ودالشمال‬

ANS – A

398. A nurse is caring a patient who had right mastectomy 2 days ago.
Which of the following is the appropriate nursing goal for this type
of surgery
a. Acceptance of altered body image
b. Avoid large crowd
c. Limit right arm movement
d. Perform range of motion for left arm

ANS – A

399. Which instruction take priority in reducing anxiety related to


Surgical procedure and post-operative exercise

a. Risk of infection after surgery


b. Advanced directives and what it means
c. Pre-operative laboratory result and what to expect on it

ANS-b
400. The nurse is assigned a patient who had surgery under GA. The
patient respiratory rate is 4/mnt and the O2 saturation on 3mL/mnt
of O2 via nasal cannula is 84%. The nurse is awaiting the result of
an ABG and anticipate that which of the following elevated ?
a. Arterial O2 saturation (SaO2)
b. HYDROGEN ion concentration (PH)
‫مناقشة امتحانات البرومتريك للتمريض‬

https://www.facebook.com/groups/1639727976293954 Page 187


Prometric exam collection 2017

)‫)النسخة الرابعة‬

4‫ودالشمال‬

c. Partial pressure of arterial CO2 (PaCO2)


Ans – c

401. The traction and urinary catheter have been discontinued for a
patient who was immobilized in traction for 6 weeks . The pt is
now having a problem with urinary incontinence .which of the
following interventions would the nurse most likely implement?
a. Behavioral training
b. Bladder training
c. Scheduled toileting
d. Prompted voiding

ANS - B

402. A nurse is assigned to care for a patient with a diagnosis of


thrombotic stroke. The nurse knows that this type of stroke is most
likely caused by:
a. Blockage of large vessels as a result of
atherosclerosis
b. Emboli produced from valvular heart disease
c. Decreased cerebral blood flow due to circulatory
failure
d. A temporary disruption in oxygenation of the
brain

‫مناقشة امتحانات البرومتريك للتمريض‬

https://www.facebook.com/groups/1639727976293954 Page 188


Prometric exam collection 2017

)‫)النسخة الرابعة‬

4‫ودالشمال‬

Ans: A
403. The nurse administered a prescribed intramuscular medication to a
patient during a home health visit. How should the nurse dispose of
the used needle and syringe?
a. Recap the needle, then place the needle and
syringe into a waterproof container until safe
disposal can be made
b. Bend the needle back towards the barrel of the
syringe before putting the needle and syringe in a
metal trash container
c. Wrap the needle and syringe in disposable paper
before putting the needle and syringe into the
dirty section of the nurse’s equipment bag
d. Put the needle and syringe directly into a
puncture-resistant plastic container that has a lid
Ans: B
404. What is the most common characteristic of a stage IV pressure
ulcer?
a. Pink skin
b. Presence of sinus tracts
c. Exposure of bone
d. Infection

‫مناقشة امتحانات البرومتريك للتمريض‬

https://www.facebook.com/groups/1639727976293954 Page 189


Prometric exam collection 2017

)‫)النسخة الرابعة‬

4‫ودالشمال‬

Ans: C

405. While visiting a patient with a new colostomy, the home care nurse
observes that the skin around the stoma site is red. Which
intervention should the nurse do next?
a. Apply pectin, gelatin or synthetic skin barrier around
the stoma
b. Apply triple antibiotic to the raw skin and leave it open
to the air
c. Instruct to empty the pouch as soon as stool is present
d. Instruct to remove the bag and skin barrier after each
stool
Ans: A
406. A nurse educates a patient about the use of incentive spirometry to
prevent atlectasis after a surgery. The nurse is performing what
step of the nursing process?
a. Diagnosis
b. Assessment
c. Implementation
d. Evaluation
Ans: C
‫مناقشة امتحانات البرومتريك للتمريض‬

https://www.facebook.com/groups/1639727976293954 Page 190


Prometric exam collection 2017

)‫)النسخة الرابعة‬

4‫ودالشمال‬

407. A nurse evaluates a patient for signs of rebleeding from ruptured


intracranial aneurysm that required surgical ligation. The highest
risk for aneurysm rebleed is within:
a. 6 hours
b. 24 hours
c. 48 hours
d. 72 hours
Ans: A
408. When discussing dietary choice with a patient who is receiving
heparin therapy, the nurse should state that which of the following
foods affect the clotting time?
a. High protein foods
b. Soy- based foods
c. Foods high in vitamin K
d. Foods containing goat’s milk
Ans: C
409. A patient admitted to the hospital for pneumonia finishes a course
of levofloxacin, lungs are clear and the patient is no longer
coughing. Which of the following post-discharge laboratory results
best indicates desired outcome?
a. Normal white blood cell count
b. Normal hematocrit count
‫مناقشة امتحانات البرومتريك للتمريض‬

https://www.facebook.com/groups/1639727976293954 Page 191


Prometric exam collection 2017

)‫)النسخة الرابعة‬

4‫ودالشمال‬

c. Normal platelet level


d. Normal potassium level
Ans: A
410. A home health care nurse visits a patient diagnosed with
rheumatoid arthritis. The nurse gathers information about the pain
level after the use of prescribed pain medication to check on the
effectiveness of the intervention. This phase of nursing process is
called:
a. Assessment
b. Diagnosis
c. Implementation
d. Evaluation
Ans: D
411. A plan of care for a child with cerebral palsy should include all the
following except:
a. Physical therapy
b. Play
c. Feeding
d. Bowel and bladder training
Ans: D

‫مناقشة امتحانات البرومتريك للتمريض‬

https://www.facebook.com/groups/1639727976293954 Page 192


Prometric exam collection 2017

)‫)النسخة الرابعة‬

4‫ودالشمال‬

412. A patient is admitted to the hospital with klebsiellapneumoniae.


During the initial intravenous dose of Amikin (amikacin sulfate),
the patient develops severe respiratory distress. This is most likely:
a. A side effect
b. An indication of drug tolerance
c. A drug allergy
d. A toxic effect
Ans: A
413. A patient is diagnosed with peptic ulcer. What would be the long
term goal for this patient?
a. Patient remains free of signs and symptoms of
gastrointestinal bleeding
b. Patient maintains lifestyle alterations to prevent
recurrence of ulcer
c. Patient expresses decreased pain level
d. Patient performs activities of daily living without
difficulty
Ans: B
414. A patient visits the clinic for the first time. In order to perform an
accurate and complete assessment, which of the following would
be the nurse first step?
a. Obtain a temperature, pulse and respiration
‫مناقشة امتحانات البرومتريك للتمريض‬

https://www.facebook.com/groups/1639727976293954 Page 193


Prometric exam collection 2017

)‫)النسخة الرابعة‬

4‫ودالشمال‬

b. Obtain a complete history


c. Obtain a blood pressure
d. Perform a review of systems
Ans:B
415. The nurse is assigned to care for a patient who has recently been
diagnosed with Crohn’s disease. The initial treatment is usually:
a. Dietary changes
b. Reversible colostomy
c. Permanent colostomy
d. Watchful waiting
Ans: A
416. A patient comes to the medical office with complaints of some
urinary incontinence. The nurse discovers the incontinence occurs
because of an inability to delay voiding long enough to reach a
toilet after the patient feels a sensation of bladder fullness. This
type of incontinence is:
a. Stress
b. Urge
c. Overflow
d. Functional

‫مناقشة امتحانات البرومتريك للتمريض‬

https://www.facebook.com/groups/1639727976293954 Page 194


Prometric exam collection 2017

)‫)النسخة الرابعة‬

4‫ودالشمال‬

Ans: C
417. When caring for child with spina bifida, the nurse knows that the
child has an increased risk of allergy to:
a. Peanuts
b. Strawberries
c. Eggs
d. Latex
Ans: D
418. When planning a class on pregnancy, the nurse should include
symptoms of pregnancy that must be reported immediately, such
as:
a. Leg cramps
b. Vision disturbance
c. Swelling of the legs
d. Constipation
Ans: B
419. Which of the following reacts to viruses and bacteria by increasing
in number?
a. Antigens
b. Antibodies
c. Rh factors
‫مناقشة امتحانات البرومتريك للتمريض‬

https://www.facebook.com/groups/1639727976293954 Page 195


Prometric exam collection 2017

)‫)النسخة الرابعة‬

4‫ودالشمال‬

d. Platelets
Ans: B
420. A nurse is assessing a child with cystic fibrosis. After thoroughly
assessing respiratory status, the nurse should assess which of the
following?
a. Level of pain
b. Skin turgor
c. Genitourinary status, clarity of urine
d. Nutritional status, characteristics of stool
Ans: A
421. The nurse is preparing to administer 100 ml potassium chloride
solution. The prescriptions indicate that this should be infuse for 2
hours. The nurse should administer how many ml per hour?
a. 10
b. 25
c. 50
d. 100
Ans: C
422. A nurse is caring for a patient who is 6-hours post-left lobectomy.
On assessment the nurse observes that the patient has become very
restless and the nail beds are blue. The vital signs reveal

‫مناقشة امتحانات البرومتريك للتمريض‬

https://www.facebook.com/groups/1639727976293954 Page 196


Prometric exam collection 2017

)‫)النسخة الرابعة‬

4‫ودالشمال‬

tachycardia, tachypnoea and the blood pressure is rising. Which of


the following complications is most likely?
a. Pneumonia
b. Hypoxia
c. Postoperative bleeding
d. Bronchopleural fistula
Ans: B

423. A patient with heart failure has the following vital signs: blood
pressure level, 136/84 mmHg, heart rate 48, temperature 37.1 C
(98.8 F); and respiration rate 20 per minute. Which of these vital
signs should be reported to the physician prior to administering the
next dose of digoxin?
a. Blood pressure
b. Pulse
c. Temperature
d. Respiration rate
Ans: B
424. The nurse is caring for a patient two hours after a pacemaker
placement. The patient suddenly starts complaining of chest pain.
The nurse observes dyspnoea, cyanosis and absent breath sounds
on the right side. The nurse should anticipate what complications?
‫مناقشة امتحانات البرومتريك للتمريض‬

https://www.facebook.com/groups/1639727976293954 Page 197


Prometric exam collection 2017

)‫)النسخة الرابعة‬

4‫ودالشمال‬

a. Hemothorax
b. Perforation of the heart
c. Pneumothorax
d. Hemorrhage
Ans: C
425. A community health nurse is instructing a neighborhood class
about botulism. The nurse teaches the group that the most likely
mode of infection would be by:
a. Direct contact with contaminated soil
b. Direct contact with respiratory secretions
c. Sexual intercourse
d. Ingestion of contaminated food
Ans: d

426. A 32 year old female comes in for evaluation 14 days after an


uncomplicated caesarean section . The patient is very anxious and
complaining of sharp stabling pain in her chest .The patient has
dyspnea , tachypnea , and hypoxemia .Which of the following
postoperative complications is likely?
a. Pulmonary embolism
b. Atelectasis

‫مناقشة امتحانات البرومتريك للتمريض‬

https://www.facebook.com/groups/1639727976293954 Page 198


Prometric exam collection 2017

)‫)النسخة الرابعة‬

4‫ودالشمال‬

c. Pneumonia
d. Aspiration
• Answer A

427. A home care nurse reviews the laboratory results for a postpartum
patient who had a caesarean section . Which of the following
indicates possible wound infection ?
a. Increased WBC
b. Decreased hematocrit level
c. Increased hemoglobin
d. Decreased platelet

• Answer A
428. Three days ago a patient underwent an invasive surgery with an
open wound. The patient is febrile with drop in blood pressure.
Laboratory test results shows elevated WBC count. This could be
possible presentation of :
• a. Sepsis
• Atelectasis
• Internal hemorrhaging
• Excess fluid volume
‫مناقشة امتحانات البرومتريك للتمريض‬

https://www.facebook.com/groups/1639727976293954 Page 199


Prometric exam collection 2017

)‫)النسخة الرابعة‬

4‫ودالشمال‬

• Answer A
429. A conscious victim of motor vehicle accident arrives at the
emergency department. The patient gasping of air , is extremely
anxious , and has a deviated trachea . What diagnosis should the
nurse anticipate?
a. Pleural effusion
• Tension pneumothorax
• Pneumothorax
• Hemothorax

• Answer B
430. A patient is brought to emergency room with a severe head injury.
A craniotomy is performed to evacuate a blood clot. Which of the
following is a desired expected outcome 24 hours postoperatively?
• Gag reflux present
• Cerebral perfusion pressure , 68mm Hg
• Intracranial pressure , 21 mm Hg
• Decreased lacrimation

• Answer C

‫مناقشة امتحانات البرومتريك للتمريض‬

https://www.facebook.com/groups/1639727976293954 Page 200


Prometric exam collection 2017

)‫)النسخة الرابعة‬

4‫ودالشمال‬

431. A nurse is assigned to a patient who is scheduled for an above the


knee amputation of the left leg . During the preoperative procedure
the nurse should ask the patient to :
a. Write YES on the leg
b. Write OTHER ONE on the right leg
c. Draw an arrow on the left knee pointing upward
d. Draw an arrow on the left knee pointing downward
• Answer C
432. A patient who is 18-hour postoperative after an above-the knee
amputation complaints of feeling like something is crawling under
the dressing as well as increased pressure of the dressing. The
nurse suspects hemorrhage. The patient’s vital signs remain within
the normal range. What should the nurse do FIRST?
• Call the physician
• Place ice around the dressing
• Encourage patient to discuss fears
• Lower the temperature of the room

• Answer A
433. A patient is admitted for pain management due to lung cancer with
metastasis of the bone. With a nursing diagnosis of alteration in

‫مناقشة امتحانات البرومتريك للتمريض‬

https://www.facebook.com/groups/1639727976293954 Page 201


Prometric exam collection 2017

)‫)النسخة الرابعة‬

4‫ودالشمال‬

comfort , the nurse would anticipate the best shot-term goal for this
patient would be to :
• Not complain of pain
• Appear comfortable and sleep well
• Verbalize that pain is relived
• Verbalize that pain is tolerated

• Answer A

434. A nurse is assessing a patient who just arrived in the emergency


department (ED) after a motor vehicle collision. The patient has a
strong smell of alcohol on the breath, is restless, and has a bluish
discoloration on the abdomen by the umbilicus. The patients vital
signs are temperature 37.20C (98.90F), heart rate 120/min,
respiration rate 24/min, and blood pressure level 100/62 mmHg.
While other members of the team are evaluating the patient , the
nurse should obtain :
• A pair of elastic support stockings
• A chest tube insertion tray
• Supplies for peritoneal lavage
• A vial of hydralazine

‫مناقشة امتحانات البرومتريك للتمريض‬

https://www.facebook.com/groups/1639727976293954 Page 202


Prometric exam collection 2017

)‫)النسخة الرابعة‬

4‫ودالشمال‬

• Answer D

435. While caring for a terminally ill preschool-aged child whose death
is eminent , the child asks the nurse “ Am I going to die”? The best
nursing response is :
• I’m not sure what is wrong with you, but I hope not
• Don’t worry, when you die, you will be the angels
• We all die someday , but you are not going to die today or
tomorrow
• I can’t talk to you about that , you will have to ask your doctor

• Answer A
436. A patient with chronic obstructive pulmonary disease complains of
a frequent cough, bilateral wheezing is auscultated in the lung
fields. The nurse administers albuterol nebulizer treatment, as
ordered and educates the patient on way to decrease exacerbation.
Which of the following actions indicate that the patient
understands the instruction?
• The patient reduces number of cigarettes smoked per day
• The patient requested a pneumococcal vaccination
• The patient increases sodium and potassium intake
• The patent exercises whenever experiencing shortness of breath
‫مناقشة امتحانات البرومتريك للتمريض‬

https://www.facebook.com/groups/1639727976293954 Page 203


Prometric exam collection 2017

)‫)النسخة الرابعة‬

4‫ودالشمال‬

• Answer A

437. A nurse administers albuterol nebulizer to a child with asthma


exacerbation. The nurse measures pulse oximetry and auscultates
the lungs to determine whether the goal of clear respiratory status
has been met. The step of nursing is called :
a. Assessment
• Diagnosis
• Implementation
• Evaluation

• Answer D
438. The home care nurse observe that the asthmatic patient has a cough
wheezing . The nurse administers an albuterol (Proventil) nebulizer
treatment as ordered. Which type of implementation is this?
a. Discharge planning
• Instruct
• Monitoring and surveillance
• Therapeutic interventions

‫مناقشة امتحانات البرومتريك للتمريض‬

https://www.facebook.com/groups/1639727976293954 Page 204


Prometric exam collection 2017

)‫)النسخة الرابعة‬

4‫ودالشمال‬

• Answer D
439. A child with asthma has an order for albuterol . Prior to
administration of medication the nurse must:
a. Pre-oxygenate the patient
• Assess the patient’s heart rate
• Obtain venous access
• Feed the patient a snack
• Answer B

440. To reduce the risk of treatment methicillin resistant staphylococcus


aureus from an infectious wound which of the following standard
precautions should be implemented
a. Airborne
b. Contact
c. Droplet
d. Reverse isolation

Ans – B

441. A patient is experiencing intermittent claudication in the legs


while at rest . Which of the following should the nurse take ?
a. Vigorously massage the extremity
b. Place ice on the ankles every 20 mnts
c. Elevate the legs to heart level
‫مناقشة امتحانات البرومتريك للتمريض‬

https://www.facebook.com/groups/1639727976293954 Page 205


Prometric exam collection 2017

)‫)النسخة الرابعة‬

4‫ودالشمال‬

d. Position the legs in dependent position

ANS – C

442. The nurse is caring for a patient with chest tubes connected to
close suction .the nurse should make sure that which of the
following remains readily available at the patients bed side?
a. A sterile towel
b. Petroleum gauze
c. Normal saline solution
d. Sterile gloves

ANS—C

443. The nursing a 15 year old patient who is being admitted due to an
exacerbation of bronchial asthma. The nurse should give
PRIORITY to asking if the patient has history of?
a. Indoor allergies
b. Intubation
c. Chest trauma
d. Co sack virus

ANS – A

‫مناقشة امتحانات البرومتريك للتمريض‬

https://www.facebook.com/groups/1639727976293954 Page 206


Prometric exam collection 2017

)‫)النسخة الرابعة‬

4‫ودالشمال‬

444. A community health nurse visits a patient who had right foot
amputation. Which of the following would suggest that the patient
is meeting expected outcome for this type surgery?
a. Stays in bed
b. Verbalize constant pain
c. Avoids social gathering
d. Accepts altered body image

ANS: D
445. While reviewing stress management techniques with a patient
diagnosed with multiple sclerosis, what would the nurse identify as
most appropriate?
a. Relaxing in a warm bubble bath
b. Yoga in a cool room
c. Sunbathing
d. Cross-country running

ANS –B

446. A child comes in the clinic with several lesions to scalp .the round
lesions have dandruff like scaling with hair loss. what is the most
likely diagnosis
a. Impetigo

‫مناقشة امتحانات البرومتريك للتمريض‬

https://www.facebook.com/groups/1639727976293954 Page 207


Prometric exam collection 2017

)‫)النسخة الرابعة‬

4‫ودالشمال‬

b. Ringworm
c. Ascariasis
d. Amoebiasis

Answer: B

447. The nurse is measuring the chest tube drainage of a patient who
had open heart surgery 4 hours ago. Which of the following is the
MAXIMUM hourly amount of chest tube drainage that is expected
in this time frame?
a. 100ml
b. 200ml
c. 300ml
d. 400ml
Answer: A
448. A patient report difficulty sleeping through the night since the
death of spouse 6 months ago which of the following is an
appropriate LONG term goal?
a. Feeling well rested each morning
b. Not feeling tired each afternoon
c. Taking brief nap in the middle of the day
d. Using sleep aid on a nightly basis
Answer: A

‫مناقشة امتحانات البرومتريك للتمريض‬

https://www.facebook.com/groups/1639727976293954 Page 208


Prometric exam collection 2017

)‫)النسخة الرابعة‬

4‫ودالشمال‬

449. A patient with SLE (systemic lupus erythematous) report


decreased urinary output during the past 2-4 days and chest pain
that is aggravated by breathing and coughing. The patient vital
signs remain within the baseline normal range s1 and s2 are
present with audible friction rub. Which of the following statement
would be appropriate for the nurse to make?
a. It sounds like SLE is being well controlled
b. I need to get some nitroglycerine for your chest pain
c. There may be some inflammation surrounding your heart
d. Your symptoms may be due to a urinary tract infection
Answer: C
450. A patient has been hospitalized with a new diagnosis of crohn’s
disease. The nurse best determine the patients hydration level by
monitoring the
a. Color of urine
b. Brightness of eyes
c. Capillary refill in nail beds
d. Temperature of lower extremities
Answer: C
451. A patient who had abdominal surgery 6 days ago , has been
ambulating the halls without much difficulty. However, on day 7
postoperative the patient complains of increased pain at incisional
site and is walking hunched over the MOST likely cause of the
change is
a. Over assertion the day before
‫مناقشة امتحانات البرومتريك للتمريض‬

https://www.facebook.com/groups/1639727976293954 Page 209


Prometric exam collection 2017

)‫)النسخة الرابعة‬

4‫ودالشمال‬

b. Pulmonary edema
c. wound infection
d. deep vein thrombosis
Answer: C
452. A diabetic patient comes to the office for follow-up six weeks
undergoing below the knee amputation of the right leg for
gangrene. The nurse observes that the patient is progressing well
with the use of prosthesis and that the skin is intact. The patient
reports being generally pain free but occasionally feels severe pain
and itching of the right ankle. What should the nurse do?
a. Notify the doctor that there appears to be nerve damage of
the right leg
b. Refer to pain management specialist for long term
management
c. Refer to psychiatrist for evaluation since the patient has no
right ankle
d. Explain the phenomena of phantom pain and phantom
sensation to the patient
Answer: D
453. A 1 year old child presents at the clinic one week after
hospitalization for surgical repair of a fractured right femur. The
patient is receiving pain medications every morning and evening.
The best way to evaluate the effectiveness of the pain management
plan is;

‫مناقشة امتحانات البرومتريك للتمريض‬

https://www.facebook.com/groups/1639727976293954 Page 210


Prometric exam collection 2017

)‫)النسخة الرابعة‬

4‫ودالشمال‬

a. To ask the child in simple terms about the comfort level of


the past week
b. By direct observation of the child’s non-verbal behaviors
during the visit
c. To teach the child how to use wong/baker faces pain rating
scale
d. To interview the parent about behavior, moods, and sleep
patterns over the past week
Answer: D
454. The nurse is caring for a patient scheduled for left arm amputation
due to bone carcinoma. Adequate assessment and management of
preoperative pain will result in
a. Decreased phantom limb sensation
b. Increased range of motion after surgery
c. Decreased depression after surgery
d. Decreased likelihood of cancer recurrence
Answer: A
455. A 34 year old quadriplegia patient resides at home with his wife. In
order to prevent contractures of all extremities, the community care
nurse will instruct the patient’s wife in the performance of
a. Active range of motion exercise
b. Passive range of motion exercise
c. Active assistive range of motion exercise
d. Resistive range of motion exercise
Answer: B
‫مناقشة امتحانات البرومتريك للتمريض‬

https://www.facebook.com/groups/1639727976293954 Page 211


Prometric exam collection 2017

)‫)النسخة الرابعة‬

4‫ودالشمال‬

456. A 7 year old child is brought to the emergency room with


complaints of feeling sick for 3 weeks with sore throat, cough, and
muscle pain. Upon examination, the nurse notes a low grade fever,
shortness of breath, and a wheeze on auscultation. The child lives
with parents, 6 siblings, and grandfather in a 3 bedroom house.
Based on these findings, which of the following diagnosis MOST
likely?
a. Staphylococcal pneumonia
b. Pneumocystis carinii pneumonia
c. Bronchiolitis
d. Mycoplasma pneumonia
Answer: D
457. A patient comes to the emergency department complaining of
severe crushing substernal pain that radiates to the left arm and
jaw. The patient is diaphoretic and pale with cool clammy skin.
The patient is diagnosed with acute myocardial infarction. The
nursing diagnosis would be decreases cardiac output related t:
a. Structural factors (incompetent valves)
b. Impaired ventricular expansion
c. Impaired contractility
d. Fluid volume deficit
Answer: C
458. After a hearing restoration operation, a patient has no signs of
complications and soon recovers which of the following is an
expected outcome 5 days after the hearing restoration surgery?
‫مناقشة امتحانات البرومتريك للتمريض‬

https://www.facebook.com/groups/1639727976293954 Page 212


Prometric exam collection 2017

)‫)النسخة الرابعة‬

4‫ودالشمال‬

a. Regain full hearing


b. Minimal facial nerve paralysis
c. Minimal urinary incontinence
d. Ambulate without difficulty
Answer A
459. When teaching a patient how to use a cane after a cerebral vascular
accident (CVA), the nurse should make sure the patient:
a. Uses the cane on the unaffected side
b. Advances the cane simultaneously with affected limb
c. Holds the cane away from the body
d. Moves the cane past the toes of the affected limb
Answer: A
460. A home care nurse visits a patient diagnose with diabetes mellitus
whose current glucose level ranges from 150mg/dl to 200mg/dl.
The patient has not been able to self-administer prescribed insulin
and complains of blurred vision and an inability to read the
marking on the syringe for proper insulin dosage. Which of the
following referrals would be MOST beneficial to the patient?
a. A dietician
b. An endocrinologist
c. An ophthalmologist
d. A physical therapist
Answer: C

‫مناقشة امتحانات البرومتريك للتمريض‬

https://www.facebook.com/groups/1639727976293954 Page 213


Prometric exam collection 2017

)‫)النسخة الرابعة‬

4‫ودالشمال‬

461. Which nursing diagnosis takes priority for newly diagnosed patient
with a left-sided stroke?
a. Risk for impaired swallowing related to absent gag reflex
b. Risk for impaired skin integrity related to immobility
c. Risk for infection related to invasive line placement
d. Risk for impaired speech related to left side stroke
Answer: A
462. A nurse is taking care of a patient who underwent abdominal
surgery 3 years ago. The patient has not been breaths deeply and
refuses to get out of bed since the surgery due to pain. Also the
patient complains of shortness of breath and the lung sounds are
diminished upon auscultation. Vital signs are. Blood pressure level
120/70mm Hg, heart rate 22, temperature 36.4C(97.6 F), o2
saturation 89%. Which of the following condition should the nurse
suspect?
a. Sepsis
b. Atelectasis
c. Congestive heart failure
d. Emphysema
Answer: B
463. A nurse visits the home of a patient who is 1 week post-left-breast
mastectomy. Which of the following should be including in patient
education?
a. It is OK to use a straight edge razor when shaving
b. Blood pressure checks should be done in the left arm
‫مناقشة امتحانات البرومتريك للتمريض‬

https://www.facebook.com/groups/1639727976293954 Page 214


Prometric exam collection 2017

)‫)النسخة الرابعة‬

4‫ودالشمال‬

c. Cuticle should not be cut


d. Avoid insect repellent on the left arm
Answer: C
464. A patient is 24 hours post-operative after having a right total hip
arthroplasty, the patient complains of pain in the right calf rated 6
on a scale of 0 no pain10 severe pain. The nurse observes that the
right calf is warm and tender to touch, while the right foot is pale
and cool. There is edema from the toes up the knee. The nurse
recognizes that these are the classic signs of:
a. Ineffective tissue perfusion
b. Fluid overload
c. Arterial occlusion
d. Deep vein thrombosis
Answer: D
465. A patient with dementia is being treated for dehydration. The
patient is confused and has been immobile for the past month.
Currently, the patient is incontinent and unable to feed self. The
nursing care plan should include
a. Coughing and deep breathing every 30 minutes
b. Positioning and turning every 2 hours
c. Range of motion exercise to all extremities every hour
d. Ambulates at least 20 steps every shift
Answer: B

‫مناقشة امتحانات البرومتريك للتمريض‬

https://www.facebook.com/groups/1639727976293954 Page 215


Prometric exam collection 2017

)‫)النسخة الرابعة‬

4‫ودالشمال‬

466. A patient is 3 week postoperative left below the knee amputation.


Which of the following is an expected outcome for this patient?
a. Verbalize relief of incisional pain, has intense phantom
sensation
b. Participates in care plan, express concern about independence
c. Full passive range of motion, requires assistance with
transfers
d. Low grade temperature, dressing reinforced every hour
Answer: B
467. During postoperative neuromuscular assessment of a patient who
had a total knee replacement nurse assesses the peroneal nerve by
testing sensation:
a. On the bottom of the foot
b. In the space between great and second toe
c. In the anterior to the rectum
d. In the anterior portion of the calf
Answer: B
468. The nurse is caring for a patient who sustained a traumatic brain
injury 4 days ago. The patient remains in a pharmacologic induced
coma while receiving mechanical ventilation. The patient is on
NPO status and the vital signs are within the normal range. The
patients bowel sounds are absent and nasogastric tube is connected
to low, intermittent suction. The nurse should prepare to begin:
a. NG feeding
b. Rapid weaning from the ventilator
‫مناقشة امتحانات البرومتريك للتمريض‬

https://www.facebook.com/groups/1639727976293954 Page 216


Prometric exam collection 2017

)‫)النسخة الرابعة‬

4‫ودالشمال‬

c. Total parenteral nutrition


d. Chest physiotherapy
Answer: C
469. The doctor has ordered the patient to be on 1 to 3 litters of oxygen
using a nasal cannula at all times. the home care nurse notes the
oxygen is currently at 2 L/minut. the oxygen saturation( SaO2)
reading is currently 85% and the partial pressure of CO2 is within
normal limits. Based on an evaluation of this information, which of
the following actions would the nurse MOST likely perform?
a. Decrease the O2 to 1 L/minut and monitor O2 saturation
b. continue the O2 at 2 L/minut and monitor O2 saturation
c. Increase the O2 to 3 L/minut and monitor O2 saturation
d. continue to monitor O2 saturation and call the doctor for new
orders
Answer: C
470. A child is treated for bacterial meningitis with an intravenous
antimicrobial agent. Which of the following BEST indicates
effectiveness of the treatment?
a. Increased appetite
b. Temperature 37.2 C(99 F)
c. Episodes of apnoea
d. Increased intra cranial pressure
Answer: B

‫مناقشة امتحانات البرومتريك للتمريض‬

https://www.facebook.com/groups/1639727976293954 Page 217


Prometric exam collection 2017

)‫)النسخة الرابعة‬

4‫ودالشمال‬

471. A patient with gastro esophageal reflux disease (GERD) is to start


taking prescribed omeprazole (prilosec). The nurse would istruct
the patient to take the medication:
a. 30 to 60 minutes before meal
b. 90 to 120 minutes before meal
c. With apple sauce
d. With milk
Answer: A
472. A patient recently diagnosed with multiple sclerosis has been
taking the following prescribed medications: baclofen (lioresal),
diazepam (valium), Amantadine (symmetrel), and phenytoin (
dilantin). When the patient presents with complaints of fatigue, the
nurse should address the dosage and frequency of which
medication?
a. Baclofen
b. Diazepam
c. Amantadine
d. Phenytoin
Answer B
473. A 12 year old patient had a cast removed from the left leg after
wearing it for 8 weeks. The patients wants to resume sports as soon
as possible. In order to regain muscle strength lost while wearing
the cast, the nurse will instruct the patient in the performance of:
a. resistive range of motion exercise
b. passive range of motion exercise
‫مناقشة امتحانات البرومتريك للتمريض‬

https://www.facebook.com/groups/1639727976293954 Page 218


Prometric exam collection 2017

)‫)النسخة الرابعة‬

4‫ودالشمال‬

c. Active assistive range of motion exercise


d. Active range of motion exercise
Answer: B
474. During the intra operative period of surgical procedure a 39 year
old male has the following vital signs: core temperature 37
C(98.6F)( heart rate 62, blood pressure126/78 mm Hg, and an O2
saturation level of 89%. The patient has received two units of
packed cell volume (PRBs) and is intubated. Which of the vital
signs is considered out of normal range?
a. heart rate
b. O2 saturation
c. Core temperature
d. Blood pressure
Answer: B
475. A 28 year old male is recovering from a moderate concussion
following a motor vehicle accident 2 weeks ago, when he suddenly
develops an increased thirst, craving coldwater. The patient
urinates very large amount of dilute, water like urine with a
specific gravity of 1.001 to 1.005 the patient is MOST likely
develop[ing
a. Diabetic mellitus
b. Diabetic insipidus
c. Hypothyroidism
d. Thyroid storm
Answer: B
‫مناقشة امتحانات البرومتريك للتمريض‬

https://www.facebook.com/groups/1639727976293954 Page 219


Prometric exam collection 2017

)‫)النسخة الرابعة‬

4‫ودالشمال‬

476. A nurse is caring for a patient who is 6 hours post left lobectomy.
On assessment the nurse observes that the patient has become very
restless and the nail beds are blue. the vital signs reveal
tachycardia, tachypnea and blood pressure is rising. Which of the
following complication is MOST likely?
a. Pneumonia
b. Hypoxia
c. Postoperative bleeding
d. Broncho pleural fistula
Answer: B
477. A patient presents to the office for a physical assessment. The
patient is found to be healthy and fit but occasionally drinks
alcohol and has unprotected sex. What is the BEST nursing
diagnosis?
a. Health- seeking behavior
b. knowledge deficit , high risk behavior
c. Low self esteem
d. Altered thought process
Answer: B
478. During surgery, the nurse is assigned the following duties: setting
up the sterile field, preparing sutures and ligatures assisting the
surgeon during the procedure by anticipating the instruments and
supplies that will be required and labeling tissue specimen
obtained during surgery. The nurse MOST likely performing in
what role?
‫مناقشة امتحانات البرومتريك للتمريض‬

https://www.facebook.com/groups/1639727976293954 Page 220


Prometric exam collection 2017

)‫)النسخة الرابعة‬

4‫ودالشمال‬

a. Circulating nurse
b. Scrub Nurse
c. RN first assistance
d. Nurse anesthetist
Answer: A
479. A nurse completes discharge instruction for patient who was
admitted 5 days ago with pneumonia. Which statement by the
patient would alert the nurse that more discharge teaching is
needed?
a. I need to gradually increase my activities
b. I will not need the influenza or pneumonia vaccine
c. I may experience fatigue and weakness for a prolonged time
d. I need to have another chest x-ray in 4-6 weeks
Answer: B
480. The nurse is assessing a patient recently diagnosed with acquired
immune deficiency syndrome (AIDS). Which of the following
nursing diagnosis has PRIORITY?
a. Fear of disease progression, treatment effects, isolation and
death related having aids
b. Risk for infection related immunodeficiency
c. Ineffective breathing pattern related to opportunistic infection
d. Disturbed body image related to rapid body changes from
debilitating disease
Answer: C

‫مناقشة امتحانات البرومتريك للتمريض‬

https://www.facebook.com/groups/1639727976293954 Page 221


Prometric exam collection 2017

)‫)النسخة الرابعة‬

4‫ودالشمال‬

481. A patient to have an elective surgical procedure to repair an


umbilical hernia. The patient is 68 year old, weighs 136 kg( 300lb),
and has diabetic mellitus. Which of the following approaches
would be the MOST beneficial in order to reduce the patient
surgical risk?
a. Monitor blood glucose level monthly
b. Avoid fluid overload by restricting fluid
c. Discourage any changes in routine before surgery
d. Encourage weight reduction
Answer: D
482. A nurse caring for a patient following cardiac catheterization
evaluates the patient post procedure, which of the following signs
and/or symptoms would MOST likely indicate the patient is having
a vagal reaction?
a. diaphoresis
b. Chest pain
c. Tingling in extremities
d. Hematoma formation
Answer: B
483. A home health nurse visits a patient who is newly diagnosed with
diabetes. The glucose level ranges from 120mg/dl to 150mg/dl
while current glycosylated hemoglobin (hbA1C) level is 6.9 %.
The patient is complaint with taking prescribed hypoglycemic
medications and eats 3 meals a day followed by desserts sweetened
with granulated sugar. The patient also exercises 30 minutes a day
‫مناقشة امتحانات البرومتريك للتمريض‬

https://www.facebook.com/groups/1639727976293954 Page 222


Prometric exam collection 2017

)‫)النسخة الرابعة‬

4‫ودالشمال‬

3 times a week. Which of the following educational intervention


takes PRIORITY?
a. Glucose monitoring
b. Medications
c. Dietary requirements
d. Exercise regimen
Answer: C
484. Which of the following Best describes the assessment step of the
nursing process?
a. Identifying nursing interventions as appropriate for short-
term, intermediate, and long-term goal attainment
b. Assigning priorities to the nursing diagnosis
c. Establishing goals or expected outcomes
d. Obtaining a nursing history and complete a physical
examination of the patient
Answer: D
485. A nurse is providing care to a patient with a new skin graft on left
leg. The patient is upset and the nurse notes copious red drainage
oozing around the dressing. The nurse should immediately:
a. Lift the dressing to assess the area
b. Ask if the patient is having any pain
c. Apply firm pressure for 10 to 15 minutes
d. Assess the apical pulse
Answer: C

‫مناقشة امتحانات البرومتريك للتمريض‬

https://www.facebook.com/groups/1639727976293954 Page 223


Prometric exam collection 2017

)‫)النسخة الرابعة‬

4‫ودالشمال‬

486. An elderly patient had surgery two days for an intestinal


obstruction. Vital signs at 10 am are temperature 37.5c (99.5 f),
heart rate 86, respiratory rate 16 blood pressure level 132/72 mm
Hg, pain level of 4 on a scale of 0 to 10. The abdominal dressing is
dry and intact. The nasal gastric tube to low intermittent suction.
The patient is on strict input and output every two hours. At 12.20
pm, the patient complains abdominal pain, upon assessment the
vital signs are temperature 37.5 C, heart rate 98, respiration rate
24, blood pressure level 146/ 88 mm Hg, pain level is 8 out of 10.
The patient abdomen is distended and rigid, the dressing remains
dry and intact. The nurse should first:

a.
Reposition the patient on the right side
b.
Irrigate the nasal gastric tube to check patency
c.
Medicate the patient for pain as ordered
d.
Increase the suction on his nasal gastric tube to high intermittent
suction
Answer: C
487. While preparing post operative paper work for a patient scheduled
for neurosurgery, the nurse asks about the patient’s use of
medications, the patient reports taking an aspirin tablet every day,
but has not taken it today. The patient has had nothing by mouth
since midnight of the day before, the nurse should:
a. Inform the anesthesiologist immediately
b. Tell the patient the surgery must be rescheduled

‫مناقشة امتحانات البرومتريك للتمريض‬

https://www.facebook.com/groups/1639727976293954 Page 224


Prometric exam collection 2017

)‫)النسخة الرابعة‬

4‫ودالشمال‬

c. Record the information on the form in red ink


d. Obtain blood sample and notify the attending physician
Answer: C
488. A nurse is preparing an assessment of a patient’s nutritional status.
Which of the following diagnostic test would be the best measure
of the patient’s recent nutritional status with a half- life of 2-3
days?
a. Prealbumin
b. Hemoglobin
c. Albumin
d. 24- urine creatinine
Answer: B
489. A nurse is caring for a patient who had a pneumonectomy 2 days
ago for lung cancer. Which observation would indicate that the
patient is progressing towards discharge goal?
a. Cough productive of serosanguineous fluid
b. 1+ pretibial edema
c. Nap after completing bed bath
d. Frequent premature ventricular contractions (PVC)
Answer: C
490. The nurse is caring for a patient with parkinson’s disease. Which
of the following is an expected outcome related to the nursing
diagnosis of constipation related to diminished motor function,
inactivity and medications?

‫مناقشة امتحانات البرومتريك للتمريض‬

https://www.facebook.com/groups/1639727976293954 Page 225


Prometric exam collection 2017

)‫)النسخة الرابعة‬

4‫ودالشمال‬

a. The patient will use a laxative every other day


b. The patient will have a soft bowel movement daily
c. The patient will report minimal pain with bowel movements
d. The patient will limit the intake of complex carbohydrates
Answer: B
491. The parent of a child with chronic asthma is hesitant to discipline
because the child often doesn’t feel well. The nurse should
encourage the patient to:
a. Set consistent behavior limits
b. Be more lenient during times of illness
c. Cherish the limited time the child has to live
d. Avoid upsetting the child with limit setting
Answer: A
492. In developing care plan for a hospitalized 3 year old child with
asthma, the nurse plans to talk calmly in an appropriate language
and explains all procedures. Which of the following statements by
the BEST demonstrates implementation of the approach?
a. “You can use the stethoscope to listen to your heart and your
doll’s, and then I will listen’
b. “You must not wiggle while listen to your heart. You can hold
your doll’
c. The stethoscope will feel cold on your chest. You can tell your doll
how cold it feels”
d. “ let go of your doll and place your hands on your tummy while I
use the stethoscope”
‫مناقشة امتحانات البرومتريك للتمريض‬

https://www.facebook.com/groups/1639727976293954 Page 226


Prometric exam collection 2017

)‫)النسخة الرابعة‬

4‫ودالشمال‬

Answer: A
493. A nursing process which involves the performance of the nursing
plan care is:
a. Assessment
b. Nursing diagnosis
c. Implementation
d. Evaluation
Answer: C
494. A patient who is receiving chemotherapy has a platelet count of
49,000/mm3 (normal value 150,000 to 400,000/ mm3 ). Which of
the following nursing action is necessary?
a. Minimize invasive procedure
b. Crush oral medications
c. Limit intake of vitamin K rich foods
d. Monitor the temperature every 4 hours
Answer: A
495. An elderly patient with a long history of diabetes mellitus comes in
for a routine check-up. Which of the following nursing diagnosis
would the nurse anticipate?
a. Risk for impaired skin integrity related to decreases sensation and
circulation
b. Excess fluid volume related to disease process
c. Risk for injury to decrease gastric mobility and stress response

‫مناقشة امتحانات البرومتريك للتمريض‬

https://www.facebook.com/groups/1639727976293954 Page 227


Prometric exam collection 2017

)‫)النسخة الرابعة‬

4‫ودالشمال‬

d. Deficient fluids volume related to diarrhea and loss of fluids and


electrolytes
Answer: A
496. A 3 year old child is brought to the office by the parents who have
been toilet training the child for the past 5 months, with little
success. The parent has been using rewards for the keeping the
parent clean and dry. Today the parent realizes that the child
abdomen was very firm, the appetite was poor, and there had not
been bowel movement for 6 days. With a nursing diagnosis of
alteration in bowel elimination, what is BEST goal?
a. The child will recognize the urge to defecate daily
b. The parent will use praise when the child defecates in the toilet
c. Predictable, regular bowel habits will be restored and maintained
d. Toilet training will be delayed until the child is cognitively ready
Answer: A
497. The nurse is teaching a patient about spironolactone (aldactone).
Which of the following instructions should the nurse review with
the patient?
a. Increasing intake of foods that are high in potassium
b. Taking the medication right before going to sleep
c. Avoiding seasoning that are labeled as salt substitutes
d. Scheduling the medication so that a multi vitamin is taken an hour
later
Answer: A

‫مناقشة امتحانات البرومتريك للتمريض‬

https://www.facebook.com/groups/1639727976293954 Page 228


Prometric exam collection 2017

)‫)النسخة الرابعة‬

4‫ودالشمال‬

498. Position maintained in self enema administration

ANS - SIMS

499. Position maintained post vitrectomy

ANS – PRONE

‫مناقشة امتحانات البرومتريك للتمريض‬

https://www.facebook.com/groups/1639727976293954 Page 229


Prometric exam collection 2017

)‫)النسخة الرابعة‬

4‫ودالشمال‬

500. The nurse in preparing to insert RYLE’S tube (NGT) into an


infant, the nurse knows that the length of the tube should be taken
as following:
a. From the nose down to the chin and then to the umbilicus
b. From the nose to the earlobe and then to the xiphoid process
c. From the nose to the mouth to the xiphoid process

501. A nurse is admitting a six month- old infant with pneumonia.


Which of the following interventions supports this infant’s
emotional needs?
a. Allow the parents to leave the room during painful procedures
b. Encourage parents to distract the infant from crying
c. Interview the patents to learn the infant’s comforting habits
d. Enforce strict visiting schedule and routines
502. A patient visits the clinic for a 2- week checkup after a corneal
transplantation (keratoplasty). The nurse observes the patent’s
sclera is red and the patient complains of the eye feeling irritated.
The nurse suspects the patient may have:
a. Infection
b. Hemorrhage
c. Graft rejection
d. Postoperative glaucoma

‫مناقشة امتحانات البرومتريك للتمريض‬

https://www.facebook.com/groups/1639727976293954 Page 230


Prometric exam collection 2017

)‫)النسخة الرابعة‬

4‫ودالشمال‬

503. A patient has an order for 100 milliliters (ml) of intravenous (IV)
fluid to infuse over eight hours. The available IV tubing has a drip
factor of 10 gtts/ml. Which of the following rates is correct?
a. 125 ml/hour
b. 125 drop/minute
c. 21 drops/minute
d. 21 ml/hour

504. When performing a newborn assessment, the nurse should measure


the vital signs in the following sequence:
a. Pulse, respirations, temperature
b. Temperature, pulse, respirations
c. Respirations, temperature, pulse
d. Respirations, pulse, temperature

505. A patient is scheduled for a pneumonectomy in the morning.


Which of the following diagnosis is the MOST likely indication for
this type of surgery?
a. Lung carcinoma
b. Pulmonary tuberculosis
c. Benign pulmonary nodule
d. Mediastinal shift

506. A 40 year- old woman presented with right hip pain. Palpation of
the pelvic girdle is normal. An X- ray shows bone deformities,
with osteolytic lesions and bone enlargement. The patient has not
‫مناقشة امتحانات البرومتريك للتمريض‬

https://www.facebook.com/groups/1639727976293954 Page 231


Prometric exam collection 2017

)‫)النسخة الرابعة‬

4‫ودالشمال‬

suffered any trauma and has been generally healthy. Which serum
laboratory analysis would be most useful?
a. Prothrombin time
b. Alkaline phosphatase (if this high, calcium will be low and
opposite)
c. Acid phosphatase
d. Parathyroid hormone

507. What is the first intervention for a client experiencing MI?


a. Administer morphine
b. Administer oxygen (wasn’t from the list)
c. Administer sublingual nitroglycerin
d. Obtain an ECG

508. In planning home care for an immune-compromised child, the


nurse instructs the parents to use cream or emollients to prevent or
manage dry and cracked skin. A parent will BEST demonstrate
understanding of the rationale for this be stating:
a. Creams will prevent breaks in the skin and decrease the
chance of infection
b. Pleasantly scented creams will mask other less pleasant smells
c. Micronutrients in the creams will help prevent malnutrition
d. Creams will help prevent dehydration when my child does not
drink enough

‫مناقشة امتحانات البرومتريك للتمريض‬

https://www.facebook.com/groups/1639727976293954 Page 232


Prometric exam collection 2017

)‫)النسخة الرابعة‬

4‫ودالشمال‬

509. A 50 year-old male presents to the medical office 3 weeks after


cardiac surgery with complaints of a feeling of weakness, difficulty
breathing, and joint pains. Upon examination the nurse finds a
fever and a friction rub on auscultation of the chest. The nurse
recognizes that the MOST likely surgical complication is:
a. Neuropsychological dysfunction
B. Postpericardiotomy syndrome
c. Cardiac tamponade
d. Phrenic nerve damage

510. A 3-week-old infant is hospitalized with jaundice. When


considering the fluid needs relative to body size of the infant as
compared to the fluid needs of an adult. The nurse knows this
infant requires:
a. Less fluids
b. More fluids
c. Same amount of fluids
d. Much less fluid

511. Patient with diagnosis of Dilated cardiomyopathy. [700] The


Medication order:
Glucophage 850mg po. qd Imdur 60mg. po .qd
Lasix 80mg. po. qd Zocor 40mg. po. Qd
Past medical history: Heart failure, DM type 2, Peripheral vascular
disease (pvd) & primary hyperlipidemia

‫مناقشة امتحانات البرومتريك للتمريض‬

https://www.facebook.com/groups/1639727976293954 Page 233


Prometric exam collection 2017

)‫)النسخة الرابعة‬

4‫ودالشمال‬

Based on the patient's history which of the following medication


orders should the nurse verify?
a. Glucophage
b. Zocor
c. Lasix
d. Imdur

512. The nurse is discussing the human immunodeficiency virus (HIV)


with a group of high - risk patients. The nurse should state that this
virus is found MOST commonly in which of the following body
fluids?
a. Blood
b. Saliva
c. Breast milk
d. vaginal secretions

513. The nurse assesses a patient who is 16-weeks pregnant. The patient
states that she had taken isotretinoin (Accutane) , a known
teratogen for acne during her third, fourth, and fifth week of
pregnancy According to the chart, the nurse CAN expect fetal
damage to the central nervous system as well as the:
a. Palate and eare.
b. Heart, lower limbs, and palate.
c. Limbs, eyes, and teeth.
d. Heart, eyes, and limbs.

‫مناقشة امتحانات البرومتريك للتمريض‬

https://www.facebook.com/groups/1639727976293954 Page 234


Prometric exam collection 2017

)‫)النسخة الرابعة‬

4‫ودالشمال‬

514. A community health nurse visits a patient who has suffered a


stroke. The patient’s spouse explains to the nurse that the patient
chokes while eating some times. Which of the following referral
orders would the nurse anticipate needing for this patient?
a. Speech therapist
b. Dietician
c. Physical therapist
d. Neurologist

515. A patient who is 4 days postoperative after a total hip replacement


surgery, is obese and has not been able to ambulate since the
surgery. The patient is now diaphoretic, has chills, and complains
of pain in the thigh. There is tenderness over the anteromedial
surface of the thigh. The MOST likely cause is.
a. Wound infection
b. Deep vein thrombosis (DVT)
c. Pulmonary edema
d. Dehydration

516. You educate group of people about the prevention of Cerebro-


Vascular Accident (CVA), this education about CVA prevention
consider as:
a. Primary
b. Secondary
c. Thread

‫مناقشة امتحانات البرومتريك للتمريض‬

https://www.facebook.com/groups/1639727976293954 Page 235


Prometric exam collection 2017

)‫)النسخة الرابعة‬

4‫ودالشمال‬

Explanation: Primary stroke prevention refers to the treatment of


individuals with no history of stroke.
Secondary stroke prevention refers to the treatment of individuals
who have already had a stroke or transient ischemic attack.

517. Question about Kubler theory…


e.g. As identified by Dr. Elizabeth Kubler-Ross, which stage of
dying is characterized by the transition from, "NO, not me" to
"Yes, me, but ... "
a. Anger
b. Depression
c. Acceptance
d. Bargaining

518. Kubler-Ross’s five successive stages of death and dying are:


a. Anger, bargaining, denial, depression, acceptance
b. Denial, anger, depression, bargaining, acceptance
c. Denial, anger, bargaining, depression acceptance
d. Bargaining, denial, anger, depression, acceptance

519. A client undergoes right mastectomy for carcinoma. When


teaching the client post-mastectomy exercises, it is important for
the nurse to:
a. Exercise both arms simultaneously
b. Exercise the right arm only
c. Have the client wear a sling between exercise periods
‫مناقشة امتحانات البرومتريك للتمريض‬

https://www.facebook.com/groups/1639727976293954 Page 236


Prometric exam collection 2017

)‫)النسخة الرابعة‬

4‫ودالشمال‬

d. Wait until the incision has healed

520. A patient with blood transfusion, the patient has reaction, what is
the highest priority to do as intervention?
a. Stop the I.V

521. (similar question) Nurse caring for a patient receiving a


transfusion assesses that the patient is wheezing and is
complaining of back pain. After the nurse stops the transfusion, the
nurse should:
a. discontinue the IV.
b. notify the charge nurse.
c. administer heparin.
d. raise the patient’s head.
522. (similar question) Adult patient complains of diarrhea, vomiting,
abdomen cramp and pain within the past 2 weeks. The patient
reported that the pain increases when he eats and relieves when he
passes stool. Which of the following may be the cause:
a. Appendicitis
b. Crohnâ€TMs disease
c. Ectopic pregnant
d. Cholecystitis

523. A surgery procedure that must be done within 24 hours is called:


a. Selective surgery
b. Elective surgery
‫مناقشة امتحانات البرومتريك للتمريض‬

https://www.facebook.com/groups/1639727976293954 Page 237


Prometric exam collection 2017

)‫)النسخة الرابعة‬

4‫ودالشمال‬

c. Urgent surgery
d. Emergency surgery

524. Aortic anyrism operation what type of surgical intervention:


a. Elective
b. Urgent
c. Emergency
d. Diagnostic

525. A patient is scheduled for a bowel resection. The preoperative plan


of care includes putting on antiembolism stockings prior to the
transferring the patient to the operating room. What these stockings
will do‘?
a. Promote venous return
b. Minimize joint stillness
c. Encourage sustained maximum inspiration
d. Support intestinal peristalsis

526. The nurse performs a routine assessment of newborn boy who was
born 30 minutes before. One testicle is descended and the urinary
meatus opens on the underside of the ventral shaft. Based on the
findings, which additional body part should be examined
carefully?
a. Anus
b. Buttocks
c. Umbilicus
‫مناقشة امتحانات البرومتريك للتمريض‬

https://www.facebook.com/groups/1639727976293954 Page 238


Prometric exam collection 2017

)‫)النسخة الرابعة‬

4‫ودالشمال‬

d. Groin

527. A boy with skin disease, when you recommend the boy to go back
to school?
a. When you see scaly over the skin
b. When he has temperature
c. When all symptom of skin disease are disappear
528. In the summer months, a five year old girl present with a sore
throat and a dry cough that has slowly become worse over the past
three weeks, her body temperature is 38 c, on auscultation, there is
a wheezing and shortness of breath. She lives in an overcrowded
house with three brothers, parents and grandparents in a low-
income neighborhood where she attend school. Which is the
greatest risk favtor?
a. Resident in low-income neighborhood
b. Attending School
c. Exposure to pathogens in summer season
d. living crowded condition

529. A young girl that living with her parents and her grandfather going
to bad hygienic school she affected with a virus what the possible
cause for her disease :
a. The virus transferred to her from the school
b. The summer wither is the cause
c. The virus transferred from the unclean city she lives in.
d. The crowded home.
‫مناقشة امتحانات البرومتريك للتمريض‬

https://www.facebook.com/groups/1639727976293954 Page 239


Prometric exam collection 2017

)‫)النسخة الرابعة‬

4‫ودالشمال‬

530. Instrument which is used to examine the eye structure is called:


a. Ophthalmoscope
b. Laryngoscope
c. Otoscope
d. Bronchoscope

531. To examine the ear canal of the child, this is done by:
a. Pull the ear down and back
b. Pull the ear up and back
c. Pull the ear only back
d. Do not pull the ear, direct examine the ear by otoscope

532. To instill drops in the adult patient , the ear canal is opened by
pulling the ear :-
a. up and back
b. down and back
c. up and forward
d. back and forward

533. A nurse is preparing to perform an otoscopic examination on an


adult client. The nurse does which of the following to perform this
examination?
a. Pulls the pinna up and back before inserting the speculum
b. Pulls the earlobe down and back before inserting the speculum

‫مناقشة امتحانات البرومتريك للتمريض‬

https://www.facebook.com/groups/1639727976293954 Page 240


Prometric exam collection 2017

)‫)النسخة الرابعة‬

4‫ودالشمال‬

c. Uses the smallest speculum available to decrease the discomfort


of the exam
d. Tilts the client's head forward and down before inserting the
speculum

534. Patient came to emergency department with coughing and


difficulty in breathing, you suspect patient has asthma attack, the
sound you expect to hear is:
a. Wheezing
b. Chronic
c. Crackle

535. Question about ECG, what is the most appropriate diagnosis?

a. Atrial Fibrillation
b. Atrial Flutter

536. A child came to the emergency complaining of diarrhea,


abdominal pain and vomiting. After the investigation it reveals that

‫مناقشة امتحانات البرومتريك للتمريض‬

https://www.facebook.com/groups/1639727976293954 Page 241


Prometric exam collection 2017

)‫)النسخة الرابعة‬

4‫ودالشمال‬

he eats contaminated food and got germs, what kind of germs you
suspect to find in the test result?
a. Streptococcus
b. typhoid

537. Patient came or diagnose with bradycardia, what is the appropriate


treatment?
a. Atropine

538. Patient came to the clinic with pimples over all his body, the
patient start to be alone and keep away from people arround him.
As well he starts to disappear due to how he looks?
a. Social isolation
b. anxiety
c. depression

539. A female patient pregnant 9 months, came to the emergency with


bleeding and clot discharge with blood, what is the diagnosis?
a. Placenta Pravia
b. Abrubito
c. Bleeding

540. A patient has received a unit of blood, after 1 hour the patient start
to have chills and difficult breathing, there is a high temperature,
what might this indicate?
a. Septicemia
‫مناقشة امتحانات البرومتريك للتمريض‬

https://www.facebook.com/groups/1639727976293954 Page 242


Prometric exam collection 2017

)‫)النسخة الرابعة‬

4‫ودالشمال‬

541. What of the following has propriety to check before start giving
patient blood transfusion
a. Blood group
b. Name
c. Expiry date

542. Which of the following disease consider as an epidemic diseases?


TB

543. A new infant has just born, what is the most important and first
priority to do for this infant?
Avoid heat loss

544. A child with the cast in a hand, how the nurse can assess the
circulation, (capillary refill),?
Pulse

545. A patient post-operative of tonsillectomy, after few days he came


to follow up and inform the doctor of strange taste in this throat
and ''Oder'' present< what is the best diagnosis for this patient?
Infection

546. A child complain of abdominal pain, has bloody stool and greenish
vomiting, what the appropriate nursing intervention?
Give an enema
‫مناقشة امتحانات البرومتريك للتمريض‬

https://www.facebook.com/groups/1639727976293954 Page 243


Prometric exam collection 2017

)‫)النسخة الرابعة‬

4‫ودالشمال‬

547. Which of the following is the most common yypes of leukemia


that occur in the child period?
Lympho

548. A child diagnosis with bronchitis, what is the most appropriate


instructions can be giving to the child parents when discharge
home?
Hand Washing

549. Which of the following patient with the heart diseases has fluid
volume exceed?
Right heart failure

550. A patient with post-operative of a kidney surgery, what is the most


complication that may happen for this patient?
Hemorrhage

551. Patient with Alzheimer diseases, he looks confuse and often leave
his room and went out. What i the first safety take into account for
this patient?
Raise side reel of the bed

552. A patient has allergy to fish, why this could be important data to
obtain from the patient?
Because the patient will be affected due to Iodine
‫مناقشة امتحانات البرومتريك للتمريض‬

https://www.facebook.com/groups/1639727976293954 Page 244


Prometric exam collection 2017

)‫)النسخة الرابعة‬

4‫ودالشمال‬

553. An infant he born 2 days ago, has difficulty in breathing and


swelling over all his body and around the umbilical, the body tend
to yellow color, what is the best diagnosis for this case?
a. Hepatitis

554. Patient has complain of diarrhea, whcih look like ''rice diarrhea'',
what is the most common causes for this kind of diarrhra?
a. Cholera

555. A nurse do pevention procedure for cancer patient, this consider


as?
a. primary prevention

556. A patient always feel she's full stomach, and feel lazy, what you
recommend her?
a. eat less amount in a short time

557. Which of the following consider as epdomic diseases?


a. TB

558. Scenario; patient pass stool look like clay, what may cause this
problem?
a. clay stool – hepatitis

‫مناقشة امتحانات البرومتريك للتمريض‬

https://www.facebook.com/groups/1639727976293954 Page 245


Prometric exam collection 2017

)‫)النسخة الرابعة‬

4‫ودالشمال‬

559. A pregant lady was taking food with - - calorie, how she supposed
to take a calorie after delivery?
a. 1500 K.calorie

560. When the colostomy site look is red color that indicate?
a. normal

561. Patient feels cold even in the summer season, what the blood test
you should do?
a. FSH

562. An old man with swelling in the tips of finger and has a fever, the
patient expiernce what?
a. rheumetic arthritis

563. A child with bronchitis, what you should teach to his parents?
a. Hand hygiene

564. A child has tongue reflex, what might be the reason?


a. cerebral damage

565. A child is expeirencig difficulty in breathing due to bronchitis,


what you should encourage this child to do?
a. encourage him to drink fluide

‫مناقشة امتحانات البرومتريك للتمريض‬

https://www.facebook.com/groups/1639727976293954 Page 246


Prometric exam collection 2017

)‫)النسخة الرابعة‬

4‫ودالشمال‬

566. You have a patient with increasing in Ph; 7.50, and she has a
vomiting as well, what you suspect her diagnosis based on PH?
a. metabolic acidosis

567. A child has an operation and has lapoctomy, complicated to


breathing difficulty and increase in heart rate, what you suspected?
a. hypoxia

568. A child with burns injury, what make this patient not eating?
a. acute pain

569. You have a cancer patient, what priority you should do for this
patient?
a. pain management

570. A diabetic patient schedule for operation, which of the following


medication he can take while he is NPO (nothing per oral)?
a. Glucophage

571. A patient has got insuline prescribed to him, what you suspect this
patient has?
a. thrombosis

572. A patient with CVA has plan for discharge, as a nurse what you
should educate the patient?
a. using aspirin
‫مناقشة امتحانات البرومتريك للتمريض‬

https://www.facebook.com/groups/1639727976293954 Page 247


Prometric exam collection 2017

)‫)النسخة الرابعة‬

4‫ودالشمال‬

573. On the auscultation, the physician heard the patient's heart with
crackle sound and wheezing, where could the problem is exist?
a. left ventricular

574. The pulse pressure is?


a. different between systolic and diastolic

575. A patient complains of pain the eye with high pressure on the eye
as well, what is the right surgery for this patient?
a. trabeculectomy

576. Patient who look confused, does not know his name and not orient
to time, what the diagnosis for this patient/
a. Dementia

577. A child post-operative, how you can know and assess if he comfort
or in pain?
a. Through observation, non-verbal (because he is a child)..

578. Patient has swelling on both legs, to assess and diagnosis this,
which of the following i should do?
a. auscultation
b. palpitation
c. inspection

‫مناقشة امتحانات البرومتريك للتمريض‬

https://www.facebook.com/groups/1639727976293954 Page 248


Prometric exam collection 2017

)‫)النسخة الرابعة‬

4‫ودالشمال‬

579. Patient with swalloing difficulty and has a fever, the appropriate
diagnosis is?
a, pharyngitis

580. A mother came to the clinic and says that her baby does not feed
well and lose weight as well he has a yellowish color, what you
expect the baby has?
a. problem in the liver

581. A patient with TB history, after the investigation he has the TB


positive, what you should do next?
a. x-ray

582. A nurse educates a group of people about hyper-active, and said


this how can be releaved?
a. x-ray
b. has no investigation or blood test
c. through blood

583. Patients sugar and sugar naturally what is the first step that you
need to re-evaluate?
a. Glyccaylon

584. The patient has a surgical procedure what is the thing that is
important to make sure of it?
a. consent
‫مناقشة امتحانات البرومتريك للتمريض‬

https://www.facebook.com/groups/1639727976293954 Page 249


Prometric exam collection 2017

)‫)النسخة الرابعة‬

4‫ودالشمال‬

585. After the surgery the patient was not conscious as side effect of?
a. Anesthetics

586. The patient has allergies and Dr. distract him medication allergenic
what interventions
a. Call Dr. to change medicine

587. A [child / boy] in the emergency-department, oriented, crying and


open his eyes spontaneously, the child move as responding to
pain, based on Glasgow Coma Scale, the child scores at:
a. 15
b. 13
c. 11
d. 9

588. The nurse is caring for a full-term new born who was delivered
vaginally 5 minutes ago. The infant's APGAR score was 8 at one
minute and 10 at 5 minutes. Which of the following has the highest
PRIORITY?
a. Maintaining the infant in the supine position
b. Assessing the infant's red reflex
c. Preventing heat loss from the infant- check !
d. Administering humidified oxygen to the infant

‫مناقشة امتحانات البرومتريك للتمريض‬

https://www.facebook.com/groups/1639727976293954 Page 250


Prometric exam collection 2017

)‫)النسخة الرابعة‬

4‫ودالشمال‬

589. A child with deformity (broken) nose, the child went to the school
and his friends find this funny, the child was upset and went to the
nurse in the school and told him, he will stop coming to school, the
nurse toke a paper and draw the child face and nose and tell him
that ‘he will look like them after the procedure’. In which step the
nurse perform:
a. Self-confidence
b. Self-deception

590. A child with burn injury, the burn covers 80 % of the child body,
what is the appropriate diagnosis:
a. Liquid deficiency
b. Ineffective airways clearance

591. A boy has done tonsillectomy surgery, 2 hours later, the child
complain of pain 7 from 10, what is the appropriate diagnosis,
A. Acute pain
b. Swallowing difficulty

592. To minimize a toddler from scratching and picking at healing skin


graft, the nurse should utilize:
a. Mild sedatives
b. Hand mittens
c. Punishment for picking
d. Distractions

‫مناقشة امتحانات البرومتريك للتمريض‬

https://www.facebook.com/groups/1639727976293954 Page 251


Prometric exam collection 2017

)‫)النسخة الرابعة‬

4‫ودالشمال‬

593. A newborn in 38 weeks, the infant was cyanotic, what is the best
position for him:
a. Supine
b. Prone
c. Lateral

594. A nurse is administering IM injection to an infant, the nurse should


use any area:
a. Left glottal
b. Right glottal
c. Rectus femoris

595. A 4 years old girl, was playing outside, she came to her mom
crying and holding her right upper arm, she went to the hospital
with swelling over the upper arm, pain and itching, the appropriate
management is:
a. Maintain patent airway
b. Administer s/c epinephrine
c. Prepare for intubation

596. A woman she is on the18 week gestation her physicaian will insert
a fine needle in her abdomen for anlaysis the nurse is assistant in
this procedure as nurse what is the color of liquid you expect to
come out:
a. White
b. Yellow
‫مناقشة امتحانات البرومتريك للتمريض‬

https://www.facebook.com/groups/1639727976293954 Page 252


Prometric exam collection 2017

)‫)النسخة الرابعة‬

4‫ودالشمال‬

c. Browen
d. Green browen

597. Patient with fecal ileostomy, in the lower left part of abdomen, the
stool form will be:
a. Mushy
b. Solid
c. Watery

598. A child / boy in the emergency-department, oriented, crying and


open his eyes spontaneously, the child move as responding to pain,
based on Glasgow Coma Scale, the child scores at:
a. 15
b. 13
c. 11
d. 9

599. A patient schedule for pneumonectomy, what is the name of this


surgery, (what is the appropriate surgery) for this patient;
a. Lung carcinoma

600. A patient got high dose of Morphine, what is the antidote of


Morphine:
a. Nalaxone

601. A hand photo with opened fingers is this:


‫مناقشة امتحانات البرومتريك للتمريض‬

https://www.facebook.com/groups/1639727976293954 Page 253


Prometric exam collection 2017

)‫)النسخة الرابعة‬

4‫ودالشمال‬

a. Abduction

602. Patient with pancreatectomy, what is the most cause of the surgery
(this surgery to what may lead):
a. Diabetes mellitus

603. Patient came to emergency with lacenation in the left arm, what the
first intervention for this patient:
a. Elevate the right arm and put ice
b. Give analgesic in the wound
c. Do pressure on the wound

604. To minimize a toddler from scratching and picking at healing skin


graft, the nurse should utilize:
a. Mild sedatives
b. Hand mittens
c. Punishment for picking
d. Distractions

605. The nurse administered a dose of morphine sulfate, as prescribed


to a patient who is in the post-anesthesia care unit (PACU). The
patient appears to be resting comfortably; the respiratory rate is 8
and the O2 saturation on 2L of oxygen via nasal cannula is 86%.
The nurse should IMMEDIETLY administer.
a. Flumazenil (Romazicon)
b. Midazolam (Versed)
‫مناقشة امتحانات البرومتريك للتمريض‬

https://www.facebook.com/groups/1639727976293954 Page 254


Prometric exam collection 2017

)‫)النسخة الرابعة‬

4‫ودالشمال‬

c. Naloxone (Narcan)
d. Ondansetron (Zofran)

606. The nurse is caring for a full-term new born who was delivered
vaginally 5 minutes ago. The infant's APGAR score was 8 at one
minute and 10 at 5 minutes. Which of the following has the highest
PRIORITY?
a. Maintaining the infant in the supine position
b. Assessing the infant's red reflex
c. Preventing heat loss from the infant
d. Administering humidified oxygen to the infant

607. A 23 year- old male comes to the Emergency Department in a


sickle cell crisis. He reports that his pain level is a 10/10 in all
extremities. During the assessment, he cannot lie still because of
the pain. There is no cyanosis or clubbing in the extremities and all
examination findings are normal. The vital signs recorded were:
Blood pressure 132/82 mmHg Heart rate 110/min Respiratory rate
18/mm Temperature 38.4°C Oxygen Saturation 94 % an room air
Which nursing diagnosis is first priority?
a. Acute pain
b. Fluid volume deficit
c. Ineffective tissue perfusion
d. Ineffective airway clearance

‫مناقشة امتحانات البرومتريك للتمريض‬

https://www.facebook.com/groups/1639727976293954 Page 255


Prometric exam collection 2017

)‫)النسخة الرابعة‬

4‫ودالشمال‬

608. A newborn was delivered pre-term weighing 2700 grams with.


Apgar scores of 4 and 6, respectively. When the mother had
presented to the Obstetrical Triage Unit, she was already 7
centimeters dilated and fully effaced. Her due date was unknown
as she had no parental care. The infant showed signs of fetal
distress and was finally delivered by Cesarean section. At birth a
large, thin, membranous sac was protruding from the umbilical
base. What is the priority nursing intervention at birth?
a. Maintain cardio respiratory stability
b. Protect the herniated viscera
c. Manage fluid intake and output
d. Establish vascular access

609. An 82 year-old woman with Alzheimer's disease had moved into a


long-term care facility two weeks previously. Since then, the staff
has found her wondering in the hallways in middle of the night.
When approached, she is confused and frustrated, often forgetting
where she is. Which intervention would most likely decrease the
patient's confusion?
a. Administer a sleeping sedative
b. Provide full-time nursing care
c. Place a nightlight in the room
d. Provide a large meal before bed

610. A 16 year-old boy is in the Post-Operative Care Unit two hours


after a tonsillectomy. He is alert and oriented but complains of
‫مناقشة امتحانات البرومتريك للتمريض‬

https://www.facebook.com/groups/1639727976293954 Page 256


Prometric exam collection 2017

)‫)النسخة الرابعة‬

4‫ودالشمال‬

severe throat pain and difficulty swallowing. He rates the pain at a


level 7, on a scale of 1-10. The urine output from the folly catheter
is 45 ml over the past two hours.
Blood pressure 130/74 mmHg Heart rate 64/min Respiratory rate
18/min
Oxygen saturation 98 % on room air, Which clinical finding is
most important to report to the doctor?
a. Oxygen saturation
b. Difficulty swallowing
c. Urinary output
d. Pain level

611. According to information provided in the accompanying graphic.


A descending colon would be expected to produce
a. Fluid feces
b. Mushy faces
c. Semi-solid faces
d. Solid faces

612. A 45 year-old patient has had difficulty sleeping and has lost ten
kilograms despite having a large appetite on examination there is a
palpable thyroid gland.
Blood pressure 108/58 mmHg Heart rate 116/min Respiratory rate
22/min Body temperature 38.0 c oral Height 164 Weight 50
kilograms
Which additional symptom is most likely?
‫مناقشة امتحانات البرومتريك للتمريض‬

https://www.facebook.com/groups/1639727976293954 Page 257


Prometric exam collection 2017

)‫)النسخة الرابعة‬

4‫ودالشمال‬

a. Heart palpitations.
b. Depression.
c. Anorexia.
d. Paresthesia.
613. Dyspnea is defined as:
a) Pallor

b) Absence of breathing

c) Cyanosis

d) Difficult respiration

614. Mrs Ahmed age 53, her pulse rate is found to be 52 per minute.
Her heart rate could be described as:
a) Tachypnea

b) Tachycardia

c) Bradypnea

d) Bradycardia

615. When you assess the respiratory rate for the patient, you should do
all of the following EXCEPT:
a) Instruct the patient to breath in and out from his mouth.

b) Count each inspiration followed by expiration as one breath.


‫مناقشة امتحانات البرومتريك للتمريض‬

https://www.facebook.com/groups/1639727976293954 Page 258


Prometric exam collection 2017

)‫)النسخة الرابعة‬

4‫ودالشمال‬

c) Make sure that the patient is not aware that you are counting his
respiratory rate.

d) Count the respiratory rate for 30 seconds.

616. To examine the ear canal of the child, this is done by:
a) Pull the ear down and back

b) Pull the ear up and back

c) Pull the ear only back

d) Do not pull the ear, direct examine the ear by otoscope

617. The advantage of use head to toe approach when you assess the
patient:
a) It increase the number of position changes

b) It helps to prevent overlooking some aspect of data collection

c) It takes more time

d) It is difficult to detect the disease or problem

618. Discharge planning for hospitalized patient begins:


a) When the treating Dr. gives discharge order

‫مناقشة امتحانات البرومتريك للتمريض‬

https://www.facebook.com/groups/1639727976293954 Page 259


Prometric exam collection 2017

)‫)النسخة الرابعة‬

4‫ودالشمال‬

b) When all of the specific needs of the patient have been


identified

c) When the patient begins to ask about his discharge plans

d) From the first day of admission

619. Abnormal skin color which indicates yellowish color is called:


a) Erythema

b) Ecchymosis

c) Jaundice

d) Pallor

620. Which of the following is considered as subjective data?


a) Anxiety

b) Skin color

c) Height

d) Temperature

621. The physician has ordered an indwelling urinary catheter inserted


in a hospitalized patient, the nurse is aware that:
a) The procedure requires surgical asepsis
‫مناقشة امتحانات البرومتريك للتمريض‬

https://www.facebook.com/groups/1639727976293954 Page 260


Prometric exam collection 2017

)‫)النسخة الرابعة‬

4‫ودالشمال‬

b) Lubricant not needed for catheter insertion

c) Smaller catheters are used for male catheter

d) Normally a clean technique is required for catheter insertion

622. Diarrhea is best described by its:


a) Amount

b) Consistency

c) Frequency

d) Odor

623. Medication is instilled between the skin & the muscle and used to
administer Heparin.
a) Intravenous

b) Intramuscular

c) Intradermal

d) Subcutaneous

624. The angle of the syringe and needle for intramuscular injections is:
a) 90 degrees

‫مناقشة امتحانات البرومتريك للتمريض‬

https://www.facebook.com/groups/1639727976293954 Page 261


Prometric exam collection 2017

)‫)النسخة الرابعة‬

4‫ودالشمال‬

b) 45 degrees

c) 15 degrees

d) 10 degrees

625. Is the term used to administered undiluted medication quickly into


a vein:
a) Bolus

b) Secondary infusion

c) Intermittent

d) Continuous

626. A primary concern when giving heparin subcutaneously to prevent


bleeding is:
a) Don’t make massage on the injection site

b) To make massage on the injection site

c) Use the smallest gauge needle that is appropriate

d) Use Z technique

627. To ensure that medications are prepared and administered


correctly, the nurse should:

‫مناقشة امتحانات البرومتريك للتمريض‬

https://www.facebook.com/groups/1639727976293954 Page 262


Prometric exam collection 2017

)‫)النسخة الرابعة‬

4‫ودالشمال‬

a) Give the medication without question

b) Use the patient's rights

c) Give the medication only when requested

d) Use the FIVE rights

628. The doctor order is 300 cc of normal saline solution, to be finished


within 4 hours, how many drop/min you will regulate this IV (drop
factor is 20 drop/minute).
a) 10 drop/min

b) 15 drop/min

c) 25 drop/min

d) 35 drop/min

629. All of the following is used Central Venous Catheter, EXCEPT:


a) Clients require long term IV medication

b) IV medications are irritating to peripheral veins

c) Clients require short term IV medication

d) Difficult to insert peripheral catheter

‫مناقشة امتحانات البرومتريك للتمريض‬

https://www.facebook.com/groups/1639727976293954 Page 263


Prometric exam collection 2017

)‫)النسخة الرابعة‬

4‫ودالشمال‬

630. A pre-operative check list form that should be completed before


surgery, it should be including which of the following?
a) The surgical consent form

b) All laboratory test

c) Vital Signs

d) All of above

631. The process of removing poisonous substance through gastric


intubation is called:
a) Gastric Lavage

b) Gastric Gavage

c) Gastric Decompression

d) Gastric Tamponade

632. Of the following, which are the earliest signs of excessive


pressure:
a) Pale appearance of the skin

b) Reddened appearance of the skin

c) Ulcer formation on the skin

‫مناقشة امتحانات البرومتريك للتمريض‬

https://www.facebook.com/groups/1639727976293954 Page 264


Prometric exam collection 2017

)‫)النسخة الرابعة‬

4‫ودالشمال‬

d) Dark or cyanotic color to the skin

633. To prevent the formation of thrombi in the postoperative patient,


the nurse should
a) Teach foot and leg exercises

b) Have the patient lie still

c) Place pillows under the knee

d) Lie in lateral position

634. Which of the following is used to determine the activity of the


brain:
a) Electrocardiography

b) Electromyography

c) Electroencephalography

d) Echocardiography

635. An infection that the patient acquires in the hospital is called:


a) A local infection

b) An endogenous infection

c) A nosocomial infection

‫مناقشة امتحانات البرومتريك للتمريض‬

https://www.facebook.com/groups/1639727976293954 Page 265


Prometric exam collection 2017

)‫)النسخة الرابعة‬

4‫ودالشمال‬

d) A secondary infection

636. The nursing activity most likely to prevent the clogging of a


nasogastric feeding tube is:
a) Attaching the tubing to suction after each feeding

b) Clamping the tubing after formula feeding

c) Flushing the tubing with water and clamping it after each


feeding

d) Aspirate as much as possible from the tubing using a 50 ml


syringe

637. When an order reads that a drug be administered t.i.d, how often
should this drug be given?
a) Every three hours

b) Three times a day

c) Four times a day

d) Every other day

638. Dorsal recumbent position is used when performing the following


procedures EXCEPT:
a) Suppository insertion

‫مناقشة امتحانات البرومتريك للتمريض‬

https://www.facebook.com/groups/1639727976293954 Page 266


Prometric exam collection 2017

)‫)النسخة الرابعة‬

4‫ودالشمال‬

b) Cystoscopic examination

c) Urinary catheter insertion

d) Vaginal examination

639. Which action is believed to be the most useful in preventing


wound infections?:
a) Using sterial dressing supplies

b) Performing careful hand hygiene

c) Suggesting dietary supplements

d) All of the above

640. The single most effective way to prevent nosocomial infections is


to:

a) Isolate patients with infections

b) Cover the mouth and nose when coughing

c) Wash all equipment detergents

d) Practice continuous hand washing

‫مناقشة امتحانات البرومتريك للتمريض‬

https://www.facebook.com/groups/1639727976293954 Page 267


Prometric exam collection 2017

)‫)النسخة الرابعة‬

4‫ودالشمال‬

641. Vitamin K. 10 mg given IM is ordered. Vitamin K is available as 5


mg/ml. How much would the nurse administer?

a) 2 ml

b) 6 ml

c) 4 ml

d) 8 ml

642. Nonverbal communication includes all of the following EXCEPT?

a) Facial expression

b) Grooming and crying

c) Posture and mode of dress

d) Speaking in low voice

643. The condition in which a person is aware of his or her own heart
contraction without having to feel the pulse is called:

a) Arrhythmia

b) Dysrhythmia

‫مناقشة امتحانات البرومتريك للتمريض‬

https://www.facebook.com/groups/1639727976293954 Page 268


Prometric exam collection 2017

)‫)النسخة الرابعة‬

4‫ودالشمال‬

c) Pulse rhythm

d) Palpitation

644. Cyanosis is blue color of skin, is caused by:

a) Fever

b) Hypertension

c) Low tissue oxygenation

d) Kidney disease

645. Instrument which is used to examine the eye structure is called:

a) Ophthalmoscope

b) Laryngoscope

c) Otoscope

d) Bronchoscope

646. A pattern in which the nursing personnel divide the patient into
groups and complete their care together is called:

a) Primary method

‫مناقشة امتحانات البرومتريك للتمريض‬

https://www.facebook.com/groups/1639727976293954 Page 269


Prometric exam collection 2017

)‫)النسخة الرابعة‬

4‫ودالشمال‬

b) Team nursing

c) Nursing managed care case method

d) Case method

647. The term used to describe blood in the urine.

a) Glycosuria

b) Hematuria

c) Pyuria

d) Albuminuria

648. The normal color of urine is:

a) Dark amber

b) Reddish brown

c) Cloudy

d) Light yellow

649. When a person has a fever or diaphoresis, the urine output will be
which of the following:

‫مناقشة امتحانات البرومتريك للتمريض‬

https://www.facebook.com/groups/1639727976293954 Page 270


Prometric exam collection 2017

)‫)النسخة الرابعة‬

4‫ودالشمال‬

a) Increased and diluted

b) Increased and concentrated

c) Decreased and highly diluted

d) Decreased and highly concentrated

650. When administering medication via nasogastric tubing, clamp the


tube for at least:

a) One half hour prior to medication administration to prevent


complication

b) One half hours after instilling medication to allow for absorption

c) One hour prior to medication administration to prevent


complication

d) One and one half hours after instilling medication to allow for
absorption

651. The nurse chooses to inject a prescribed intramuscular medication


into the ventrogluteal site. If the nurse selects the site correctly, the
injection is administered into the.

‫مناقشة امتحانات البرومتريك للتمريض‬

https://www.facebook.com/groups/1639727976293954 Page 271


Prometric exam collection 2017

)‫)النسخة الرابعة‬

4‫ودالشمال‬

a) Hip

b) Arm

c) Thigh

d) Buttock

652. 500 mg of a drug is order. It is supplied in tablets of 1 gm per


tablet. How many tablets should be administered?

a) 0.5 tablet

b) 1 tablet

c) 1.5 tablet

d) 2 tablet

653. One of your patients complains of difficulty of breathing, all of the


following measurement which help improve breathing EXCEPT:

a) Put your patient in semi- fowler's position

b) Teach patient breathing techniques

c) Put the patient in prone position

‫مناقشة امتحانات البرومتريك للتمريض‬

https://www.facebook.com/groups/1639727976293954 Page 272


Prometric exam collection 2017

)‫)النسخة الرابعة‬

4‫ودالشمال‬

d) Give oxygen therapy

654. The process of removing poisonous substance through gastric


intubation is called:

a) Gastric Lavage

b) Gastric Gavage

c) Gastric Decompression

d) Gastric Tamponade

655. During the inflammation process, which of the following


characteristics occur first:

a) Swelling

b) Pain

c) Redness

d) Decreased functioning

656. Paracentesis is best described as:

a) The removal of fluid from the lung

‫مناقشة امتحانات البرومتريك للتمريض‬

https://www.facebook.com/groups/1639727976293954 Page 273


Prometric exam collection 2017

)‫)النسخة الرابعة‬

4‫ودالشمال‬

b) The removal of fluid or air from pleural cavity

c) The removal of body fluid from the abdominal cavity

d) The removal of secretion from the stomach

657. Which of the following is used to determine the activity of the


brain:

a) Electrocardiography

b) Electromyography

c) Electroencephalography

d) Echocardiography

658. When planning Mr. Asem care (50 years) who demonstrates
difficulty in breathing. Which of the following positions is most
appropriate?

a) On either side

b) Flat on his back

c) On his abdomen

d) Mid-Flower's position
‫مناقشة امتحانات البرومتريك للتمريض‬

https://www.facebook.com/groups/1639727976293954 Page 274


Prometric exam collection 2017

)‫)النسخة الرابعة‬

4‫ودالشمال‬

659. The following manifestations are commonly associated with a


fever, EXCEPT:

a) Headache

b) Pinkish and red skin color

c) Bradycardia

d) Convulsions in infants and child

660. Dorsal recumbent position is used when performing the following


procedures EXCEPT:

a) Suppository insertion

b) Cystoscopic examination

c) Urinary catheter insertion

d) Vaginal examination

661. If a vial of Gentamycin contains 80 mg in 2 ml, the physician order


is 16 mg every 8 hr., the nurse should give every time:

a) 0.1 ml

b) 0.2 ml
‫مناقشة امتحانات البرومتريك للتمريض‬

https://www.facebook.com/groups/1639727976293954 Page 275


Prometric exam collection 2017

)‫)النسخة الرابعة‬

4‫ودالشمال‬

c) 0.3 ml

d) 0.4 ml

662. If a nasogastric tube has been misplaced in the trachea during


preparation to obtain a gastric specimen, the nurse should
anticipate that the patient will:

a) Have difficulty in breathing

b) Swallow every few seconds

c) Gage without relief

d) Complain of feeling nauseated

663. Blood and urine analysis confirm a diagnosis of salicylate


overdose. The client is treated with gastric lavage. Which of the
following positions would be most appropriate for the client during
this procedure?

a) Lateral

b) Trednelenburg’s

c) Supine

‫مناقشة امتحانات البرومتريك للتمريض‬

https://www.facebook.com/groups/1639727976293954 Page 276


Prometric exam collection 2017

)‫)النسخة الرابعة‬

4‫ودالشمال‬

d) Lithotomy

664. All of the followings are signs of HYPOXIA Except:

a) Rapid pulse

b) Cyanosis

c) Rapid shallow respiration

d) Diarrhea.

665. Hypercabnia is:

a) Bluish discoloration of the skin nails beds and mucosal


membrane.

b) Inadequate alveolar ventilation can lead to hypoxia.

c) Accumulation of carbon dioxide in the blood.

d) Slow respiration rate.

666. Which position allow maximum chest expansion to the client:

a) Prone.

b) Semi folwer.

‫مناقشة امتحانات البرومتريك للتمريض‬

https://www.facebook.com/groups/1639727976293954 Page 277


Prometric exam collection 2017

)‫)النسخة الرابعة‬

4‫ودالشمال‬

c) Supine.

d) Lateral

667. Which one of the following its major function to supply energy:

a) Protein

b) Carbohydrates

c) Fats

d) Minerals

668. What number 18 indicate, regarding body mass index scale (BMI):

a)Underweight

b) Morbidly obese

c) Malnourished

d) Normal

669. Constipation is:

a) Fewer than 5 bowel movement per week.

b) Fewer than three bowel movement per week.


‫مناقشة امتحانات البرومتريك للتمريض‬

https://www.facebook.com/groups/1639727976293954 Page 278


Prometric exam collection 2017

)‫)النسخة الرابعة‬

4‫ودالشمال‬

c) Passage of liquid feces and increased frequency of defecation.

d) air or gas in GI tract

670. All of the followings are normal characteristics of feces Except:

a) Brown color.

b) Very hard and solid.

c) Amount: varies with diet.

d) Odor: affected by digested food

671. Oliguria is:

a) Production of abnormally large amount of urine by kidneys.

b) Lack of urine production, with no effective urine production.

c) voiding that either painful or difficult.

d) Low urine output, usually less than 500 ml a day, or 30 ml an


hour.

672. All of the followings are normal characteristics of urine Except:

a) Volume: 1200- 1500 ml per day

‫مناقشة امتحانات البرومتريك للتمريض‬

https://www.facebook.com/groups/1639727976293954 Page 279


Prometric exam collection 2017

)‫)النسخة الرابعة‬

4‫ودالشمال‬

b) Straw color.

c) Sterile, no microorganisms.

d) Cloudy.

673. Medical asepsis includes:

a) Hand washing.

b) Personal protective equipment (uniform, gown, gloves, face


mask).

c) Cleaning, Disinfecting, Sterilizing.

d) All of the above.

674. Acute infection is:

a) Infection may occur slowly and takes long time.

b) is limited to the specific part of the body where micro-


organisms remain.

c) Infection appears suddenly or in short time.

d) When microorganisms spread and damage different part of


body.
‫مناقشة امتحانات البرومتريك للتمريض‬

https://www.facebook.com/groups/1639727976293954 Page 280


Prometric exam collection 2017

)‫)النسخة الرابعة‬

4‫ودالشمال‬

675. Indirect transmission of infection includes:

a) Vehicle born transmission.

b) Vector borne transmission.

c) A only correct.

d) A and B are correct.

676. Incubation period of infection is:

a) Person is most infectious and nonspecific sign and symptom.

b) Organism growing and multiplying.

c) Recovery from infection.

d) Presence of specific sign and symptom.

677. All of the following laboratory data indicating infection EXCEPT:

a) Increased specific type of leukocyte.

b) Increased in body temp.

c) Decreased WBC.

d) Positive culture in blood, urine or sputum.


‫مناقشة امتحانات البرومتريك للتمريض‬

https://www.facebook.com/groups/1639727976293954 Page 281


Prometric exam collection 2017

)‫)النسخة الرابعة‬

4‫ودالشمال‬

678. Health is defined as:

a) State of complete physical, social, mental, and spiritual


wellbeing and not merely absence of disease.

b) Felling of wellbeing.

c) Absence of disease.

d) State of complete physical and mental wellbeing only.

679. The patient should be fasts (NPO) before the surgery for:

a) 24 hours.

b) 16 hours.

c) 6 to 8 hours.

d) 12 hours.

680. Complications of wound healing include:

a) Hematoma.

b) Nausea.

c) Hypertension.

‫مناقشة امتحانات البرومتريك للتمريض‬

https://www.facebook.com/groups/1639727976293954 Page 282


Prometric exam collection 2017

)‫)النسخة الرابعة‬

4‫ودالشمال‬

d) All of the above

681. Which of the following vital signs are normal?

a) Temp: 37, RR: 22, HR: 90, BP: 135/85.

b) 75, BP: 125/80. Temp: 36.7, RR: 18, HR:

c) Temp: 38, RR: 12, HR: 110, BP: 155/90.

d) Temp: 37.2, RR: 16, HR: 50, BP: 115/65

682. Hyperthermia is:

a) Body temperature between 36.4 – 37.4

b) Body temperature between 39 – 41

c) Body temperature below 36.

d) Body temperature between 38 – 40.

683. All of the following are clinical sign of fever except:

a) Palled.

b) Shivering.

c) Decrease thirst.
‫مناقشة امتحانات البرومتريك للتمريض‬

https://www.facebook.com/groups/1639727976293954 Page 283


Prometric exam collection 2017

)‫)النسخة الرابعة‬

4‫ودالشمال‬

d) Increase heart rate.

684. All of the following factors are increase respiratory rate except:

a) Decrease temp.

b) Stress.

c) Exercise.

d) Increase altitude.

685. The most safe and non-invasive site to measure the temperature is:

a) Oral site.

b) Auxiliary site.

c) Rectal site.

d) Tympanic site.

686. The difference between rectal and auxiliary temp is:

a) 0.5 c

b) 0.8 c

c) 2 c
‫مناقشة امتحانات البرومتريك للتمريض‬

https://www.facebook.com/groups/1639727976293954 Page 284


Prometric exam collection 2017

)‫)النسخة الرابعة‬

4‫ودالشمال‬

d) 1 c.

687. In which phase of nursing process the nurse collect data about the
client:

a) Diagnosis.

b) Assessment.

c) Implementation.

d) Evaluation.

688. Chief complains includes:

a) Immunization and childhood illness.

b) Hoppies and sleep pattern.

c) Risk factor for certain diseases.

d) The answer given to question “what brought you to the hospital”

689. The chief complain should be record using:

a) Nurse words.

b) Patient words.

‫مناقشة امتحانات البرومتريك للتمريض‬

https://www.facebook.com/groups/1639727976293954 Page 285


Prometric exam collection 2017

)‫)النسخة الرابعة‬

4‫ودالشمال‬

c) Physician words.

d) None of the above.

690. Nursing diagnosis is:

a) A statement that describes actual health problem only.

b) A statement that describes potential health problem only.

c) A statement that describes actual and potential health problems.

d) Puts the nursing care plan into action.

691. The nursing diagnosis includes all the following except:

a) Problem.

b) Time.

c) Sign.

d) Etiology.

692. Pressure ulcer risk factors include:

a) Immobility and inactivity.

b) Fecal and urinary incontinence.


‫مناقشة امتحانات البرومتريك للتمريض‬

https://www.facebook.com/groups/1639727976293954 Page 286


Prometric exam collection 2017

)‫)النسخة الرابعة‬

4‫ودالشمال‬

c) Decreased mental status.

d) All of the above.

693. All of the following are clinical manifestations of fluid volume


excess except:

a) Edema

b) Oliguria

c) Distended neck veins

d) Increased CVP

694. Signs and symptoms of Hypovolemic shock are all of the


following except:

a) Tachycardia

b) Hypertension

c) Pallor and cyanosis

d) Tachypnea

695. Normal Saline 0.9% solution is considered as which of the


following:
‫مناقشة امتحانات البرومتريك للتمريض‬

https://www.facebook.com/groups/1639727976293954 Page 287


Prometric exam collection 2017

)‫)النسخة الرابعة‬

4‫ودالشمال‬

a) Isotonic solution

b) Hypotonic solution

c) Hypertonic solution

d) None of the above

696. Apnea is medical term means:

a) Rapid pulse

b) Increase body temperature

c) Stop breathing

d) Low blood pressure

697. When blood sugar level is above normal range, this means that
patient has:

a) Hypotension

b) Hypoglycemia

c) Hyperglycemia

d) Bradycardia

‫مناقشة امتحانات البرومتريك للتمريض‬

https://www.facebook.com/groups/1639727976293954 Page 288


Prometric exam collection 2017

)‫)النسخة الرابعة‬

4‫ودالشمال‬

698. The most important thing should be done after any nursing action
is:

a) Documentation

b) Nursing diagnosis

c) Planning

d) All of the above

699. All of the following are assessment sites for body temperature
except:

a) Oral Site.

b) Rectal Site.

c) Axillary Site.

d) Apical

700. Mr. Ashraf aged 35 years old, his pulse rate is found to be 120
bpm. His heart rate could be described as:

a) Tachypnea.

b) Tachycardia.
‫مناقشة امتحانات البرومتريك للتمريض‬

https://www.facebook.com/groups/1639727976293954 Page 289


Prometric exam collection 2017

)‫)النسخة الرابعة‬

4‫ودالشمال‬

c) Bradypnea

d) Bradycardia.

701. Which of the following factors are affecting body temperature:

a) Food intake.

b) Age and Gender

c) Climate.

d) All of the above

702. The condition in which the body temperature is above the average
normal is called:

a) Bradypnea

b) Fever.

c) Hypertension.

d) Hypothermia.

703. Which of the following best describes the technique of palpation:

a) The use of the sense of hearing to listen for sounds.

‫مناقشة امتحانات البرومتريك للتمريض‬

https://www.facebook.com/groups/1639727976293954 Page 290


Prometric exam collection 2017

)‫)النسخة الرابعة‬

4‫ودالشمال‬

b) The use of tapping on a particular part of the body to produce


sounds.

c) The use of the senses to evaluate general appearance of the


patient.

d) The sense of touch to feel the body by slight or deep pressure.

704. Inhalation or inspiration means:

a) Stop breathing.

b) Difficult breathing.

c) Breathing out.

d) Breathing in

705. Normal body temperature is ranging from:

a) 35.8 – 37.4 C

b) 34.5 – 36.5 C

c) 35.0 – 38.0 C

d) 36.5 – 38.5 C

‫مناقشة امتحانات البرومتريك للتمريض‬

https://www.facebook.com/groups/1639727976293954 Page 291


Prometric exam collection 2017

)‫)النسخة الرابعة‬

4‫ودالشمال‬

706. Pulse pressure is defined as which of the following:

a) Difference between systolic and diastolic pressure.

b) Expansion of the artery as blood moves through it.

c) Difference between arterial and venous pressure.

d) Difference between venous and systolic pressure.

707. Unoccupied bed making means:

a) Changing linen when the patient remains in the bed.

b) Changing linen when the bed empty.

c) State in which a person is relieved of distress.

d) None of the above.

708. The most accurate time for measuring pulse rate is:

a) 30 seconds.

b) 15 seconds.

c) 60 seconds.

d) 45 seconds.
‫مناقشة امتحانات البرومتريك للتمريض‬

https://www.facebook.com/groups/1639727976293954 Page 292


Prometric exam collection 2017

)‫)النسخة الرابعة‬

4‫ودالشمال‬

709. Practices that promote health through personal cleanliness is


called:

a) General appearance.

b) Self-image.

c) Insomnia.

d) Hygiene.

710. All of the following are therapeutic bathes except:

a) Shampooing bath.

b) Whirlpool bath,

c) Medicated bath.

d) Sitz bath.

711. Orthopnea is defined as:

a) Breathing facilitated by sitting or standing up.

b) Absence of breathing.

c) Rapid breathing.

‫مناقشة امتحانات البرومتريك للتمريض‬

https://www.facebook.com/groups/1639727976293954 Page 293


Prometric exam collection 2017

)‫)النسخة الرابعة‬

4‫ودالشمال‬

d) Difficult breathing.

712. Which of the following is a form of radiography:

a) Fluoroscopy.

b) Contrast medium.

c) C.T. Scan

d) All of the above

713. The examination that indicates physical inspection of the vagina


and cervix with palpation of uterus and ovaries is called:

a) A Pap test.

b) Electrocardiography.

c) Pelvic examination.

d) Paracentesis.

714. Procedure that involves the insertion of a needle between lumber


vertebra in the spine but below the spinal cord itself is called:

a) Lumber puncture.

‫مناقشة امتحانات البرومتريك للتمريض‬

https://www.facebook.com/groups/1639727976293954 Page 294


Prometric exam collection 2017

)‫)النسخة الرابعة‬

4‫ودالشمال‬

b) Paracentesis.

c) Pelvic Examination.

d) Electromyography.

715. Which of the following is considered as a benefit of bathing:

a) Improving self-image.

b) Eliminating body odor.

c) Stimulating circulation.

d) All of the above.

716. Ventilation means:

a) Movement of air in and out of the chest.

b) Movement of air out of the chest.

c) Movement of air in the chest.

d) None of the above.

717. All of the following are common factors that invalidated


examination or test results except:

‫مناقشة امتحانات البرومتريك للتمريض‬

https://www.facebook.com/groups/1639727976293954 Page 295


Prometric exam collection 2017

)‫)النسخة الرابعة‬

4‫ودالشمال‬

a) Inadequate specimen volume.

b) Failure to send the specimen in a timely manner.

c) Correct diet preparation.

d) Insufficient bowel cleansing.

718. All of the following are physical assessment techniques except:

a) Inspection.

b) Percussion.

c) Puncturing.

d) Palpation

719. All of the following are post procedural nursing responsibilities


except:

a) The nurse has to attend the patient for comfort and rest.

b) Care of specimens.

c) Assist the examiner.

d) Record and report of information.

‫مناقشة امتحانات البرومتريك للتمريض‬

https://www.facebook.com/groups/1639727976293954 Page 296


Prometric exam collection 2017

)‫)النسخة الرابعة‬

4‫ودالشمال‬

720. A patient is prepared for hemodialysis. He receives heparin before


therapy before primarily to help:

a) Relieve discomfort

b) Prevent blood clotting

c) Maintain blood pressure

d) Stimulate the production of urine

721. Another name for a stone in the urinary tract is:

a) Calix

b) Calculus

c) Calcemia

d) Calcitonin

722. Urine that remains in pt's bladder after he voids is called:

a) Reflux urine

b) Over flow urine

c) Retention urine

‫مناقشة امتحانات البرومتريك للتمريض‬

https://www.facebook.com/groups/1639727976293954 Page 297


Prometric exam collection 2017

)‫)النسخة الرابعة‬

4‫ودالشمال‬

d) Residual urine

723. For a normal person the urine specific gravity is ranged between:

a) 1.000 and 1.010

b) 1.015 and 1.025

c) 1.025 and 1.050

d) 1.050 and 1.070

724. Which of the following is OBJECTIVE data:

a. Dizziness

b. Chest pain

c. Anxiety

d. Blue nails

725. Among the following statements, which should be given the


HIGHEST priority?

a. Client is in extreme pain

b. Client’s blood pressure is 60/40

‫مناقشة امتحانات البرومتريك للتمريض‬

https://www.facebook.com/groups/1639727976293954 Page 298


Prometric exam collection 2017

)‫)النسخة الرابعة‬

4‫ودالشمال‬

c. Client’s temperature is 40 degree Centigrade

d. Client is cyanotic.

726. Considered as the most accessible and convenient method for


temperature taking is:

a. Oral

b. Rectal

c. Tympanic

d. Axillary

727. In cleaning the thermometer after use, The direction of the


cleaning to follow Medical Asepsis is :

a. From bulb to stem

b. From stem to bulb

c. From stem to stem

d. From bulb to bulb.

728. Which is a preferable arm for BP taking?

‫مناقشة امتحانات البرومتريك للتمريض‬

https://www.facebook.com/groups/1639727976293954 Page 299


Prometric exam collection 2017

)‫)النسخة الرابعة‬

4‫ودالشمال‬

a. An arm with the most contraptions

b. The left arm of the client with a CVA affecting the right brain

c. The right arm

d. The left arm.

729. In palpating the client’s breast, which of the following position is


necessary for the patient to assume before the start of the
procedure?

a. Supine

b. Dorsal recumbent

c. Sitting

d. Lithotomy.

730. When is the best time to collect urine specimen for routine
urinalysis and culture and sensitivity?

a. Early morning

b. Later afternoon

c. Midnight
‫مناقشة امتحانات البرومتريك للتمريض‬

https://www.facebook.com/groups/1639727976293954 Page 300


Prometric exam collection 2017

)‫)النسخة الرابعة‬

4‫ودالشمال‬

d. Before breakfast.

731. This is a process of removing pathogens but not their spores:

a. Sterilization

b. Autoclaving

c. Disinfection

d. Medical asepsis

732. This is the single most important procedure that prevents cross
contamination and infection

a. Cleaning

b. Disinfecting

c. Sterilizing

d. Hand washing.

733. The best example of the nursing order is:

a. Encourage fluids.

b. Change patient position every 2 hours on even hours.

‫مناقشة امتحانات البرومتريك للتمريض‬

https://www.facebook.com/groups/1639727976293954 Page 301


Prometric exam collection 2017

)‫)النسخة الرابعة‬

4‫ودالشمال‬

c. Provide oral fluid.

d. All of the above.

734. The term holism refers to:

a. Reactions that occur when equilibrium is disturbed.

b. How an organism responds to change.

c. A relatively stable state of physiologic equilibrium.

d. The sum of physiological, emotional, social, and spiritual health,


and determines how whole person feels.

735. When the nurse changes the client's dressing which nursing action
is correct:

a. The nurse removes the solid dressing with sterile gloves.

b. The nurse frees the tape by pulling it away from the incision.

c. The nurse encloses the solid dressing within a latex gloves.

d. The nurse cleans the wound in circles toward the incision.

736. Which of the following IS NOT true about preparing medication:

‫مناقشة امتحانات البرومتريك للتمريض‬

https://www.facebook.com/groups/1639727976293954 Page 302


Prometric exam collection 2017

)‫)النسخة الرابعة‬

4‫ودالشمال‬

a. Prepare medication under well-lighted area.

b. Work alone without interruptions.

c. Do not use medication with a missing label.

d. Check the label of the drug two times.

737. How often should an inactive patient's positions be changed?

a. At least every 3 hours.

b. At least every 2 hours.

c. At least every 4 hours.

d. At least once a shift.

738. Hypertonic saline (fleet) enema works by:

a. Distending the rectum.

b. Irritating the rectum.

c. Lubricating and softening stool.

d. Moistening stool.

‫مناقشة امتحانات البرومتريك للتمريض‬

https://www.facebook.com/groups/1639727976293954 Page 303


Prometric exam collection 2017

)‫)النسخة الرابعة‬

4‫ودالشمال‬

739. The physician orders dextrose 5% in water, 1,000 ml to be infused


over 8 hours. The I.V. tubing delivers 15 drops/ml. The nurse
should run the I.V. infusion at a rate of:

a) 15 drops/minute

b) 21 drops/minute.

c) 32 drops/minute.

d) 125 drops/minute

740. Hypertension is defined as persistent blood pressure levels in


which the systolic and diastolic above

a) 110/60 mmHg

b) 130/80 mmHg

c) 120/70 mmHg

d) 140/90 mmHg

741. Of the following, which are the earliest signs of excessive


pressure?

a) Pale appearance of the skin


‫مناقشة امتحانات البرومتريك للتمريض‬

https://www.facebook.com/groups/1639727976293954 Page 304


Prometric exam collection 2017

)‫)النسخة الرابعة‬

4‫ودالشمال‬

b) Reddened appearance of the skin

c) Ulcer formation on the skin

d) Dark or cyanotic color to the skin

742. When an order reads that a drug be administered q.i.d, how often
should this drug be given?

a) Every three hours

b) Three times a day

c) Four times a day

d) Every other day

743. Which action is believed to be the most useful in preventing


wound infections?:

a) Using sterial dressing supplies

b) Performing careful hand hygiene

c) Suggesting dietary supplements

d) All of the above

‫مناقشة امتحانات البرومتريك للتمريض‬

https://www.facebook.com/groups/1639727976293954 Page 305


Prometric exam collection 2017

)‫)النسخة الرابعة‬

4‫ودالشمال‬

744. The single most effective way to prevent nosocomial infections is


to:

a) Isolate patients with infections

b) Cover the mouth and nose when coughing

c) Wash all equipment detergents

d) Practice continuous hand washing

745. Vitamin K. 10 mg given IM is ordered. Vitamin K is available as 5


mg/ml. How much would the nurse administer?

a) 2 ml

b) 6 ml

c) 4 ml

d) 8 ml

746. Which one of the following diets include only water, tea, coffee,
clear juice:

a) Clear liquid diet

b) Soft diet
‫مناقشة امتحانات البرومتريك للتمريض‬

https://www.facebook.com/groups/1639727976293954 Page 306


Prometric exam collection 2017

)‫)النسخة الرابعة‬

4‫ودالشمال‬

c) Full liquid diet

d) Diabetic diet

747. The intentional wound is:

a) Occur during therapy.

b) Occur accidentally.

c) The mucous membrane or skin surface is broken.

d) The tissue is traumatized without a break in the skin.

748. The abrasion is:

a) Open wound involving skin only

b) It is due to surface scrape.

c) It is painful wound.

d) All of the above.

749. All of the followings are risk factors for nosocomial infections
EXCEPT:

a) Poor hand washing.

‫مناقشة امتحانات البرومتريك للتمريض‬

https://www.facebook.com/groups/1639727976293954 Page 307


Prometric exam collection 2017

)‫)النسخة الرابعة‬

4‫ودالشمال‬

b) Using sterile techniques.

c) Contamination of closed drainage system.

d) Improper procedure technique (dressing, suctioning,


catherization).

750. Acute infection is:

a) Infection may occur slowly and takes long time.

b) is limited to the specific part of the body where microorganisms


remain.

c) Infection appears suddenly or in short time.

d) When microorganisms spread and damage different part of


body.

751. Indirect transmission of infection includes:

a) Vehicle born transmission.

b) Vector borne transmission.

c) A only correct.

d) A and B are correct.


‫مناقشة امتحانات البرومتريك للتمريض‬

https://www.facebook.com/groups/1639727976293954 Page 308


Prometric exam collection 2017

)‫)النسخة الرابعة‬

4‫ودالشمال‬

752. Incubation period of infection is:

a) Person is most infectious and non-specific sign and symptom.

b) Organism growing and multiplying.

c) Recovery from infection.

d) Presence of specific sign and symptom.

753. All of the followings are Signs of systematic infection EXCEPT:

a) Fever.

b) Vomiting.

c) Fatigue and loss of energy.

d) Enlargement and tenderness of lymph node.

754. All of the followings are etiologies of self-care deficit EXCEPT:

a) scar and abrasions.

b) Activity intolerance or weakness.

c) Mental impairment.

d) Visual impairment.
‫مناقشة امتحانات البرومتريك للتمريض‬

https://www.facebook.com/groups/1639727976293954 Page 309


Prometric exam collection 2017

)‫)النسخة الرابعة‬

4‫ودالشمال‬

755. Skin turgor is:

a) Decrease sensation in extremities due to nerves system disease.

b) Collection of fluids inside the tissue.

c) Elasticity by lifting and pulling the skin on an extremity.

d) None of the above.

756. Pressure ulcer risk factors include:

a) Immobility and inactivity.

b) Fecal and urinary incontinence.

c) Decreased mental status.

d) All of the above.

757. Scrub nurse role includes:

a) Assist the surgeons.

b) Draping the client with sterile drapes.

c) Handling sterile equipment’s and supplies.

d) All of the above.


‫مناقشة امتحانات البرومتريك للتمريض‬

https://www.facebook.com/groups/1639727976293954 Page 310


Prometric exam collection 2017

)‫)النسخة الرابعة‬

4‫ودالشمال‬

758. Example of health promotion:

a) Immunization.

b) Direct care (give medication).

c) Stop cigarette smoking.

d) All of the above is correct.

759. Immunization is an example of:

a) Health promotion.

b) Prevent illness.

c) Restoring health.

d) Care of the dying.

760. The primary methods used to examine the skin, mucus membrane
and hair are:

a) Inspection and palpation

b) Percussion and inspection

c) Palpation and auscultation

‫مناقشة امتحانات البرومتريك للتمريض‬

https://www.facebook.com/groups/1639727976293954 Page 311


Prometric exam collection 2017

)‫)النسخة الرابعة‬

4‫ودالشمال‬

d) Auscultation and percussion

761. Hemoptysis is:

a) Cough with secretions.

b) Cough without secretions.

c) Difficult breathing.

d) Blood in the sputum.

762. Which of the following best define pulse deficit?

a) The difference between the apical and radial pulse rates.

b) The difference between the systole and diastole values.

c) The difference between the brachial and radial pulse rates.

d) The difference between the temporal and femoral pulse rates.

763. When the blood pressure is 140 / 100 mmHg, the pulse pressure is:

a) 40 mmHg

b) 142 mmHg

c) 100 mmHg
‫مناقشة امتحانات البرومتريك للتمريض‬

https://www.facebook.com/groups/1639727976293954 Page 312


Prometric exam collection 2017

)‫)النسخة الرابعة‬

4‫ودالشمال‬

d) 242 mmHg

764. The relationship between the pulse and respiratory rate is


represented by Which of the following ratios:

a) One respiration to 2 or 3 heartbeat.

b) One respiration to 3 or 4 heartbeat.

c) One respiration to 4 or 5 heartbeat.

d) One respiration to 5 or 6 heartbeat.

765. All the following can cause hypertension except:

a) Excessive exercise and activity

b) Pain and emotions

c) Diseases such as atherosclerosis

d) Hemorrhage

766. The type of pulse that is strong and doesn’t disappear with
moderate pressure is known as:

‫مناقشة امتحانات البرومتريك للتمريض‬

https://www.facebook.com/groups/1639727976293954 Page 313


Prometric exam collection 2017

)‫)النسخة الرابعة‬

4‫ودالشمال‬

a) Absent pulse

b) Thready pulse

c) Weak pulse

d) Bounding pulse

767. A process of heat loss which involves the transfer of heat from one
surface to Another without contact is:

a) Radiation

b) Conduction

c) Convection

d) Evaporation

768. The geriatric client with a history of heart attack and hypertension
presented with complaints of unusual weakness and fatigue. Upon
examination, the nurse noted diminished breath sounds throughout
the lung fields and crackles on both the lower lobes. Which of the
following should be the next action of the nurse?
a. Notify the physician and document initial findings.

‫مناقشة امتحانات البرومتريك للتمريض‬

https://www.facebook.com/groups/1639727976293954 Page 314


Prometric exam collection 2017

)‫)النسخة الرابعة‬

4‫ودالشمال‬

b. Facilitate chest x-ray

c. Start a thorough physical examination and history.

d. Recheck the client after five minutes and see if there are
changes.

769. The client presented with complaints of rapid and shallow


breathing. Upon lung auscultation, breath sound is diminished on
both lung bases and there’s audible course crackles on the upper
lobes. If the condition worsens, which of the following tests can be
used to determine if intubation is necessary?
a. Peak flow meter
b. Partial Oxygen Saturation in Arterial Blood Gas
c. Oxygen Saturation in Pulse Oximeter
d. Lung Function test
770. The client with a genetic enzyme deficiency had an abnormal and
different response to the drug administered for the first time. This
reaction is called as:
a. Allergic reaction
b. Cumulative effect
c. Idiosyncratic effect
‫مناقشة امتحانات البرومتريك للتمريض‬

https://www.facebook.com/groups/1639727976293954 Page 315


Prometric exam collection 2017

)‫)النسخة الرابعة‬

4‫ودالشمال‬

d. Synergistic effect
771. The ICU nurse is preparing the instruments needed for
endotracheal intubation. The nurse is knowledgeable that clients in
the ICU often need mechanical assistance to maintain a patent
airway. Which of the following is NOT an indication for
endotracheal intubation?
a. Respiratory distress
b. Prolonged mechanical ventilation
c. High risk of aspiration
d. Ineffective clearance of secretions
772. The client with severe sensory alteration is transferred to the
intensive care unit. Moments later, the client became restless and
agitated with complaints of hallucinations. The nurse noted the
change in the level of consciousness as:
a. Delirium
b. Dementia
c. Stupor
d. Confusion

‫مناقشة امتحانات البرومتريك للتمريض‬

https://www.facebook.com/groups/1639727976293954 Page 316


Prometric exam collection 2017

)‫)النسخة الرابعة‬

4‫ودالشمال‬

773. The client complained of abdominal discomfort on the first


postoperative day. Upon percussion of the lower abdomen, the
nurse expects to hear:
a. Dull
b. b. Flat
c. Tympanic
d. Resonant
774. The nurse is performing a physical examination to the client with
hearing difficulty. The nurse activated the tuning fork and placed it
on top of the client’s head. What test did the nurse perform?
a. Whisper test
b. Rinne test
c. Audiometer
d. Weber test
775. The nurse is in charge of the client with cardiac complaints. As a
part of the cardiac assessment, the nurse auscultates the mitral area
which is located at:
a. 2nd ICS, Right sternal borde
b. 2nd ICS, Left sternal border
c. 5th ICS, Left sternal borde
‫مناقشة امتحانات البرومتريك للتمريض‬

https://www.facebook.com/groups/1639727976293954 Page 317


Prometric exam collection 2017

)‫)النسخة الرابعة‬

4‫ودالشمال‬

d. 5th ICS, Medial to the midclavicular line

776. The nurse is caring for a client with abalanced suspension traction
with a Thomas splint. The nurse observes that the left leg of the
client is externally rotated. Which of the following is the priority of
the nurse?
a. Place a trochanter roll outside the thigh.
b. Perform resistive range of motion of the affected leg
c. Adduct and internally rotate the left leg.
d. Maintain the left leg in a neutral position.
777. The client is admitted and is on the fourth cycle of chemotherapy.
During the night shift, the nurse noted signs of extravasation.
Which of the following is NOT a sign of extravasation?
a. Local infection
b. Tissue breakdown
c. Redness and heat on the site
d. Pain on the IV site

‫مناقشة امتحانات البرومتريك للتمريض‬

https://www.facebook.com/groups/1639727976293954 Page 318


Prometric exam collection 2017

)‫)النسخة الرابعة‬

4‫ودالشمال‬

778. The nurse is instructing the client about early detection of cancer.
The nurse should instruct the client to perform breast self-
examination during:
a. The first day of every month
b. B The first day of menstruation
c. Before menstruation
d. After menstruation
779. The geriatric client presented with complaints of difficulty in
swallowing, fatigue, alternating constipation and diarrhea,
abdominal pain, and blood in the stools. Which of the following
symptoms is NOT included in the warning signs of cancer?
a. Irregular pattern of constipation and diarrhea
b. Blood in the stools
c. Difficulty in swallowing
d. Frequent vomiting
780. The client presented with complaints of body weakness, dizziness
and chest pain. Upon careful assessment, the nurse suspects
Angina Pectoris. Which of the following statements made by the
client can confirm this?

‫مناقشة امتحانات البرومتريك للتمريض‬

https://www.facebook.com/groups/1639727976293954 Page 319


Prometric exam collection 2017

)‫)النسخة الرابعة‬

4‫ودالشمال‬

a. I suddenly felt a pain on my chest which radiates to my back and


arms.
b. I suddenly felt a sharp pain on my lower abdomen.
c. The pain does not subside even if I rest.
d. The pain goes all the way down to my stomach.
781. The client from the OR is transferred to the post-anesthesia care
unit after surgical repair of abdominal aortic aneurysm. Which of
the following assessment findings would indicate that the repair
was successful?
a. Urine output of 50 mL/hr.
b. Presence of non-pitting, peripheral edema
c. Clear sclera.
d. Presence of carotid bruit
782. The client is scheduled for cardiac catheterization because the
physician wants to view the right side of the heart. Which of the
following would the nurse expect to see in this procedure?
a. A dye is injected to facilitate the viewing of the heart
b. Thallium is injected to facilitate the scintillation camera
c. A probe with a transducer tip is swallowed by the client.
d. A tiny ultrasound probe is inserted into the coronary artery
‫مناقشة امتحانات البرومتريك للتمريض‬

https://www.facebook.com/groups/1639727976293954 Page 320


Prometric exam collection 2017

)‫)النسخة الرابعة‬

4‫ودالشمال‬

783. The nurse is caring for an adult patient with extensive burns on the
front of the trunk, including the genitalia, and the fronts of right
legs. Using the rule of nines, the nurse would document that the
burn size as:
a. 13%
b. 17%
c. 28%
d. 37%
784. Following abdominal surgery, a child has a nasogastric tube
connected to suction. Several hours after surgery, the child tells the
nurse that he is nauseated and then vomits approximately 200 ml
of fluid. Which of these actions should the nurse take first?
a. Notify the physician
b. Check if the nasogastric tube can be irrigated
c. Discontinue the section attached to the nasogastric tube
d. Auscultate for bowel sounds
785. During the first 24 hours post burn, fluid replacement is the
treatment priority. The assessment that would alert the nurse that
the fluid protocol is ineffective is:
a. Marked edema in the burn area.
‫مناقشة امتحانات البرومتريك للتمريض‬

https://www.facebook.com/groups/1639727976293954 Page 321


Prometric exam collection 2017

)‫)النسخة الرابعة‬

4‫ودالشمال‬

b. Rectal temperature of 101º F


c. Crackles in the lower left lobe.
d. Urine output of 20 mL/hour.
786. A client who had a myocardial infarction has an order for digoxin
(Lanoxin)0.25 mg po daily and furosemide (Lasix) 40 mg
intravenously, stat. Because the client is receiving these
medications, the client should be assessed for which of the
following manifestations?
a. Tachycardia
b. Hypokalemia
c. Hypertension
d. Oliguria
787. During the nurse’s preoperative assessment, the nurse notices that
the patient is extremely anxious. The patient’s blood pressure is
142/92 mm Hg, heart rate is 104 per minute, and respirations are
34. The nurse should:
a. Notify the anesthesiologist or surgeon
b. Call the operating room and cancel the surgery.
c. Go ahead and give the preoperative medicine early to help calm
the patient.
‫مناقشة امتحانات البرومتريك للتمريض‬

https://www.facebook.com/groups/1639727976293954 Page 322


Prometric exam collection 2017

)‫)النسخة الرابعة‬

4‫ودالشمال‬

d. Instruct the patient on possible postoperative complications.


788. A client who has Alzheimer’s disease is told by the nurse to brush
his teeth. He shouts angrily, “Tomato soup!” Which of these
comments by the nurse would be appropriate?
a. Here is your toothbrush. First get your toothpaste, then wet your
brush.”
b. “I don’t understand what you mean by “tomato soup.”
c. “You seem upset.”
d. “I’ll brush your teeth for you.”
789. An infant on the pediatric unit suddenly starts to have generalized
seizure activity. Which of these actions should the nurse take first?
a. Insert a padded tongue blade into the infant’s mouth.
b. Test the infant’s pupillary response to light at least twice during
the seizure.
c. Suction the infant’s oropharynx several times during the seizure.
d. Note the duration of the infant’s seizure.
790. Because an infant has a meningomyelocele in the lumbosacral
area, the infant should be observed for early symptoms of
hydrocephalus, which include
a. Overriding of cranial bones
‫مناقشة امتحانات البرومتريك للتمريض‬

https://www.facebook.com/groups/1639727976293954 Page 323


Prometric exam collection 2017

)‫)النسخة الرابعة‬

4‫ودالشمال‬

b. A soft, flat anterior fontanel


c. An increase in head circumference
d. Prominent scalp veins
791. A nurse in a newborn nursery receives a phone call to prepare for
the admission of a 43-week-gestation newborn with Apgar scores
of 1 and 4. In planning for the admission of this infant, the nurse’s
highest priority should be to:
a. Connect the resuscitation bag to the oxygen outlet
b. Turn on the apnea and cardiorespiratory monitors
c. Set the radiant warmer control temperature at 36.5* C (97.6*F)
d. Set up the intravenous line with 5% dextrose in water
792. A newly delivered infant has a pink trunk and blue hands and feet,
pulse rate of 60 and does not respond to your attempts to stimulate
her. She also appears to be limp and taking slow, gasping breaths.
What is her Apgar score
a. 3
b. 6
c. 7
d. 9

‫مناقشة امتحانات البرومتريك للتمريض‬

https://www.facebook.com/groups/1639727976293954 Page 324


Prometric exam collection 2017

)‫)النسخة الرابعة‬

4‫ودالشمال‬

793. (TB) has a productive cough and hemoptysis. Which of the


following types of isolation room would be the BEST choice for
the patient?
a. Negative-pressure
b. Positive-pressure
c. Reverse isolation
d. D Standard isolation
794. A hospitalized child develops exanthema (rash) that covers the
trunk and extremities. The nurse reviews the child’s health history
and notes that the child was exposed to varicella 2 weeks ago.
Which nursing intervention is most appropriate to implement?
a. Immediately admit the client to any available bed.
b. Place the child in a private room on strict isolation.
c. Assess the progression of the exanthema and report it to the health
care provider.
d. Allow the child to play in the playroom until the health care
provider can be contacted
795. A nurse in the postpartum unit is caring for a client who has just
delivered a newborn infant following a pregnancy with placenta
previa. The nurse reviews the plan of care and prepares to monitor
‫مناقشة امتحانات البرومتريك للتمريض‬

https://www.facebook.com/groups/1639727976293954 Page 325


Prometric exam collection 2017

)‫)النسخة الرابعة‬

4‫ودالشمال‬

the client for which of the following risks associated with placenta
previa?
a. Infection
b. Hemorrhage
c. Chronic hypertension
d. Disseminated intravascular coagulation
796. A nurse is suctioning fluids from a client through an endotracheal
tube. During the suctioning procedure, the nurse notes on the
monitor that the heart rate decreases. Which of the following is the
most appropriate nursing intervention?
a. Ensure that the suction is limited to 15 seconds
b. Continue to suction
c. Hold the procedure and re-oxygenate the client
d. Notify the physician immediately.
797. An intubated patient is receiving continuous enteral feedings
through a Salem sump tube at a rate of 60ml/hr. Gastric residuals
have been 30-40ml when monitored Q4H. You check the gastric
residual and aspirate 220ml. What is your first response to this
finding?
a. Notify the doctor immediately.
‫مناقشة امتحانات البرومتريك للتمريض‬

https://www.facebook.com/groups/1639727976293954 Page 326


Prometric exam collection 2017

)‫)النسخة الرابعة‬

4‫ودالشمال‬

b. Discard the 220ml, and clamp the NG tube


c. Give a prescribed GI stimulant such as metoclopramide (Reglan).
d. Discontinue the feeding, and clamp the NG tube.
798. You’re performing an general assessment on patient who is 52 y.o.
In which order do you proceed if the abdomen is not included in
this session of assessment?
a. Observation, percussion, palpation, auscultation
b. Inspection, palpation, auscultation, percussion
c. Percussion, palpation, auscultation, observation
d. Inspection, percussion, observation, auscultation
799. When teaching a client with a cardiac problem, who is on a a low
saturated fat diet, the nurse should stress the importance of
decreasing the intake of:
a. Vegetables and whole grains
b. Fish and shrimp
c. Nuts and beans
d. Butter, lamb meats
800. A 68-year-old woman is scheduled to undergo mitral valve
replacement for severe mitral stenosis and mitral regurgitation.
Although the diagnosis was made during childhood, she did not
‫مناقشة امتحانات البرومتريك للتمريض‬

https://www.facebook.com/groups/1639727976293954 Page 327


Prometric exam collection 2017

)‫)النسخة الرابعة‬

4‫ودالشمال‬

have any symptoms until 4 years ago. Recently, she noticed


increased symptoms, despite daily doses of digoxin and
furosemide. During the initial interview with the nice lady, the
nurse would most likely learn that the client’s childhood health
history included:
a. Chicken pox
b. Rheumatic fever
c. Meningitis
d. Poliomyelitis
801. A client has driven himself to the ER. He is 50 years old, has a
history of hypertension, and informs the nurse that his father died
of a heart attack at 60 years of age. The client is presently
complaining of indigestion. The nurse connects him to an ECG
monitor. The nurse’s next action would be to:
a. Start intravenous line and draw blood for cardiac enzymes
b. Begin intravenous line then call doctor
c. Administering oxygen at 2 L/minute per NC then begin
intravenous line
d. Administering oxygen 12 l/minute per oxygen face mask and
obtain a portable chest radiograph
‫مناقشة امتحانات البرومتريك للتمريض‬

https://www.facebook.com/groups/1639727976293954 Page 328


Prometric exam collection 2017

)‫)النسخة الرابعة‬

4‫ودالشمال‬

802. A nurse is assessing the blood pressure of a client diagnosed with


primary hypertension. The nurse ensures accurate measurement by
considering which of the following?
a. Taking a blood pressure within 5 minutes after nicotine or caffeine
ingestion.
b. Measuring the blood pressure after the client has been seated
quietly for 5 minutes.
c. Using a cuff with a rubber bladder that encircles at least 30% of
the limb.
d. Seating the client with arm bared, supported, lower than heart
level.
803. A 35-year-old male was knifed in the street fight, admitted through
the ER, and is now in the ICU. An assessment of his condition
reveals the following symptoms: respirations shallow and rapid,
CVP 15 cm H2O, BP 90 mm Hg systolic, skin cold and pale,
urinary output 30 mL/hr for the last 2 hours. Analyzing these
symptoms, the nurse will base a nursing diagnosis on the
conclusion that the client has which of the following conditions?
a. Hypovolemic shock
b. Cardiac tamponade
‫مناقشة امتحانات البرومتريك للتمريض‬

https://www.facebook.com/groups/1639727976293954 Page 329


Prometric exam collection 2017

)‫)النسخة الرابعة‬

4‫ودالشمال‬

c. Wound dehiscence
d. Atelectasis
804. A nurse is assessing the neurovascular of a client who has returned
to the surgical nursing unit 4 hours ago after undergoing aortoiliac
bypass graft. The affected leg is warm, and the nurse notes redness
and edema. The pedal pulse is palpable and unchanged from
admission. The nurse interprets that the neurovascular status is:
a. Slightly deteriorating and should be monitored for another hour
b. Moderately impaired, and the surgeon should be called
c. Normal because of increased blood flow through the leg
d. Adequate from an arterial approach, but venous complications are
arising.
805. A client comes into the E.R. with acute shortness of breath and a
cough that produces pink, frothy sputum. Admission assessment
reveals crackles and wheezes, a BP of 85/46, a HR of 122 BPM,
and a respiratory rate of 38 breaths/minute. The client’s medical
history included DM, HTN, and heart failure. Which of the
following disorders should the nurse suspect?
a. Pneumonia
b. Pneumothorax
‫مناقشة امتحانات البرومتريك للتمريض‬

https://www.facebook.com/groups/1639727976293954 Page 330


Prometric exam collection 2017

)‫)النسخة الرابعة‬

4‫ودالشمال‬

c. Pulmonary edema
d. Pulmonary embolism
806. 55-year-old client is admitted with an acute inferior-wall
myocardial infarction. During the admission interview, he says he
stopped taking his metoprolol (Lopressor) 5 days ago because he
was feeling better. Which of the following nursing diagnoses takes
priority for this client?
a. Ineffective tissue perfusion
b. ineffective airway clearance
c. Ineffective therapeutic regimen management
d. Ineffective communication pattern
807. Which of the following would the nurse identify as the priority
nursing diagnosis during a toddler’s vasoocclusive sickle cell
crisis?
a. Pain related to tissue anoxia
b. Pain related to fear of unknown
c. Pain related to sever anxiety
d. Pain related to increased cardiac output
808. The client experiencing 7th cranial nerve (facial Nerve ) damage
will most likely report which of the following symptoms?
‫مناقشة امتحانات البرومتريك للتمريض‬

https://www.facebook.com/groups/1639727976293954 Page 331


Prometric exam collection 2017

)‫)النسخة الرابعة‬

4‫ودالشمال‬

a. Bell's palsy
b. Vertigo
c. Impaired vision
d. Headache
809. A client with COPD has developed secondary polycythemia.
Which nursing diagnosis would be included in the plan of care
because of the polycythemia?
a. Impaired tissue perfusion related to thrombosis
b. Activity intolerance related to dyspnea
c. Impaired tissue perfusion related to decrease cardiac output
d. Impaired tissue perfusion related to blood loss
810. A female client comes into the emergency room complaining of
SOB and pain in the lung area. She states that she started taking
birth control pills 3 weeks ago and that she smokes. Her VS are:
BP :140/80, Pulse 110, R 40.
The physician orders ABG’s, results are as follows:
pH: 7.50
PaCO2 29 mm Hg
PaO2 60 mm Hg
HCO3– 24 mEq/L
‫مناقشة امتحانات البرومتريك للتمريض‬

https://www.facebook.com/groups/1639727976293954 Page 332


Prometric exam collection 2017

)‫)النسخة الرابعة‬

4‫ودالشمال‬

SaO2 86%.
Considering these results, the first intervention is to:
a. Place the client on oxygen
b. Give the client sodium bicarbonate
c. Begin mechanical ventilation
d. Monitor for pulmonary embolism
811. A nurse is preparing to obtain an arterial blood gas specimen from
a client and plans to perform the Allen’s test on the client. after
explaining the procedure to patient, what is the next step nurse
need to do for performing the Allen’s test ?.
a. Ask the client to open and close the hand repeatedly.
b. Assess the color of the extremity distal to the pressure point
c. Release pressure from the ulnar artery
d. Apply pressure over the ulnar and radial arteries.
812. A 34-year-old woman with a history of asthma is admitted to the
emergency department. The nurse notes that the client is dyspneic,
with a respiratory rate of 35 breaths/minute, nasal flaring, and use
of accessory muscles. Auscultation of the lung fields reveals
greatly diminished breath sounds. Based on these findings, what
action should the nurse take to initiate care of the client?
‫مناقشة امتحانات البرومتريك للتمريض‬

https://www.facebook.com/groups/1639727976293954 Page 333


Prometric exam collection 2017

)‫)النسخة الرابعة‬

4‫ودالشمال‬

a. Administer bronchodilators
b. Initiate oxygen therapy and reassess the client in 10 minutes.
c. Draw blood for an ABG analysis and send the client for a chest x-
ray.
d. Encourage the client to relax and breathe slowly through the mouth
813. A couple has brought in their Toddler daughter for examination.
The parents tell the nurse that they are worried about all the safety
risks for this age group. As the nurse plans to teach the parents
about these risks, the nurse remembers that toddler are at a greater
risk for injury from:
a. Poisoning and child abduction
b. Home accidents
c. Physiological changes of aging
d. Automobile accidents, suicide, and substance abuse

814. When providing health maintenance teaching to new employees in


the food-handling department, the nurse emphasizes the need to
perform hand hygiene after using the bathroom to prevent:
a. Spread of hepatitis A
b. Spread of hepatitis E
‫مناقشة امتحانات البرومتريك للتمريض‬

https://www.facebook.com/groups/1639727976293954 Page 334


Prometric exam collection 2017

)‫)النسخة الرابعة‬

4‫ودالشمال‬

c. Salmonella contamination
d. Cholera contamination

815. As a member of the hospital's bioterrorism team, the nurse


understands the importance of knowing how an organism is
transmitted. Smallpox has the potential to spread quickly because it
is transmitted via which route?
a. Air-borne
b. Droplets
c. Contact
d. Absorption

816. A nurse plans to teach a group of 20 to 25 year-old women about


oral contraceptives. The nurse should instruct that side effects of
intrauterine device may include contraceptive may cause:
a. Increase risk of pelvic inflammatory disease
b. Cause acne to worsen
c. Decrease the risk of breast and cervical cancer
d. Decrease the risk of endometriosis

‫مناقشة امتحانات البرومتريك للتمريض‬

https://www.facebook.com/groups/1639727976293954 Page 335


Prometric exam collection 2017

)‫)النسخة الرابعة‬

4‫ودالشمال‬

817. A community health nurse assesses a68 year-old patient who lives
in a group home. During physical assessment nurse notice that
patient skin and mucus membrane are dry and and pale. What type
of data is the nurse collecting from the above information?
a. Subjective
b. Objective
c. Medical History
d. Analytical

‫مع تمنياتي بالتوفيق والنجاح للجميع‬


‫م‬2017‫ودالشمال‬

‫مناقشة امتحانات البرومتريك للتمريض‬

https://www.facebook.com/groups/1639727976293954 Page 336

You might also like